Currently Empty: ₹0.00
Rheumatology Module July 2022 MRCP Part-1
Time limit: 0
Quiz Summary
0 of 114 Questions completed
Questions:
Information
You have already completed the quiz before. Hence you can not start it again.
Quiz is loading…
You must sign in or sign up to start the quiz.
You must first complete the following:
Results
Quiz complete. Results are being recorded.
Results
0 of 114 Questions answered correctly
Your time:
Time has elapsed
You have reached 0 of 0 point(s), (0)
Earned Point(s): 0 of 0, (0)
0 Essay(s) Pending (Possible Point(s): 0)
Categories
- Rheumatology Module July 2022 MRCP Part-1 0%
- 1
- 2
- 3
- 4
- 5
- 6
- 7
- 8
- 9
- 10
- 11
- 12
- 13
- 14
- 15
- 16
- 17
- 18
- 19
- 20
- 21
- 22
- 23
- 24
- 25
- 26
- 27
- 28
- 29
- 30
- 31
- 32
- 33
- 34
- 35
- 36
- 37
- 38
- 39
- 40
- 41
- 42
- 43
- 44
- 45
- 46
- 47
- 48
- 49
- 50
- 51
- 52
- 53
- 54
- 55
- 56
- 57
- 58
- 59
- 60
- 61
- 62
- 63
- 64
- 65
- 66
- 67
- 68
- 69
- 70
- 71
- 72
- 73
- 74
- 75
- 76
- 77
- 78
- 79
- 80
- 81
- 82
- 83
- 84
- 85
- 86
- 87
- 88
- 89
- 90
- 91
- 92
- 93
- 94
- 95
- 96
- 97
- 98
- 99
- 100
- 101
- 102
- 103
- 104
- 105
- 106
- 107
- 108
- 109
- 110
- 111
- 112
- 113
- 114
- Current
- Review
- Answered
- Correct
- Incorrect
-
Question 1 of 114
1. Question
Acute iritis is associated with which of one the following diagnoses?
CorrectIncorrectHint
Iritis is associated with conditions such as Reactive arthritis, Behcet’s, psoriatic arthropathy (about 20%) and inflammatory bowel disease.
_x000D_
A chronic iritis is rarely described in association with Lyme disease.
_x000D_
Osteogenesis imperfecta is associated with blue sclera.
_x000D_
Keratoconus, meaning “cone shaped,” describes a condition in which the cornea (the clear front window of the eye) becomes thin and protrudes. This abnormal shape can cause serious distortion of visual images. It is not associated with iritis.
_x000D_
Refsum’s disease is associated with retinitis pigmentosa.
-
Question 2 of 114
2. Question
A 72-year-old gentleman presents to the Outpatient Department with hyperosmolar nonketotic hyperglycaemia. He has red and hot swelling in both of his knees.
Which of the following is most useful in the diagnosis of the swollen knee joint?CorrectIncorrectHint
Joint aspiration is the best option in this context. It is a simple procedure with a high diagnostic yield. A destructive septic arthritis is a potential diagnosis and it must be excluded as a matter of urgency. Joint aspirate is the most crucial step in excluding a septic arthritis, and allows the timely commencement of surgical management and appropriate antibiotics.
_x000D_
Sending the joint aspiration for M/C/S in a blood culture bottle may increase yield. Whilst joint washout is appropriate management of septic arthritis, a diagnosis must be made prior to this being considered.
_x000D_
The risk of introducing infection into the knee joint during simple aspiration by non-experts is 1 in 10,000 procedures, so the procedure is safe.
-
Question 3 of 114
3. Question
Regarding infliximab, which one of the following statements is most true?
CorrectIncorrectHint
According to the National Institute for Health and Care Excellence (NICE) guidelines, the statement about infliximab that is most true is: C. Is licensed for the treatment of rheumatoid arthritis.
_x000D_
Infliximab is indeed licensed for the treatment of rheumatoid arthritis, among other conditions. It is a tumour necrosis factor (TNF)-alpha inhibitor used in the management of autoimmune diseases by reducing inflammation.
_x000D_
Now, let us address the other options:
_x000D_
- _x000D_
- Must not be used in combination with methotrexate due to increased toxicity: This statement is incorrect. Infliximab is often used in combination with methotrexate to reduce the formation of anti-drug antibodies and increase the efficacy of treatment in conditions like rheumatoid arthritis.
- Is first line treatment for ulcerative colitis: Infliximab is not the first line of treatment for ulcerative colitis. It is considered for patients with moderate to severe ulcerative colitis when conventional therapy is not suitable or has not worked.
- Is a monoclonal antibody to the glycoprotein IIb-IIIa receptor: This statement is incorrect. Infliximab is a monoclonal antibody against TNF-alpha, not the glycoprotein IIb-IIIa receptor.
- It prevents relapse of Crohn’s disease in patients who are in remission: While infliximab is used in the management of Crohn’s disease, the National Institute for Health and Care Excellence (NICE) guidelines specifically recommend it for treating severe active Crohn’s disease and not explicitly for preventing relapse in patients who are in remission.
_x000D_
_x000D_
_x000D_
_x000D_
_x000D_
In summary, option C is the most accurate statement regarding infliximab according to the National Institute for Health and Care Excellence (NICE) guidelines.
-
Question 4 of 114
4. Question
Which one of the following is a recognised feature of polymyalgia rheumatica (PMR)?
CorrectIncorrectHint
The correct option for a recognized feature of polymyalgia rheumatica (PMR) is:
_x000D_
- _x000D_
- Weight loss.
_x000D_
_x000D_
Weight loss is a recognized systemic symptom associated with polymyalgia rheumatica (PMR), along with other constitutional symptoms like fever and fatigue.
_x000D_
Now, let us rule out the incorrect options:
_x000D_
- _x000D_
- Elevated serum creatine phosphokinase activity: This is incorrect because polymyalgia rheumatica (PMR) typically does not involve elevated creatine phosphokinase levels, which would suggest muscle damage rather than the inflammation characteristic of polymyalgia rheumatica (PMR).
- An association with bronchial carcinoma: There is no recognized association between PMR and bronchial carcinoma. Polymyalgia rheumatica (PMR) is primarily an inflammatory condition affecting the shoulders, hip girdle, and neck.
- A peak incidence in the fourth decade of life: Polymyalgia rheumatica (PMR) most commonly affects individuals over the age of 50, with the peak incidence occurring between ages 70 and 80.
- Weakness of distal muscle groups: Polymyalgia rheumatica (PMR) is characterized by aching and morning stiffness in the proximal muscle groups, such as the shoulders and hips, rather than weakness in the distal muscles.
_x000D_
_x000D_
_x000D_
_x000D_
_x000D_
In summary, weight loss is a recognized feature of polymyalgia rheumatica (PMR), reflecting the systemic nature of the disease. The other options listed do not accurately represent the features of polymyalgia rheumatica (PMR).
-
Question 5 of 114
5. Question
Which one of the following drugs works by inhibiting the Tumour Necrosis Factor (TNF)?
CorrectIncorrectHint
Montelukast works as a leukotriene receptor antagonist and is used in the treatment of asthma.
_x000D_
Etanercept and infliximab inhibit TNF and are licensed in the treatment of rheumatoid arthritis.
_x000D_
Infliximab is given with methotrexate and is associated with the development of tuberculosis.
-
Question 6 of 114
6. Question
Which of the following drugs is most likely to cause Drug-Induced Lupus Erythematosus (DILE) Syndrome?
CorrectIncorrectHint
A recessive gene is responsible for the activity of hepatic N-acetyl transferase resulting in slow or fast (intermediate and fast groups get lumped together) acetylation.
_x000D_
45% of the United Kingdom population are slow acetylators.
_x000D_
Drugs affected include:
_x000D_
- _x000D_
- isoniazid
- hydralazine
- dapsone
- procainamide, and
- sulphasalazine.
_x000D_
_x000D_
_x000D_
_x000D_
_x000D_
_x000D_
Slow acetylators have increased risk of isoniazid-induced peripheral neuropathy, and hydralazine or procainamide-induced systemic lupus erythematosus (SLE).
_x000D_
At least 38 drugs currently in use can cause DILE. However, most cases have been associated with these three:
_x000D_
- _x000D_
- procainamide
- hydralazine, and
- quinidine.
_x000D_
_x000D_
_x000D_
_x000D_
The risk for developing lupus-like disease from any of the other 35 drugs is low or very low; with some drugs only one or two cases have been reported.
_x000D_
- _x000D_
- Isoniazid (INH) – low risk
- Sulfasalazine – low risk.
_x000D_
_x000D_
-
Question 7 of 114
7. Question
Which one of the following is not liable for the development of Systemic Lupus Erythematosus (SLE)?
CorrectIncorrectHint
The specific cause of SLE is unknown, but multiple genetic predispositions and environmental factors have been identified. The underlying mechanism is thought to be associated with a deficiency in immune clearance, which results in autoimmunity.
Patients with HLA-DR3, HLA-A1, and HLA-B8 are throught to have an innate susceptibility, as are those with abnormal complement levels (especially congenital deficiencies of C4 or C2) or specific immunoregulatory genotypes (mainly related to B-cell receptor pathway signalling). UV exposure, microbial response and drugs can then interact with this innate susceptibility to result in the development of SLE.
_x000D_
There is an increased risk in relatives of those affected, with 40% concordance in monozygotic twins.
The role of infection in the aetiology of SLE has been extensively investigated, and it has been found that patients have higher antibody titres against EBV than the general population which is postulated to result in a T-cell abnormality with causes failure of the normal regulation of the B-cell response. In addition, chronic infections can result in anti-DNA antibodies and bacterial infections can precipitate a flare of SLE symptoms.
_x000D_
Environmental factors are more complex, but some associations have been found:
Silica dust, exogenous oestrogen administration and cigarette smoking may increase the risk of developing SLE.
Breastfeeding seems to decrease the risk of developing SLE.
UV light is a clear precipitant of skin disease, by stimulating keratinocytes resulting in overexpression of surface nuclear ribonucleoproteins and cytokine secretion with subsequent increased autoantibody production.
Vitamin D deficiency may also result in increased autoantibody production. -
Question 8 of 114
8. Question
A 17-year-old boy comes to the Clinic with complaint of feeling unwell. He has recently suffered from a streptococcal throat infection but feels that he has not really improved since then, although he has completed a course of oral penicillin.
_x000D_
He complains of an extensive purpuric rash involving his buttocks, the back of his legs, and the ulnar side of his arms. He has a history of abdominal pain and joint pains as well.
_x000D_
On examination his blood pressure is 100/70 mmHg, with a pulse rate of 74/minute. He has a purpuric rash, mainly affecting his buttocks and the tops of his legs.
_x000D_
Investigations Show:
_x000D_
_x000D_ _x000D_
_x000D_ _x000D_ Haemoglobin
_x000D_
_x000D_
_x000D_ 112 g/L
_x000D_
_x000D_
_x000D_ (115-160)
_x000D_
_x000D_
_x000D_
_x000D_ _x000D_ Total Leucocyte Count
_x000D_
_x000D_
_x000D_ 11.4 ×109/L
_x000D_
_x000D_
_x000D_ (4-11)
_x000D_
_x000D_
_x000D_
_x000D_ _x000D_ Platelet Count
_x000D_
_x000D_
_x000D_ 234 ×109/L
_x000D_
_x000D_
_x000D_ (150-400)
_x000D_
_x000D_
_x000D_
_x000D_ _x000D_ Serum Sodium
_x000D_
_x000D_
_x000D_ 141 mmol/L
_x000D_
_x000D_
_x000D_ (135-146)
_x000D_
_x000D_
_x000D_
_x000D_ _x000D_ Serum Potassium
_x000D_
_x000D_
_x000D_ 4.1 mmol/L
_x000D_
_x000D_
_x000D_ (3.5-5)
_x000D_
_x000D_
_x000D_
_x000D_ _x000D_ Serum Creatinine
_x000D_
_x000D_
_x000D_ 138 μmol/L
_x000D_
_x000D_
_x000D_ (79-118)
_x000D_
_x000D_
_x000D_
_x000D_ _x000D_ Dipstick Urine
_x000D_
_x000D_
_x000D_ Blood 2+; Protein 2+
_x000D_
_x000D_
_x000D_
_x000D_
_x000D_
Which one of the following immunoglobulins is most likely to be raised?
CorrectIncorrectHint
This boy has Henoch-Schönlein purpura (HSP). HSP is more commonly seen in infants, but may also be seen in older individuals.
_x000D_
The buttock rash seen here is characteristic as is the blood and proteinuria. Creatinine may be elevated, although progression to renal failure is rare. Platelet count can be low, normal or elevated. IgA levels are elevated and the patient may also have an eosinophilia.
_x000D_
HSP is usually managed with supportive measures only. There is no evidence that steroid therapy impacts on progression to renal failure.
_x000D_
Non-steroidals may be of value in treating joint pain.
-
Question 9 of 114
9. Question
A 23-year-old lady was started on minocycline for the treatment of her acne.
_x000D_
A week later she presented with fever, myalgia, arthralgia and a fixed erythematous rash over the malar eminences that spared the nasolabial folds.
_x000D_
Which autoantibody test would confirm the diagnosis?
CorrectIncorrectHint
This lady has developed symptoms suggestive of drug-induced lupus erythematosus (DILE) after starting minocycline for her acne treatment. The autoantibody test that would help to confirm the diagnosis is E. Antihistone.
_x000D_
Antihistone antibodies are the hallmark of drug-induced lupus. In the context of minocycline use, these antibodies are frequently present and can help confirm the diagnosis of drug-induced lupus erythematosus (DILE). Minocycline is a well-known trigger for this condition, and the presence of antihistone antibodies in this scenario would be supportive of drug-induced lupus erythematosus (DILE).
_x000D_
The other options are incorrect for the following reasons:
_x000D_
- _x000D_
- Anti-dsDNA: Anti-double-stranded DNA (dsDNA) antibodies are highly specific for systemic lupus erythematosus (SLE) rather than drug-induced lupus. While they may be present in some cases of drug-induced lupus erythematosus (DILE), they are not the primary antibodies used for confirmation of the diagnosis.
- Anti-RNP: Anti-ribonucleoprotein (RNP) antibodies are associated with mixed connective tissue disease (MCTD) and are not typically seen in drug-induced lupus erythematosus (DILE). They are also not specific for the condition and would not be the best choice for confirming a diagnosis of drug-induced lupus erythematosus (DILE).
- Anti-Scl-70: Anti-Scl-70 antibodies, also known as anti-topoisomerase I antibodies, are associated with systemic sclerosis (scleroderma) and are not indicative of drug-induced lupus erythematosus (DILE). These antibodies are not relevant to this lady’s presentation and would not be useful in confirming a diagnosis of drug-induced lupus erythematosus (DILE).
- Anti-Jo-1: Anti-Jo-1 antibodies are specific for polymyositis and dermatomyositis and are not associated with drug-induced lupus erythematosus (DILE). They would not be appropriate for confirming a diagnosis in a patient with suspected drug-induced lupus erythematosus (DILE) following minocycline therapy.
_x000D_
_x000D_
_x000D_
_x000D_
_x000D_
In summary, the clinical presentation of fever, myalgia, arthralgia, and a characteristic rash, along with the recent use of minocycline, points towards a diagnosis of drug-induced lupus erythematosus (DILE). The detection of antihistone antibodies would be the most confirmatory test for this diagnosis, making option E the correct answer. The other options listed are associated with different autoimmune conditions and are not the best tests for confirming drug-induced lupus erythematosus (DILE) in a patient with this clinical presentation and drug history. The diagnosis of drug-induced lupus erythematosus (DILE) is further supported by the resolution of symptoms upon cessation of the offending drug, which in this case would be minocycline.
-
Question 10 of 114
10. Question
A 22-year-old young rugby player is diagnosed with chronic fatigue syndrome.
_x000D_
Which one of the following treatments is necessary?
CorrectIncorrectHint
NICE have published guidance on the diagnosis and management of Chronic fatigue syndrome/myalgic encephalomyelitis (or encephalopathy) (CG53). To confirm a diagnosis of fatigue the following main features need to be present:
_x000D_
- _x000D_
- It must be new in onset, persistent or recurrent and unexplained by other conditions.
- It should be characterised by post-exertional malaise.
- It should result in a substantial reduction in activity level.
_x000D_
_x000D_
_x000D_
_x000D_
Associated symptoms include:
_x000D_
- _x000D_
- Hypersomnia or insomnia
- Muscle or joint pain without inflammation
- Painful lymph nodes without lymphadenopathy
- Headaches
- Cognitive dysfunction.
_x000D_
_x000D_
_x000D_
_x000D_
_x000D_
_x000D_
Red flag symptoms which suggest another diagnosis include:
_x000D_
- _x000D_
- Significant weight loss
- Inflammatory arthropathy or connective tissue disease
- Localising or focal neurological signs.
_x000D_
_x000D_
_x000D_
_x000D_
The diagnosis of CFS is one of exclusion, and features must have been present for at least four months in an adult.
_x000D_
Clinicians should check:
_x000D_
- _x000D_
- Full blood count (FBC)
- Urea and electrolytes (U&Es)
- Urinalysis
- Liver function tests (LFTs)
- Thyroid function
- Erythrocyte sedimentation rate (ESR)
- C reactive protein (CRP)
- Blood glucose
- Creatinine
- Gluten sensitivity calcium
- Creatinine kinase, and
- Ferritin.
_x000D_
_x000D_
_x000D_
_x000D_
_x000D_
_x000D_
_x000D_
_x000D_
_x000D_
_x000D_
_x000D_
_x000D_
_x000D_
Initial treatment should focus on management of symptoms, and minimising their impact on daily activities. Patients must be encouraged to continue work and studies. Any therapy should be person-centred and should aim to improve the patient’s capacity to manage their symptoms.
_x000D_
The majority of research evidence is for cognitive behavioural therapy (CBT) and/or graded exercise therapy and these should be offered to all people with mild or moderate CFS.
_x000D_
In addition, patients should be given tailored sleep management advice including how to introduce rest periods into their daily routine. Relaxation techniques should be offered for the management of pain, sleep problems, stress and anxiety.
_x000D_
There is no research evidence to support the experience of some patients with CFS that they are more intolerant of drug treatment. In addition, there is insufficient evidence to recommend the use of complementary therapies or vitamin supplementation.
_x000D_
The following drugs should not be used:
_x000D_
- _x000D_
- Monoamine oxidase inhibitors
- Glucocorticoids
- Mineralocorticoids
- Dexamphetamine
- Thyroxine
- Antivirals.
_x000D_
_x000D_
_x000D_
_x000D_
_x000D_
_x000D_
_x000D_
Referral to specialist CFS care should be offered within six months of presentation to people with mild CFS, within three to four months for moderate CFS and immediately for severe CFS.
_x000D_
If chronic pain is a predominant feature, referral to a pain management clinic should be considered. Amitriptlyline should be considered for patients with poor sleep or pain.
_x000D_
Patients should be advised that relapses and setbacks are to be expected.
-
Question 11 of 114
11. Question
A 22-year-old student presents to the Clinic with typical erythema nodosum. She has a low grade fever and bilateral ankle arthritis but no other symptoms and has no medical history of note. She has no history of travelling abroad and she is on no regular medication.
_x000D_
Which one of the following would be the most appropriate investigation for this lady?
CorrectIncorrectHint
Erythema nodosum is commonly idiopathic.
_x000D_
It can also be related to streptococcal infections, acute sarcoidosis or related to drugs such as the oral contraceptive pill, sulphonamides and penicillins.
_x000D_
Rarer causes include inflammatory bowel disease, tuberculosis, Behçet’s disease and other connective tissue disorders.
_x000D_
In this case, a chest x ray would be the most helpful investigation as this may identify bilateral hilar lymphadenopathy which together with a bilateral ankle arthropathy would strongly support a diagnosis of acute sarcoidosis.
_x000D_
Investigation of the bowel is unlikely to help in the absence of any bowel symptoms.
_x000D_
Viral titres and ESR are non-specific.
-
Question 12 of 114
12. Question
A 22-year-old lady is taking oral medication for management of acne. She develops polyarthritis and has raised liver function enzymes.
_x000D_
Investigations Show:
_x000D_
_x000D_ _x000D_
_x000D_ _x000D_ Serum Aspartate Aminotransferase (AST)
_x000D_
_x000D_
_x000D_ 97 U/L
_x000D_
_x000D_
_x000D_ (1-31)
_x000D_
_x000D_
_x000D_
_x000D_ _x000D_ Serum Alanine Aminotransferase (ALT)
_x000D_
_x000D_
_x000D_ 168 U/L
_x000D_
_x000D_
_x000D_ (5-35)
_x000D_
_x000D_
_x000D_
_x000D_ _x000D_ Serum Bilirubin
_x000D_
_x000D_
_x000D_ 15 µmol/L
_x000D_
_x000D_
_x000D_ (1-22)
_x000D_
_x000D_
_x000D_
_x000D_ _x000D_ Antinuclear Antibodies (ANA)
_x000D_
_x000D_
_x000D_ Strongly Positive At 1/20
_x000D_
_x000D_
_x000D_
_x000D_ _x000D_ Negative At 1/640
_x000D_
_x000D_
_x000D_
_x000D_
_x000D_
Which one of the following drugs is she most likely to have been advised for her acne?
CorrectIncorrectHint
The correct answer is D. Minocycline.
_x000D_
Minocycline is a tetracycline antibiotic that is used to treat acne and other bacterial infections. It is one of the drugs that can cause drug-induced lupus erythematosus (DIL), a condition that mimics the symptoms of systemic lupus erythematosus (SLE), such as polyarthritis, fatigue, serositis, and positive antinuclear antibodies (ANA).
_x000D_
The other options are incorrect for the following reasons:
_x000D_
- _x000D_
- A. Erythromycin is a macrolide antibiotic that is also used to treat acne and other infections. It is not associated with drug-induced lupus erythematosus (DIL), but it can cause liver toxicity, especially when combined with other drugs that are metabolized by the liver.
- B. Oxytetracycline is another tetracycline antibiotic that is used to treat acne and other infections. It is also not associated with drug-induced lupus erythematosus (DIL), but it can cause photosensitivity, gastrointestinal disturbances, and tooth discolouration.
- C. Isotretinoin is a retinoid that is used to treat severe acne that does not respond to other treatments. It is not associated with drug-induced lupus erythematosus (DIL), but it can cause teratogenicity, depression, dryness, and elevated liver enzymes.
- E. Trimethoprim is a folate antagonist that is used to treat urinary tract infections and other infections. It is not associated with drug-induced lupus erythematosus (DIL), but it can cause megaloblastic anaemia, hyperkalaemia, and rash.
_x000D_
_x000D_
_x000D_
_x000D_
-
Question 13 of 114
13. Question
A 21-year-old gentleman presents to the Outpatient Department with a five week history of a painful swollen left knee. He had been treated for a sexually transmitted disease two months ago.
_x000D_
On examination, there was a large effusion in the left knee. Synovial fluid analysis revealed a white cell count of 14 ×109/L (4-11) but culture was negative.
_x000D_
Which one of the following organisms is the most likely cause of his pathological condition?
CorrectIncorrectHint
The most likely cause of this gentleman’s knee effusion, given the history of a sexually transmitted disease and the current presentation, is B. Neisseria gonorrhoeae. This organism can cause gonococcal arthritis, which is a rare complication of gonorrhoea and presents with symptoms such as a swollen, painful joint, and in this case, a large effusion in the knee. The synovial fluid analysis showing an elevated white cell count without bacterial growth in culture is also consistent with gonococcal arthritis, as the bacteria are fastidious and may not grow in standard cultures.
_x000D_
Now, let us rule out the other options:
_x000D_
- _x000D_
- Trichomonas vaginalis is a protozoan that causes trichomoniasis, primarily affecting the urogenital tract. It does not typically cause joint effusions or systemic manifestations.
- Herpes simplex virus causes mucocutaneous infections and can lead to severe complications like encephalitis and neonatal herpes, but it is not known to cause joint effusions or septic arthritis.
- Treponema pallidum, the causative agent of syphilis, can cause a variety of symptoms including musculoskeletal manifestations in tertiary syphilis. However, the presentation of knee effusion in the context of recent sexually transmitted disease (STD) treatment makes acute syphilis less likely.
- Human papilloma virus (HPV) is associated with warts and certain cancers, but it does not cause joint effusions or septic arthritis.
_x000D_
_x000D_
_x000D_
_x000D_
_x000D_
Given the clinical context and the available information, Neisseria gonorrhoeae is the most likely cause of this gentleman’s condition.
-
Question 14 of 114
14. Question
A 21-year-old gentleman presents to the pain management clinic with a two-month history of acute polyarthralgia and pronounced whole body stiffness in the early mornings.
_x000D_
His symptoms respond well to aceclofenac but he is becoming increasingly worried about his symptoms which appear to be gradually progressing and hindering his daily life activities. He is elseways well apart from a history of acne which is well controlled on minocycline. His father has severe rheumatoid arthritis.
_x000D_
Investigations Show:
_x000D_
_x000D_ _x000D_
_x000D_ _x000D_ Erythrocyte Sedimentation Rate (ESR)
_x000D_
_x000D_
_x000D_ 52 mm/hr
_x000D_
_x000D_
_x000D_ (0-20)
_x000D_
_x000D_
_x000D_
_x000D_ _x000D_ Serum C-Reactive Protein (CRP)
_x000D_
_x000D_
_x000D_ 101 mg/L
_x000D_
_x000D_
_x000D_ (<10)
_x000D_
_x000D_
_x000D_
_x000D_ _x000D_ Rheumatoid Factor
_x000D_
_x000D_
_x000D_ Negative
_x000D_
_x000D_
_x000D_
_x000D_ _x000D_ Antinuclear Antibody (ANA)
_x000D_
_x000D_
_x000D_ Strongly Positive (1:1590)
_x000D_
_x000D_
_x000D_
_x000D_ _x000D_ Anti-dsDNA Antibodies
_x000D_
_x000D_
_x000D_ Negative
_x000D_
_x000D_
_x000D_
_x000D_ _x000D_ Serum IgG
_x000D_
_x000D_
_x000D_ 23 g/L
_x000D_
_x000D_
_x000D_ (<15)
_x000D_
_x000D_
_x000D_
_x000D_
_x000D_
Which one of the following is the most likely cause of his pathology?
CorrectIncorrectHint
Given the clinical presentation and laboratory findings, the most likely diagnosis for this gentleman is D. Drug-induced systemic lupus erythematosus (SLE). This gentleman’s symptoms and the strongly positive antinuclear antibody (ANA) test, along with a negative anti-dsDNA, suggest systemic lupus erythematosus (SLE). However, the history of being well-controlled on minocycline, which is known to potentially induce systemic lupus erythematosus (SLE), points towards drug-induced systemic lupus erythematosus (SLE).
_x000D_
The other options are less likely as discussed below:
_x000D_
- _x000D_
- A. Systemic lupus erythematosus (SLE): While this gentleman’s symptoms and laboratory results could suggest systemic lupus erythematosus (SLE), the history of minocycline use makes drug-induced systemic lupus erythematosus (SLE) more likely.
- B. Seronegative spondyloarthropathy: This group of disorders typically presents with negative rheumatoid factor and antinuclear antibody (ANA) tests, which does not align with this gentleman’s strongly positive antinuclear antibody (ANA).
- C. Fibromyalgia: This condition is characterized by widespread pain and tenderness but does not usually present with significant laboratory abnormalities such as elevated erythrocyte sedimentation rate (ESR) or C-reactive protein (CRP) levels.
- E. Rheumatoid arthritis: This gentleman’s negative rheumatoid factor and the absence of symptoms typical of rheumatoid arthritis, such as symmetric joint pain, make this diagnosis less likely. Additionally, rheumatoid arthritis would not explain the high IgG levels or the strongly positive antinuclear antibody (ANA) test.
_x000D_
_x000D_
_x000D_
_x000D_
_x000D_
In conclusion, this gentleman’s clinical and laboratory findings are most consistent with D. Drug-induced systemic lupus erythematosus (SLE), likely related to the use of minocycline.
-
Question 15 of 114
15. Question
A 21-year-old lady presents to the rheumatology clinic for review. She complains of progressively increasing exhaustion, loss of weight and low grade fevers over the last three months and is worried that she may have an autoimmune problem. She also has dull abdominal pain after having meals, and trouble with aching arms whilst playing at the volleyball court over the past few months. She has fainted on one occasion whilst cleaning her flat. On examination she looks pale and grey. Her blood pressure is 130/90 mmHg in the left arm and 100/80 mmHg in the right arm; Heart sounds are normal and the chest is clear; Abdomen is soft and non-tender and her body mass index (BMI) is 20 kg/m2. During examination, when she was asked to hold her hands above her head, the radial pulse becomes absent in the left arm and is barely palpable in the right. There is synovitis affecting both the knees and ankles. Her erythrocyte sedimentation rate (ESR) and serum C-reactive protein (CRP) are elevated.
_x000D_
Which one of the following is the most likely diagnosis?
CorrectIncorrectHint
The most likely diagnosis is E. Takayasu’s arteritis. This is a rare type of vasculitis that affects the large arteries, especially the aorta and its branches. It can cause symptoms such as fatigue, fever, weight loss, abdominal pain, arm claudication, pulselessness, hypertension, and joint inflammation. The diagnosis is supported by the presence of a blood pressure difference between the arms, the diminished or absent radial pulses, the elevated erythrocyte sedimentation rate (ESR) and C-reactive protein (CRP), and the young age and female gender of the patient.
_x000D_
The other options are less likely for the following reasons:
_x000D_
- _x000D_
- A. Fibromuscular dysplasia: This is a condition that causes narrowing or enlargement of medium and large arteries. It can affect the renal, carotid, mesenteric, coronary, and peripheral arteries, and cause symptoms such as high blood pressure, headache, tinnitus, neck pain, abdominal pain, chest pain, and limb pain. However, this condition does not cause fever, weight loss, joint inflammation, or elevated erythrocyte sedimentation rate (ESR) and C-reactive protein (CRP). It is also more common in older adults than in young people.
- B. Temporal arteritis: This is a form of vasculitis that affects the temporal arteries and other cranial arteries. It can cause symptoms such as headache, scalp tenderness, jaw pain, vision loss, and fever. However, this condition does not affect the aorta or its branches, and does not cause blood pressure difference, pulselessness, or joint inflammation. It is also very rare in young people, and more common in people over 50 years of age.
- C. Juvenile chronic arthritis: This is a term for several types of arthritis that affect children under 16 years of age. It can cause joint inflammation, stiffness, pain, and swelling. However, this condition does not affect the large arteries, and does not cause blood pressure difference, pulselessness, or abdominal pain. It may cause fever, weight loss, and elevated erythrocyte sedimentation rate (ESR) and C-reactive protein (CRP), but these are usually mild and transient.
- D. Kawasaki’s disease: This is a rare condition that causes inflammation of the blood vessels, especially the coronary arteries. It mainly affects children under 5 years of age. It can cause symptoms such as fever, rash, red eyes, swollen lymph nodes, red lips and tongue, and peeling skin. However, this condition does not cause blood pressure difference, pulselessness, or joint inflammation. It may cause abdominal pain, but this is usually mild and transient. It is also very unlikely to affect young adults.
_x000D_
_x000D_
_x000D_
_x000D_
-
Question 16 of 114
16. Question
A 19-year-old student had completed treatment for acute lymphoblastic leukaemia eight months previously. He now presents to the Outpatient Department with a short history of acute left hip pain associated with drooping.
_x000D_
Which one of the following is the most likely diagnosis?
CorrectIncorrectHint
Given the history of treatment for acute lymphoblastic leukaemia (ALL) and the presentation of acute left hip pain, the most likely diagnosis is avascular necrosis of the femoral head. Avascular necrosis (AVN) is a condition where bone tissue dies due to a lack of blood supply, and it is a known complication of acute lymphoblastic leukaemia (ALL) treatment due to the use of corticosteroids and chemotherapy, which can affect blood supply to the bones.
_x000D_
The other options are incorrect for the following reasons:
_x000D_
- _x000D_
- Pseudogout – This condition involves the deposition of calcium pyrophosphate crystals within the joint. It typically presents with sudden, painful swelling in a joint, often the knee, rather than the hip. It is associated with metabolic conditions like hypercalcaemia and is less likely to be related to acute lymphoblastic leukaemia (ALL) treatment.
- Gout – Gout is caused by the deposition of uric acid crystals in the joint, leading to inflammation. It most commonly affects the big toe and is associated with high levels of uric acid in the blood. While gout can cause acute joint pain, it is not typically associated with acute lymphoblastic leukaemia (ALL) treatment.
- Septic arthritis – This is an infection within the joint space that causes severe pain, redness, swelling, and fever. It can occur after joint surgery or in immunocompromised individuals but is less likely in this scenario without evidence of an overlying infection or penetrating injury.
- Osteoarthritis – This is a degenerative joint disease that typically affects older adults and is characterized by the gradual wearing down of joint cartilage. It presents with chronic joint pain and stiffness, particularly with activity, and is unlikely in a young gentleman without a history of joint trauma or chronic stress on the joint.
_x000D_
_x000D_
_x000D_
_x000D_
_x000D_
In conclusion, the history of acute lymphoblastic leukaemia (ALL) treatment and the acute presentation of hip pain strongly suggest avascular necrosis of the femoral head as the most likely diagnosis.
-
Question 17 of 114
17. Question
A 28-year-old gentleman struggles to change his car gear with his right hand due to swelling over his thumb and a burning sensation along the radial side of his right wrist which is worsened on abducting the thumb. Finkelstein’s Test is positive.
_x000D_
Which one of the following is the most likely diagnosis?
CorrectIncorrectHint
Swelling and burning sensation over the thumb in a young individual with positive Finklestein’s test is characteristic of De Quervain’s tenosynovitis. Finklestein’s test is performed by grasping the thump and ulnar deviating the hand sharply. It is positive if sharp pain occurs along the distal radius.
_x000D_
De Quervain’s is caused by overuse of the thumb, as is the case here with this patient’s job. Thickening of the synovial sheaths surrounding extensor pollicis brevis and abductor polices longus leads to pain and reduced movement.
_x000D_
Osteoarthritis is a possibility but would be associated more commonly in older individuals, and tenderness is classically at the base of the thumb.
_x000D_
Carpal tunnel presents with pain and paraesthesia of the thumb, index, middle and half of the ring fingers on the affected side.
_x000D_
Radial tunnel syndrome presents with pain in the dorsal aspect of the upper forearm, and is caused by pressure on the radial nerve possibly from compression by the muscles of the forearm.
_x000D_
Rheumatoid arthritis presents with joint swelling and pain, with subsequent hand deformity if left untreated. It is classically symmetrical.
-
Question 18 of 114
18. Question
A 28-year-old Japanese gentleman presents with a three-year history of progressively worsening, lower back pain and stiffness. She denied any history of trauma.His pain radiates to the gluteal region bilaterally and is worse in the evenings. He reports some relief with exercise. Recently, he has also noted intermittent pains in his right shoulder and the heel of his right foot.
_x000D_
Clinical examination demonstrated limited spinal flexion in the sagittal and frontal planes. Right shoulder pain was reproducible with resisted abduction; there was a diminished right calf squeeze test with a tender and swollen right achilles tendon.
_x000D_
Given the probable diagnosis, which of the following is likely to be positive?
CorrectIncorrectHint
This question aims to cover the following learning points:
_x000D_
- _x000D_
- Distinguishing features of chronic inflammatory back pain
- Population specific HLA association
- Common differentials for ankylosing spondylitis.
_x000D_
_x000D_
_x000D_
_x000D_
This man has ankylosing spondylitis (AS).
_x000D_
The commonest subtype HLA associations are HLA B*2705 (Caucasians), B*2704 (Chinese, Japanese) and B*2702 (Mediterranean). The B*2706 subtype is weakly associated and commonly found in normal south east Asian individuals.
_x000D_
Chronic (more than three months) back pain characteristics that favour a spondyloarthritic aetiology include:
_x000D_
- _x000D_
- Age of onset before 40 years
- Insidious onset
- Amelioration with exercise
- Refractory with rest
- Night pain (with improvement upon arising).
_x000D_
_x000D_
_x000D_
_x000D_
_x000D_
_x000D_
His associated extra-articular manifestations include enthesitis of the Achilles and supraspinatus tendons.
-
Question 19 of 114
19. Question
A 27-year-old primigravida at 22 weeks of gestation attends the clinic for routine antenatal check-up. She is a known case of systemic lupus erythematosus (SLE).
_x000D_
Complete obstetric scan shows foetal heart rate of 45 beats per minute. Foetal echocardiography shows complete heart block.
_x000D_
Which one of the following maternal autoantibodies is most specifically responsible for this finding?
CorrectIncorrectHint
The correct answer is B. Anti-Ro (SSA) antibody. This is because maternal anti-Ro/SSA antibodies are the most common and specific autoantibodies associated with foetal congenital heart block (CHB). These antibodies cross the placenta and cause inflammation, cell death, and scarring of the fetal cardiac conduction system, leading to a block in signal transmission at the atrioventricular (AV) node.
_x000D_
The other answer options are incorrect because:
_x000D_
- _x000D_
- A. Anti-dsDNA antibody: This option is incorrect because anti-dsDNA antibodies are not associated with foetal congenital heart block (CHB). Anti-dsDNA antibodies are specific for systemic lupus erythematosus (SLE) and are involved in the pathogenesis of lupus nephritis, but they do not affect the foetal heart.
_x000D_
_x000D_
- _x000D_
- C. Anti-mitochondrial antibody (AMA): This option is incorrect because anti-mitochondrial antibodies (AMA) are not associated with foetal complete heart block (CHB). Anti-mitochondrial antibodies (AMA) are specific for primary biliary cirrhosis, a chronic liver disease, but they do not cross the placenta or target the foetal heart.
_x000D_
_x000D_
- _x000D_
- D. Anti-Jo 1 antibody: This option is incorrect because anti-Jo 1 antibodies are not associated with foetal congenital heart block (CHB). Anti-Jo 1 antibodies are specific for polymyositis and dermatomyositis, inflammatory muscle diseases, but they do not cross the placenta or target the foetal heart.
_x000D_
_x000D_
- _x000D_
- E. Anti-La (SSB) antibody: This option is incorrect because anti-La/SSB antibodies are not as specific as anti-Ro/SSA antibodies for foetal congenital heart block (CHB). Anti-La/SSB antibodies are often found together with anti-Ro/SSA antibodies in systemic lupus erythematosus (SLE) and Sjögren syndrome, but they are less frequently detected in mothers of children with congenital heart block (CHB). Moreover, anti-La/SSB antibodies alone are not sufficient to cause congenital heart block (CHB), and they may have a protective role against the development of congenital heart block (CHB).
_x000D_
-
Question 20 of 114
20. Question
A 27-year-old lady with a history of systemic lupus erythematosus (SLE) gives birth to her first child after two consecutive early pregnancy losses. She has been managed with low molecular weight heparin injections during her third pregnancy.
_x000D_
The delivery is uneventful, but the neonatologist notices that the newborn has an erythematous rash. The child is also bradycardic with a pulse rate of 72 beats per minute.
_x000D_
Which one of the following maternal antibodies is most likely to be linked to the illness in the baby?
CorrectIncorrectHint
The most likely maternal antibody linked to the newborn’s illness, given the history of systemic lupus erythematosus (SLE) in the mother and the symptoms of an erythematous rash and bradycardia, is B. Anti-Ro/SSA. Neonatal lupus is a condition that can occur when maternal anti-Ro/SSA antibodies are passed to the foetus, leading to symptoms such as skin rash and cardiac issues like bradycardia.
_x000D_
Now, let us rule out the other options:
_x000D_
- _x000D_
- A. c-anti-neutrophil cytoplasmic antibodies (cANCA): These antibodies are associated with vasculitis, not neonatal lupus, and would not explain the newborn’s symptoms.
- C. Anti-smooth muscle: These antibodies are typically associated with autoimmune hepatitis and are not linked to neonatal lupus or the symptoms described.
- D. Rheumatoid factor: This antibody is associated with rheumatoid arthritis and other autoimmune conditions but not specifically with neonatal lupus or the symptoms observed in the newborn.
- E. Anti-LKM antibodies: These are associated with autoimmune hepatitis type 2 and are not related to the symptoms of neonatal lupus described in the newborn.
_x000D_
_x000D_
_x000D_
_x000D_
_x000D_
In conclusion, the presence of anti-Ro/SSA antibodies is the most likely cause of the newborn’s condition, consistent with neonatal lupus. The other antibodies listed are not typically associated with the symptoms described in the newborn.
-
Question 21 of 114
21. Question
A 27-year-old lady presents to the clinic with an eight month history of progressively increasing fatigue and arthralgia of the both wrists and ankles. Few weeks ago she has also noted a symmetrical rash on her cheeks and a moderate amount of hair loss.
_x000D_
Which one is the most likely diagnosis?
CorrectIncorrectHint
The correct answer is C. Systemic lupus erythematosus (SLE). This is a chronic autoimmune disease that can affect any organ in the body, but mainly involves the skin, joints, kidneys, and blood. The symptoms and signs that have been described are consistent with this diagnosis, such as:
_x000D_
- _x000D_
- Fatigue, which is one of the most common and disabling symptoms of lupus, affecting up to 90% of people with the condition.
- Arthralgia, or joint pain, which affects about 95% of people with lupus at some point, especially in the wrists and ankles.
- Rash, which can take various forms, but the most characteristic one is a butterfly-shaped (malar) rash that appears over the bridge of the nose and on both cheeks, affecting about 50% of people with lupus.
- Hair loss, which can be caused by inflammation of the scalp, medications, or hormonal changes, affecting about 45% of people with lupus.
_x000D_
_x000D_
_x000D_
_x000D_
_x000D_
The other options are incorrect for the following reasons:
_x000D_
- _x000D_
- A. Porphyria cutanea tarda is a condition that affects the production of heme, a component of haemoglobin and other proteins. It causes skin blisters, itching, and increased hair growth on the face and hands, especially after exposure to sunlight. It does not cause fatigue, joint pain, or hair loss on the scalp.
- B. Scleroderma, or systemic sclerosis, is a condition that causes hardening and tightening of the skin and connective tissues. It can affect the fingers, hands, face, and other parts of the body, causing skin thickening, ulcers, Raynaud’s phenomenon, and calcinosis. It does not cause a butterfly-shaped rash or hair loss on the scalp.
- D. Hypothyroidism, or an underactive thyroid, is a condition that affects the metabolism and energy levels of the body. It can cause fatigue, weight gain, dry hair and skin, muscle aches, cold sensitivity, and feelings of depression. It does not cause joint pain, a butterfly-shaped rash, or kidney problems.
- E. Dermatomyositis is a condition that affects the muscles and the skin. It can cause muscle weakness, pain, and inflammation, as well as a distinctive skin rash on the eyelids, knuckles, elbows, knees, and chest. It does not cause hair loss on the scalp or kidney problems.
_x000D_
_x000D_
_x000D_
_x000D_
-
Question 22 of 114
22. Question
A 27-year-old lady presents to the Outpatient Department complaining of low back pain and stiffness which was gradually increasing in intensity for last five months.
_x000D_
She has no past medical history of note and the only medications she takes are anti-inflammatory drugs that relive the pain.
_x000D_
On examination, her back movements are stiff with decreased range of movement due to pain, but the spine curvature appears normal.
_x000D_
Which one of the following options is the most likely diagnosis?
CorrectIncorrectHint
All of these can be causes of back pain. The key differentiators in this case are the age of the patient and the detailed history of the type of back pain.
_x000D_
Patients often present with ankylosing spondylitis (AS) in their 20s and 30s with a history of chronic back pain and stiffness. Key features of the pain include:
_x000D_
- _x000D_
- Early morning stiffness of more than 30 minutes
- Alternating buttock pain
- Waking in the second half of the night
- Pain easing with non-steroidal anti-inflammatory drugs (NSAIDs)
- Pain which is worse with rest and eases with exercise
_x000D_
_x000D_
_x000D_
_x000D_
_x000D_
_x000D_
In the advanced stages of AS, patients develop loss of lumbar lordosis, buttock atrophy and an exaggerated thoracic kyphosis. This results in a posture commonly referred to as ‘question mark’.
_x000D_
Metastatic cancer can affect the spine and cause back pain. This is usually associated with ‘red flag’ signs suggesting malignancy. This diagnosis is less common in younger patients.
_x000D_
Muscular strain is the commonest cause of back pain in general practice but chronic pain for more than three months may indicate AS and should be investigated.
_x000D_
Reactive arthritis can cause inflammatory back pain with similar symptoms to AS but there is no history of preceding genitourinary or gastrointestinal infection.
_x000D_
Scheuermann’s disease generally presents in teenagers with thoracic back pain and increased thoracic kyphosis, and therefore does not fit with this scenario.
-
Question 23 of 114
23. Question
A 26-year-old young gentleman presents to his General Physician with triphasic Raynaud’s Phenomenon.
_x000D_
It affects his daily activities and is usually very painful. In particular, it is exacerbated when handling ice-cool raw food stock at his work in the cold storage.
_x000D_
He smokes 30 cigarettes per day. A physical examination is unremarkable. Initial investigations show him to be Antinuclear Antibody (ANA) negative.
_x000D_
What is the best initial line of management of his symptom?
CorrectIncorrectHint
This question recognises the importance of using conservative management before embarking on potentially long term medication with risks of side effects, and the recognition of the vasospastic effect of cigarette smoking.
_x000D_
The prostaglandin iloprost is useful in the treatment of Raynaud’s phenomenon and can be considered if the patient does not respond to nifedipine Retard or has developed digital ulceration or ischaemia.
_x000D_
Raynaud’s phenomenon responds well to calcium channel blockers such as nifedipine but given that this gentleman has so many factors that can be altered in his lifestyle, such as smoking and working in a cold environment, more simple measures to change these could avoid daily medication with its side effects.
_x000D_
Digital sympathectomy should be considered as a last resort when drug therapy has failed or has not been tolerated.
_x000D_
Capillaroscopy is useful especially when serum antibodies are positive but it would not change the management at this stage.
-
Question 24 of 114
24. Question
A 26-year-old lady has been receiving Sulfasalazine at a stable dose for eight months as treatment for Reactive Arthritis. Her most recent series of routine blood tests were normal.
_x000D_
When should she next be screened for her pathology?
CorrectIncorrectHint
Current United Kingdom guidance suggests that during the first three months of treatment with sulfasalazine, full blood count (FBC) should be monitored monthly for the first three months.
_x000D_
Sulphasalazine should be withheld until discussion with the specialist team if:
_x000D_
- _x000D_
- the white cell count is less than 3.5
- neutrophils is less than 2, or
- platelets are less than 150.
_x000D_
_x000D_
_x000D_
_x000D_
If mean corpuscular volume (MCV) is more than 105 fl, vitamin B12, folate and thyroid-stimulating hormone (TSH) should be checked and treated if found to be abnormal. If these are all normal it should be discussed with the specialist team.
_x000D_
If counts remain normal within the first three months, full blood count can be checked three monthly.
_x000D_
Liver function tests (LFTs) should also be checked monthly for the first three months. If either the aspartate aminotransferase (AST) or alanine aminotransferase (ALT) are more than twice the upper limit of normal sulfasalazine should be withheld until discussion with the specialist team. If the LFTs remain normal for the first three months, monitoring can be decreased to three monthly.
_x000D_
If, following the first year, the dose has not been increased and blood results have been stable, the frequency of monitoring can be reduced to every six months for the second year of treatment. Thereafter monitoring is not required, although FBC and LFTs should be checked one month after any dose increase.
_x000D_
Side effects of sulfasalazine include myelosuppression, macrocytosis, hypersensitivity and azoospermia in males.
_x000D_
There are numerous signs of sulfasalazine toxicity. Rash and oral ulceration should be asked about and, if severe, the drug should be withheld until specialist advice has been sought. Nausea, dizziness and headache can be common and sometimes necessitate dose reduction. If patients present with abnormal bruising or sore throat an urgent FBC should be done, and sulfasalazine withheld until results are available.
-
Question 25 of 114
25. Question
A 26-year-old gentleman presents to the Dermatology Clinic with red scaly plaques on his cheeks, forehead, and sides of the neck.
_x000D_
On close inspection of the patches, plugging of some hair follicles with keratin and diminution of the skin was noted.
_x000D_
Which one of the following is the most likely diagnosis?
CorrectIncorrectHint
The most likely diagnosis for this gentleman is E. Discoid lupus erythematosus. This is a chronic autoimmune skin condition that causes red, scaly, and sometimes atrophic patches on the face and other sun-exposed areas. The plugging of hair follicles with keratin and the diminution of the skin are characteristic features of discoid lupus erythematosus.
_x000D_
The other options can be ruled out as follows:
_x000D_
- _x000D_
- A. Polymorphic light eruption is a type of sun allergy that causes itchy, red, and sometimes blistering rashes on the skin after sun exposure. However, it does not cause plugging of hair follicles or skin atrophy.
- B. Porphyria cutanea tarda is a disorder of heme metabolism that causes photosensitivity, blisters, erosions, and hyperpigmentation on the skin. It does not cause scaly plaques or hair follicle plugging.
- C. Psoriasis is a common inflammatory skin condition that causes red, scaly, and sometimes pustular patches on the skin. It can affect the face, but it usually spares the central part of the face and involves the hairline and ears. It does not cause hair follicle plugging or skin atrophy.
- D. Atopic eczema is a chronic allergic skin condition that causes dry, itchy, and inflamed skin. It can affect the face, but it usually involves the eyelids, perioral area, and nasolabial folds. It does not cause scaly plaques, hair follicle plugging, or skin atrophy.
_x000D_
_x000D_
_x000D_
_x000D_
-
Question 26 of 114
26. Question
A 25-year-old lady presents with a three month history of low back pain and buttock pain that is worse in the morning. Her symptoms gradually improves with activity as the day progresses and with over the counter Ibuprofen. She has no significant past medical history or a family history.
_x000D_
On examination, there is restricted chest wall expansion (3.5 cm) and the Schober’s Test is positive. X Rays of lumbar spine and pelvis are normal. Blood tests show a normal Complete Blood Count (CBC), Serum Urea, Serum Electrolytes and Serum Creatinine and Liver Function Tests (LFTs). The Serum C-Reactive Protein (CRP) is 12 mg/L and the Erythrocyte Sedimentation Rate (ESR) is 30 mm/hr.
_x000D_
Which one of the following is the most appropriate investigation for this lady?
CorrectIncorrectHint
Based on the presence of back pain, raised inflammatory markers and reduced spinal mobility, this patient has ankylosing spondylitis (AS).
_x000D_
Ankylosing spondylitis (AS) is a chronic, potentially disabling, form of seronegative spondyloarthropathy which primarily involves the axial skeleton. The aetiology is not clearly understood, but it involves the interaction of genetic and environmental factors. The pathology mainly affects the entheses, where ligaments, tendons and capsules are attached to the bone.
_x000D_
Current British Society for Rheumatology recommendations state that the modified New York criteria should be used to diagnose ankylosing spondylitis:
_x000D_
Clinical criteria:
_x000D_
- _x000D_
- Low back pain, present for more than three months, improved by exercise but not relieved by rest
- Limitation of lumbar spine motion in both the sagittal and frontal planes
- Limitation of chest expansion relative to normal values for age and sex
_x000D_
_x000D_
_x000D_
_x000D_
Radiological criteria:
_x000D_
- _x000D_
- Sacroiliitis on x ray
_x000D_
_x000D_
Diagnosis:
_x000D_
- _x000D_
- Definite AS if the radiological criterion is present plus at least one clinical criterion
- Probable AS if three clinical criteria are present alone or if the radiological criterion is present but no clinicial criteria are present.
_x000D_
_x000D_
_x000D_
It is widely accepted that making the diagnosis early is difficult because the onset is insidious, and there is no definitive diagnostic test. One study has stated that the average time from onset to diagnosis ranges from three to 11 years. Timely diagnosis therefore requires a high index of suspicion.
_x000D_
HLA-B27 is found in 90% patients with AS but also 8% of the general population and therefore should not be relied upon in making a diagnosis.
_x000D_
MRI can visualise sacroiliitis in patients with typical symptoms of AS but normal radiographs, and it is evolving as the most important diagnostic imaging tool in early disease. However, as yet a diagnosis of ankylosing spondylitis cannot be made on MRI findings alone and these cases are currently termed non-radiographic axial spondyloarthritis (SpA).
_x000D_
Both HLA-B27 and sacroiliitis on MRI play a major role in the recently proposed Assessment of SpondyloArthritis International Society (ASAS) diagnostic algorithm. This may replace the modified New York criteria in the future.
_x000D_
An x ray of the lumbar spine may show bridging syndesmophytes between the vertebral bodies in keeping with ankylosing spondylitis but this is not part of the diagnostic criteria, and it is a late sign.
_x000D_
Radiographic sacroiliitis is a requirement for patients to be eligible for anti-TNF treatment for AS.
_x000D_
The sacroiliitis is usually bilateral and symmetrical. It progresses from blurring of the subchondral bone plate to irregular erosions of the margins of the sacroiliac joints to sclerosis, narrowing, and finally fusion.
_x000D_
Erosions of the sacroiliac joint are generally seen earlier in the lower portion due to its synovial lining, and on the iliac side due to the thinner cartilage covering this side of the joint.
_x000D_
As x ray is not available as an option here, MRI of the sacroiliac joints is the most appropriate answer.
_x000D_
PET-CT and bone scans do not have a role in the diagnosis and investigation of ankylosing spondylitis.
-
Question 27 of 114
27. Question
A 25-year-old lady presents with a one month history of swinging fever, weight loss, and erythematous nodular lesions on the shin. She has no peripheral adenopathy or any abnormal enlargement of an organ (organomegaly).
_x000D_
Chest x-ray showed bilateral hilar adenopathy. Computed tomography (CT) guided biopsy of the mediastinal lymph nodes was performed, which showed chronic inflammation with multiple non-caseating granulomas.
_x000D_
Which one of the following is the most likely diagnosis?
CorrectIncorrectHint
The most likely diagnosis for this lady with a history of swinging fever, weight loss, erythematous nodular lesions on the shin, bilateral hilar adenopathy, and non-caseating granulomas is E. Sarcoidosis. Sarcoidosis is characterised by the presence of non-caseating granulomas and can present with Löfgren’s syndrome, which includes erythema nodosum (erythematous nodular lesions), fever, bilateral hilar lymphadenopathy, and arthralgia. The absence of peripheral adenopathy and organomegaly also aligns with sarcoidosis, which typically presents with bilateral hilar lymphadenopathy without peripheral lymph node involvement.
_x000D_
The other options are less likely for the following reasons:
_x000D_
- _x000D_
- A. Lymphoma: While lymphoma can present with lymphadenopathy, it usually causes peripheral adenopathy and may show caseating granulomas if associated with tuberculosis. The absence of peripheral adenopathy and the presence of non-caseating granulomas make lymphoma less likely.
_x000D_
_x000D_
- _x000D_
- B. Histoplasmosis: This fungal infection can mimic sarcoidosis radiologically, but it is more commonly associated with caseating granulomas and often has a history of exposure to bird or bat droppings in endemic areas, which is not mentioned in this case.
_x000D_
_x000D_
- _x000D_
- C. Tuberculosis: Tuberculosis typically presents with caseating granulomas, and this lady would likely have a positive tuberculin skin test or interferon-gamma release assay, which is not mentioned here.
_x000D_
_x000D_
- _x000D_
- D. HIV infection: Human immunodeficiency virus (HIV) can cause generalized lymphadenopathy and can be associated with various opportunistic infections. However, the presentation of non-caseating granulomas and the specific symptoms described are more indicative of sarcoidosis.
_x000D_
_x000D_
In summary, based on the clinical presentation and biopsy findings, sarcoidosis is the most likely diagnosis for this lady.
-
Question 28 of 114
28. Question
A 25-year-old lady presents to the pain management clinic with a seven-month history of bilateral wrist pain, generalised aching, early morning stiffness and an intermittent subjective fever.
_x000D_
She has a medical history of grade 4 acne, which she states has become worse over her nasal bridge and cheeks despite being commenced on minocycline 14 months ago. She states that her mother has rheumatoid arthritis.
_x000D_
An autoimmune screen demonstrated positivity for ANA, P-ANCA and anti-DNA Histone; negative Anti-ds DNA Antibody; normal complement C3, C4 levels.
_x000D_
Which one of the following should be the first management step to be taken?
CorrectIncorrectHint
The first management step to be taken for this lady presenting with bilateral wrist pain, generalized aching, morning stiffness, fever, and a worsening of acne over her nasal bridge and cheeks, along with a positive autoimmune screen, should be A. Stop minocycline. Minocycline is known to cause drug-induced lupus erythematosus (DILE), which can present with symptoms similar to systemic lupus erythematosus (SLE), including musculoskeletal pain, fever, and a positive antinuclear antibody (ANA) test. The worsening of her acne and the presence of anti-DNA Histone antibodies, which are associated with drug-induced lupus, further support this decision.
_x000D_
Let us discuss why the other options are less appropriate:
_x000D_
- _x000D_
- Methylprednisolone 80 mg: While corticosteroids like methylprednisolone are used to reduce inflammation, they are not the first step in management without a definitive diagnosis and especially if the symptoms may be drug-induced.
- Methotrexate 15 mg: Methotrexate is a disease-modifying antirheumatic drug (DMARD) used in the treatment of rheumatoid arthritis and other autoimmune conditions. However, it is not the first-line treatment for suspected drug-induced lupus erythematosus (DILE) and should not be used without a definitive diagnosis.
- Naproxen 500 mg twice daily: Naproxen is a non-steroidal anti-inflammatory drug (NSAID) that can help with pain and inflammation. However, if a patient’s symptoms are due to drug-induced lupus erythematosus (DILE), the priority is to remove the offending agent rather than to treat the symptoms with non-steroidal anti-inflammatory drugs (NSAIDs).
- Hydroxychloroquine 400 mg: Hydroxychloroquine is used in the treatment of systemic lupus erythematosus (SLE) and rheumatoid arthritis. However, if a patient’s condition is related to minocycline use, the appropriate step is to discontinue the drug rather than start treatment with hydroxychloroquine.
_x000D_
_x000D_
_x000D_
_x000D_
_x000D_
In conclusion, the first step should be to stop minocycline to see if the symptoms improve, which would support a diagnosis of drug-induced lupus erythematosus (DILE).
-
Question 29 of 114
29. Question
A 25-year-old educator presents to the Pain Clinic with a six month history of pain and stiffness in her lower back. Pain is worse in the morning and improves with activity, and with Ibuprofen pro-re-nata.
_x000D_
She has no significant past medical history or family history. The Schober’s Test is positive. Magnetic Resonance Imaging (MRI) Scan of Sacroiliac Joint shows sacroiliitis and erosions. The Complete Blood Count (CBC), Serum Urea, Serum Electrolytes and Serum Creatinine and Liver Function Tests are all within normal ranges.
_x000D_
What is the next step in his management?
CorrectIncorrectHint
This lady has ankylosing spondylitis (AS), based on the history of back pain and stiffness which improves with activity.
_x000D_
The aim of treatment of AS is symptom control and maintenance of function. Physiotherapy is critical, and should be undertaken daily. There is limited evidence regarding any medication’s ability to alter the course of disease.
_x000D_
Non-steroidal anti-inflammatory drugs (NSAIDs) should be started in all cases, unless contraindicated. Where symptoms are not controlled additional analgesics (for example, amitriptyline), corticosteroid injections or oral corticosteroids can be used. She is already on them PRN and should be started on regular NSAIDs with a PPI.
_x000D_
Current NICE guidelines state that etanercept, golimumab and adalimumab can be used in patients with severe AS:
_x000D_
- _x000D_
- That satisfy the modified New York criteria
- Have confirmed, sustained active spinal disease over at least 12 weeks
- In whom maximal conventional treatment with two or more NSAIDs has failed
- Where there are no contraindications present.
_x000D_
_x000D_
_x000D_
_x000D_
_x000D_
Infliximab is not recommended. The evidence for the use of other disease modifying antirheumatic drugs is weak.
_x000D_
Surgery is occasionally useful to correct spinal or joint deformities.
_x000D_
In the last few years evidence for additional agents has been growing. Another anti-TNF alpha agent, certolizumab, is under investigation. Rituximab is also being studied as a treatment.
-
Question 30 of 114
30. Question
A 24-year-old young gentleman with a known Hereditary Angioneurotic Oedema (HAO) presents to the Rheumatology Clinic with a recurrent fever, arthralgia and a rash on the face and the upper chest for review.
_x000D_
Despite treatment for Hereditary Angioneurotic Oedema (HAO), he has always been troubled by recurrent attacks and has required adrenaline on several occasions.
_x000D_
His C4 levels have been persistently reduced secondary to his Hereditary Angioneurotic Oedema (HAO).
_x000D_
What is the most likely cause for his current symptoms?
CorrectIncorrectHint
HAO is characterised by deficiency of C1 esterase inhibitor.
_x000D_
This leads to persistent activation of the classical complement pathway and C4 levels are frequently low secondary to activation and consumption.
_x000D_
If treatment fails to normalise the C4 levels and they remain persistently low, these patients are at an increased risk of developing SLE
-
Question 31 of 114
31. Question
A 24-year-old lady presents to the outpatient department with a comprehensive illness for past three months.
_x000D_
She looks sick, with an intermittent fever. She has lost 2.5 kg of weight over the past two months, have generalised myalgia, polyarthralgia affecting both wrists, knees, ankles, elbows and metacarpophalangeal joints. She also complains of a sore throat.
_x000D_
Investigations Reveal:
_x000D_
Normochromic Normocytic Anaemia (Haemoglobin: 97 g/L); Erythrocyte Sedimentation Rate (ESR) – 80 mm in the first hour; Serum C-Reactive Protein (CRP) – 32 g/L; Serum Ferritin – 1754 mg/dL; Rheumatoid Factor – Negative; Antinuclear Antibody (ANA) – Negative; Extractable Nuclear Antigen (ENA) – Negative; Antistreptolysin (ASO) Titre < 200 IU.
_x000D_
Which one of the following is the most likely diagnosis?
CorrectIncorrectHint
The most likely diagnosis for this lady is C. Adult onset Still’s disease (AOSD). This is a rare inflammatory disorder that causes high fever, arthritis, rash, sore throat, and elevated inflammatory markers. The exact cause of adult onset Still’s disease (AOSD) is unknown, but it may be triggered by infections, genetic factors, or environmental factors. Adult onset Still’s disease (AOSD) can affect any organ system and may have serious complications such as macrophage activation syndrome, amyloidosis, or organ failure. Adult onset Still’s disease (AOSD) is diagnosed by excluding other causes of fever and inflammation, and by applying certain criteria that include clinical and immunologic features.
_x000D_
The other options are less likely for the following reasons:
_x000D_
- _x000D_
- A. Seronegative rheumatoid arthritis: This is a type of rheumatoid arthritis that does not have positive rheumatoid factor or anti-citrullinated peptide antibodies. It causes chronic and symmetrical joint inflammation, especially in the hands and feet. However, it does not usually cause high fever, rash, sore throat, or very high serum ferritin levels.
- B. Rheumatic fever: This is a complication of streptococcal pharyngitis that can cause fever, arthritis, carditis, chorea, erythema marginatum, and subcutaneous nodules. However, it is uncommon in adults and usually occurs within two to four weeks of the initial infection. It also requires evidence of recent streptococcal infection, such as positive throat culture, rapid antigen test, or elevated ASO titre.
- D. Polymyositis: This is an inflammatory myopathy that causes progressive and symmetrical muscle weakness, especially in the proximal muscles of the limbs and trunk. It may also cause muscle pain, dysphagia, interstitial lung disease, or skin rash. However, it does not usually cause fever, arthritis, sore throat, or leukocytosis. It also requires evidence of muscle damage, such as elevated creatine kinase, abnormal electromyography, or muscle biopsy.
- E. Systemic lupus erythematosus (SLE): This is a chronic autoimmune disease that can affect multiple organ systems and cause various manifestations, such as fever, arthritis, rash, oral ulcers, serositis, nephritis, or neuropsychiatric disorders. However, it usually requires positive ANA or other specific autoantibodies, such as anti-dsDNA, anti-Sm, or anti-phospholipid antibodies. It also has different criteria for diagnosis and classification, which include clinical and immunologic features.
_x000D_
_x000D_
_x000D_
_x000D_
_x000D_
Therefore, the correct answer is C. Adult onset Still’s disease (AOSD).
-
Question 32 of 114
32. Question
A 24-year-old lady delivers a male baby at term by lower segment caesarean section. The baby has complete heart block and subsequently requires pacemaker insertion.
_x000D_
Which of the following antibodies is most likely to be detected in the maternal serum?
CorrectIncorrectHint
The majority of cases of congenital heart block are due to the presence of anti-Ro/SSA antibodies in the maternal serum.
_x000D_
The mother may have no evidence of a connective tissue disorder.
_x000D_
The risks of congenital heart block in mothers with anti-Ro/SSA antibodies remains very small (<3%) but the correlation between the presence of anti-Ro/SSA antibodies and congenital heart block is very strong.
_x000D_
The heart block is generally permanent (unlike other features of neonatal lupus) and insertion of a permanent pacemaker is frequently required.
-
Question 33 of 114
33. Question
A 36-year-old gentleman attends the Rheumatology Clinic along with his wife for advice. He is currently taking Methotrexate 7.5 mg weekly. His wife is fit and well, and has no past medical history of note. She is not taking any medication apart from the oral contraceptive pill.
_x000D_
They are keen to start a family and want to know about continued contraception and whether there is a need to stop Methotrexate.
_x000D_
Which one of the following would be the most appropriate advice for this couple?
CorrectIncorrectHint
Methotrexate is teratogenic and, according to the British National Formulary (BNF), the manufacturers advise effective contraception during, and for at least three months, after stopping methotrexate. Fertility may be reduced during treatment, but this usually reverses upon stopping. Advice regarding reduced fertility and potential teratogenicity applies to both females and males who are taking methotrexate.
_x000D_
There is little published information on the potential teratogenicity following paternal exposure to methotrexate. There have, however, been reports of alterations of the spermatozoa and oligospermia following exposure to methotrexate. These seem to be reversible on cessation of treatment.
_x000D_
The National Patient Safety Agency (NPSA) state on their patient held record that: “It is recommended that men wait six months after finishing their treatment, before trying to father a child as sperm can be affected”. Paternal exposure to methotrexate is not regarded as an indication for termination of pregnancy, however
-
Question 34 of 114
34. Question
A 35-year-old lady presents to the orthopaedic clinic for follow up for a left-sided Colles’ fracture. The fracture occurred following a mechanical fall in the drawing room at home. The orthopaedic consultant is doubtful given the apparent low velocity mechanism of the injury.
_x000D_
Which one of the following measurements would indicate a diagnosis of osteoporosis?
CorrectIncorrectHint
The correct answer is C. T-score −2.5. This is a measurement of bone mineral density (BMD) that compares the person’s bone mineral density (BMD) with the average bone mineral density (BMD) of a young, healthy adult of the same sex. A T-score of −2.5 or lower indicates osteoporosis, which means the person has low bone mineral density (BMD) and a high risk of fractures.
_x000D_
The other options can be ruled out as follows:
_x000D_
- _x000D_
- A. T-score −1.5 is a measurement of bone mineral density (BMD) that indicates osteopenia, which means the person has below normal bone mineral density (BMD) but not low enough to be diagnosed with osteoporosis.
- B. Z-score −1.5 is a measurement of bone mineral density (BMD) that compares the person’s bone mineral density (BMD) with the average bone mineral density (BMD) of people of the same age, sex, and body size. A Z-score of −1.5 or lower is considered below the expected range for the person’s age group and may indicate secondary osteoporosis, which is caused by an underlying medical condition or medication.
- D. Z-score −2.0 is a measurement of bone mineral density (BMD) that is also below the expected range for the person’s age group and may indicate secondary osteoporosis. However, Z-scores are not used to diagnose osteoporosis in adults over 50 years old, as bone density naturally decreases with age. T-scores are more useful for diagnosing primary osteoporosis, which is caused by aging and hormonal changes.
- E. None of the above is not a correct answer, as T-score −2.5 is a valid measurement that indicates a diagnosis of osteoporosis.
_x000D_
_x000D_
_x000D_
_x000D_
-
Question 35 of 114
35. Question
A 35-year-old gentleman presents to the clinic with a three-week history of pain at the right elbow.
_x000D_
Which one of the following would be consistent with a diagnosis of tennis elbow?
CorrectIncorrectHint
The correct option for a diagnosis of tennis elbow is: D. Pain on wrist extension against resistance.
_x000D_
Tennis elbow, or lateral epicondylitis, is characterized by pain on the lateral side of the elbow, which can be exacerbated by extending the wrist against resistance. This is due to the overuse of the extensor muscles of the forearm, particularly where they attach to the lateral epicondyle of the elbow.
_x000D_
Now, let us address why the other options are incorrect:
_x000D_
- _x000D_
- Pain on extension of the elbow: This is not specific to tennis elbow, as pain on elbow extension can be associated with various elbow pathologies and does not specifically indicate the involvement of the extensor tendons of the forearm.
- Pain on pronation of the forearm: Pain during pronation of the forearm is not typical for tennis elbow, which primarily affects the extensor muscles that are involved in wrist extension, not pronation.
- Pain on flexion of the fingers against resistance: This would more likely suggest an issue with the flexor tendons, which is not characteristic of tennis elbow.
- Pain on pressure over the medial epicondyle: Pain over the medial epicondyle would suggest golfer’s elbow, or medial epicondylitis, which affects the flexor tendons of the forearm, not the extensor tendons affected in tennis elbow.
_x000D_
_x000D_
_x000D_
_x000D_
_x000D_
In summary, the most consistent sign of tennis elbow is pain on the lateral side of the elbow when the wrist is extended against resistance, due to the involvement of the extensor muscles of the forearm.
-
Question 36 of 114
36. Question
A 35-year-old lady with longstanding psoriasis has a four week history of persistently swollen, painful, and tender left knee. She has early morning stiffness which lasts for over an hour, and she has partial relief from the use of non-steroidal anti-inflammatory drugs (NSAIDs).
_x000D_
She is negative for rheumatoid factor. X-Ray of knee is normal.
_x000D_
Which one of the following is the next step in her long term management?
CorrectIncorrectHint
The next step in the long-term management of this lady with longstanding psoriasis and persistently swollen, painful, and tender left knee, who has partial relief from non-steroidal anti-inflammatory drugs (NSAIDs) and a normal x-ray, would be to consider disease modifying antirheumatic drugs (DMARDs) (option C). Disease-modifying antirheumatic drugs (DMARDs), such as methotrexate or leflunomide, are often used when symptoms do not sufficiently improve with non-steroidal anti-inflammatory drugs (NSAIDs) alone. They can slow the progression of psoriatic arthritis and prevent joint damage.
_x000D_
The other options are incorrect for the reasons as discussed below:
_x000D_
- _x000D_
- Anti-TNF alpha agents (A): These are typically used for moderate to severe psoriatic arthritis and may be considered if there is an inadequate response to disease-modifying antirheumatic drugs (DMARDs). They are not usually the first line of treatment due to their cost and potential side effects.
- Intra-articular corticosteroids (B): These can provide relief for acute flare-ups but are not generally used for long-term management due to potential joint damage with repeated use.
- Oral corticosteroids (D): These are not commonly used for long-term management of psoriatic arthritis due to the risk of significant side effects, especially with long-term use.
- Alternative non-steroidal anti-inflammatory drugs (NSAIDs) (E): While other non-steroidal anti-inflammatory drugs (NSAIDs) could be tried, if one non-steroidal anti-inflammatory drug (NSAID) does not provide adequate relief, it is unlikely that another will have a significantly different effect. Additionally, non-steroidal anti-inflammatory drugs (NSAIDs) do not prevent disease progression.
_x000D_
_x000D_
_x000D_
_x000D_
_x000D_
Therefore, disease-modifying antirheumatic drugs (DMARDs) are the most appropriate next step in her management, aiming to control the disease activity and prevent joint damage.
-
Question 37 of 114
37. Question
A 34-year-old gentleman presents to his General Physician with a four month history of low back pain, and stiffness. This is worse in the morning and gradually improves with activity.
_x000D_
He has no other symptoms and has no significant past medical or family history. He has tried over the counter Ibuprofen (400 mg up to three times a day), which led to a significant improvement in his symptoms.
_x000D_
X-Ray of lumbar spine and pelvis done by his General Physician is reported as normal. Blood Tests show a normal Complete Blood Count (CBC), Serum Urea, Serum Electrolytes and Liver Function Tests (LFTs). Serum C-Reactive Protein (CRP) is 22 mg/L and the Erythrocyte Sedimentation Rate (ESR) is 30 mm/hr.
_x000D_
Which one of the following is the most likely diagnosis?
CorrectIncorrectHint
The combination of lower back pain and stiffness makes ankylosing spondylitis (AS) the most likely diagnosis in this scenario.
_x000D_
Ankylosing spondylitis is a chronic, potentially disabling, form of seronegative spondyloarthropathy which primarily involves the axial skeleton. The aetiology is not clearly understood, but it involves the interaction of genetic and environmental factors. The pathology mainly affects the entheses, where ligaments, tendons and capsules are attached to the bone.
_x000D_
Current British Society for Rheumatology recommendations state that the modified New York criteria should be used to diagnose ankylosing spondylitis:
_x000D_
Clinical criteria:
_x000D_
- _x000D_
- Low back pain, present for more than three months, improved by exercise but not relieved by rest
- Limitation of lumbar spine motion in both the sagittal and frontal planes
- Limitation of chest expansion relative to normal values for age and sex
_x000D_
_x000D_
_x000D_
_x000D_
Radiological criteria:
_x000D_
- _x000D_
- Sacroiliitis on x ray
_x000D_
_x000D_
Diagnose:
_x000D_
- _x000D_
- Definite AS if the radiological criterion is present plus at least one clinical criterion
- Probable AS if three clinical criteria are present alone or if the radiological criterion is present but no clinicial criteria are present.
_x000D_
_x000D_
_x000D_
It is widely accepted that making the diagnosis early is difficult because the onset is insidious, and there is no definitive diagnostic test. One study has stated that the average time from onset to diagnosis ranges from three to 11 years. Timely diagnosis therefore requires a high index of suspicion.
_x000D_
HLA-B27 is found in 90% patients with AS but also 8% of the general population and therefore should not be relied upon in making a diagnosis.
_x000D_
MRI can visualise sacroiliitis in patients with typical symptoms of AS but normal radiographs and it is evolving as the most important diagnostic imaging tool in early disease.
_x000D_
However, as yet a diagnosis of ankylosing spondylitis cannot be made on MRI findings alone and these cases are currently termed non-radiographic axial spondyloarthritis (SpA).
_x000D_
Both HLA-B27 and sacroiliitis on MRI play a major role in the recently proposed Assessment of SpondyloArthritis International Society (ASAS) diagnostic algorithm. This may replace the modified New York criteria in the future.
_x000D_
An x ray of the lumbar spine may show bridging syndesmophytes between the vertebral bodies in keeping with ankylosing spondylitis but this is not part of the diagnostic criteria, and it is a late sign.
_x000D_
Radiographic sacroiliitis is a requirement for patients to be eligible for anti-TNF treatment for AS. The sacroiliitis is usually bilateral and symmetrical. It progresses from blurring of the subchondral bone plate to irregular erosions of the margins of the sacroiliac joints to sclerosis, narrowing, and finally fusion.
_x000D_
Erosions of the sacroiliac joint are generally seen earlier in the lower portion due to its synovial lining, and on the iliac side due to the thinner cartilage covering this side of the joint.
_x000D_
Discitis is inflammation of the vertebral disc space, often related to infection. It typically presents with an insidious onset of pain and localised tenderness, which is worsens with activity (unlike the stiffness described here).
_x000D_
Mechanical back pain and osteoarthritis pain also improve with NSAIDs. However, they cause low back pain that is worse with activity, and relieved with rest.
_x000D_
Spondylolisthesis is the movement of one vertebra due to instability. It presents with pain which is worse with activity, and may be associated with nerve root compression.
-
Question 38 of 114
38. Question
A 33-year-old lady presents to the orthopaedic clinic with a painful swollen left knee two months after an episode of acute gastroenteritis. She has no personal or family history of chronic skin diseases and she drinks limited amount of alcohol occasionally but does not smoke.
_x000D_
On examination there is a left knee effusion, and the knee aspirate shows plenty of leucocytes, no crystals, and no organisms on Gram stain or culture.
_x000D_
Which one of the following is the next step in the management of her symptom?
CorrectIncorrectHint
The next step in the management of the lady’s symptoms should be E. Oral non-steroidal anti-inflammatory drugs (NSAIDs). Non-steroidal anti-inflammatory drugs (NSAIDs) are indicated for relief of pain and swelling in cases of inflammatory arthritis when infection has been ruled out, as in this case where the knee aspirate showed no organisms.
_x000D_
Let us consider why the other options are less appropriate:
_x000D_
A. Intra-articular corticosteroids: These are typically used for inflammatory arthritis but should be avoided until infection has been definitively excluded, as corticosteroids can worsen an existing infection.
_x000D_
B. Oral antibiotics for six weeks: Long-term antibiotics are prescribed for confirmed infectious arthritis, which is not indicated here due to the absence of organisms on Gram stain or culture.
_x000D_
C. Commence disease modifying antirheumatic drug (DMARD): Disease-modifying antirheumatic drugs (DMARDs) are used for chronic inflammatory conditions like rheumatoid arthritis, which is not indicated here without a definitive diagnosis and persistent inflammatory signs.
_x000D_
D. Oral antibiotics for five days: Short-term antibiotics might be considered if there was suspicion of a bacterial infection, but again, the knee aspirate did not show any organisms, making this option inappropriate.
_x000D_
In conclusion, given the absence of infection and the presence of inflammation indicated by the effusion and leucocytes, E. Oral non-steroidal anti-inflammatory drugs (NSAIDs) is the most appropriate next step for symptom management.
-
Question 39 of 114
39. Question
A 33-year-old lady presents to the Rheumatology Clinic with Raynaud’s Phenomenon for a review.
_x000D_
Which one of the following clinical features suggests an underlying connective tissue disease?
CorrectIncorrectHint
Raynaud’s phenomenon is a common clinical presentation, which may be primary or secondary to underlying disease.
_x000D_
It can be diagnosed if there is a history of clearly demarcated pallor of the digit(s) followed by at least one other colour change (cyanosis and/or erythema). Symptoms are usually precipitated by cold (or less commonly emotion). Vasospasm without endothelial damage is thought to be the main cause for primary RP. The pathogenesis of secondary forms is probably initiated primarily by endothelial damage.
_x000D_
Physical examination, nailfold capillaroscopy and immunological tests can differentiate between primary and secondary Raynaud’s.
_x000D_
One should suspect secondary Raynaud’s phenomenon if any of the following are present:
_x000D_
- _x000D_
- Onset at more than 30 years of age
- Intense, painful or asymmetrical episodes
- Presence of additional clinical features suggestive of underlying disease
- Positive anti-nuclear antibody
- Abnormal nail-fold capillaries
- Digital ulcers, gangrene or severe ischaemia of one or more digits.
_x000D_
_x000D_
_x000D_
_x000D_
_x000D_
_x000D_
_x000D_
Primary Raynaud’s can be diagnosed if all the following are present:
_x000D_
- _x000D_
- No suspicion of underlying disease
- Symmetrical episodes affecting both hands, but not necessarily all fingers
- No tissue necrosis, ulceration, gangrene or severe ischaemia
- Normal nail-fold capillaries
- Normal ESR and negative anti-nuclear antibodies.
_x000D_
_x000D_
_x000D_
_x000D_
_x000D_
_x000D_
Treatment involves prevention so that permanent ischaemic damage can be avoided. Patients should avoid exposure to the cold.
_x000D_
The mild forms of primary RP can be controlled by non-pharmacological approaches alone. If insufficient, the first choice therapy is calcium channel blockers. In severe forms, intravenous prostaglandin, endothelin-1 receptor antagonists and phosphodiesterase-5 inhibitors are used.
_x000D_
Future treatment options may include selective alpha-2c adrenergic receptor blockers, tyrosine and Rho-kinase inhibitors and calcitonin gene-related peptide.
_x000D_
Differential diagnosis of Raynaud’s phenomenon includes:
_x000D_
- _x000D_
- Chilblains (perniosis): erythematous itchy swellings on fingers and toes in response to cold
- Acrocyanosis: continuous blueness of the extremities aggravated by cold
- Erythromelalgia: painful erythema caused by paroxysmal dilatation of blood vessels
- Vascular embolism
- Livedo reticularis
- Mottled, cyanotic discolouration of skin.
_x000D_
_x000D_
_x000D_
_x000D_
_x000D_
_x000D_
_x000D_
Whilst it is not a strict criteria given in the current guidance, it is recognised that episodes do tend to be longer in secondary Raynaud’s. Episodes typically terminate within 15 minutes following warming in primary disease, but can often be prolonged in secondary disease. All the other features described here would be consistent with a diagnosis of primary Raynaud’s disease.
_x000D_
Whilst miscarriage can be associated with connective tissue disease, in particular antiphospholipid syndrome, it is common in the population especially in early pregnancy.
-
Question 40 of 114
40. Question
A 33-year-old gentleman presents to the clinic with a progressively worsening right knee pain and a three month history of loss of weight.
_x000D_
He has a good appetite but has had occasional episodes of diarrhoea over this period of time and tends to pass a loose motion at least thrice daily. He does not take any regular medication. He has a family history of hypothyroidism. He does not smoke and drinks frugal quantities of alcohol occasionally.
_x000D_
Examination reveals a swollen, tender right knee joint with a small effusion.
_x000D_
Which one of the following is the most likely diagnosis?
CorrectIncorrectHint
The correct answer is B. Inflammatory bowel disease (IBD).
_x000D_
This gentleman’s clinical presentation of weight loss, chronic diarrhoea, and mono/oligo-arthropathy is indicative of inflammatory bowel disease (IBD)-associated arthropathy, which is a recognized extraintestinal manifestation of inflammatory bowel disease (IBD). This form of arthritis is typically non-erosive and may correlate with the activity of the intestinal disease. The lack of correlation between axial arthritis and gastrointestinal symptoms further supports the diagnosis of inflammatory bowel disease (IBD) over other forms of spondyloarthropathy.
_x000D_
The other options are incorrect for the following reasons:
_x000D_
- _x000D_
- Tuberculosis: Tuberculous arthritis typically presents with pain, swelling, and reduced range of motion in the affected joint, and is now rare in Europe. This gentleman’s symptoms do not align with the common presentation of tuberculous arthritis, which often includes fever and weight loss, and there is no mention of a history of tuberculosis or risk factors associated with it.
- Thyrotoxicosis: Thyrotoxicosis generally presents with symptoms such as rapid heart rate, weight loss, anxiety, and sweating due to an overactive metabolism. While diarrhoea and arthralgia can occur, the presentation of monoarthritis is uncommon, making thyrotoxicosis an unlikely diagnosis in this case.
- Reactive arthritis: Reactive arthritis is commonly associated with the triad of urethritis, conjunctivitis, and arthritis, often following infections with organisms like Salmonella or Chlamydia. The chronic diarrhoea described does not fit the acute presentation typically associated with reactive arthritis, and there is no mention of urethritis or conjunctivitis.
- Behçet’s disease: Behçet’s disease is characterized by recurrent oral and genital ulcers and ocular involvement. While arthropathy can occur, this gentleman’s symptoms do not include the hallmark features of Behçet’s disease, and bowel habit disturbances are not expected, making this an unlikely diagnosis.
_x000D_
_x000D_
_x000D_
_x000D_
_x000D_
In summary, this gentleman’s symptoms and the correlation of arthritis with intestinal disease activity strongly suggest inflammatory bowel disease (IBD) as the most likely diagnosis. The other options are less consistent with this gentleman’s presentation and lack supporting clinical evidence or characteristic features.
-
Question 41 of 114
41. Question
A 33-year-old lady presents to the Cardiology Clinic with a deep vein thrombosis.
_x000D_
Her past medical history is significant with episodes of two miscarriages, both in her early pregnancies.
_x000D_
Investigations Revealed:
_x000D_
_x000D_ _x000D_
_x000D_ _x000D_ Haemoglobin
_x000D_
_x000D_
_x000D_ 130 g/L
_x000D_
_x000D_
_x000D_ (115-165)
_x000D_
_x000D_
_x000D_
_x000D_ _x000D_ White Blood Cell Count
_x000D_
_x000D_
_x000D_ 3.5 ×109/L
_x000D_
_x000D_
_x000D_ (4-11)
_x000D_
_x000D_
_x000D_
_x000D_ _x000D_ Platelet Count
_x000D_
_x000D_
_x000D_ 33 ×109/L
_x000D_
_x000D_
_x000D_ (150-400)
_x000D_
_x000D_
_x000D_
_x000D_
_x000D_
Which of the following investigation is most likely to be abnormal?
CorrectIncorrectHint
The combination of thrombophilia, recurrent miscarriage, thrombocytopenia and leucopenia in this patient indicates a diagnosis of antiphospholipid syndrome, probably in associated with systemic lupus erythematosus.
_x000D_
Antiphospholipid syndrome is a common cause of acquired thrombophilia and characterised by arterial and/or venous thrombosis and pregnancy mortality in association with circulating antiphospholipid antibodies. These are a heterogenous group of approximately twenty autoantibodies directed against phospholipid binding plasma proteins. Three of the most clinically important are the lupus anticoagulant, anti-beta-2 glycoproetin I antibodies and the anticardiolipin antibodies. They can be detected either by phospholipid-dependent coagulation test for lupus anticoagulant or ELISA test for anticoagulation and anti-β2GPI antibodies. Antibodies should be demonstrated on at least two occasions separated by 12 weeks. Antiphospholipid syndrome may be primary, or associated with other conditions (such as systemic lupus erythematosus).
_x000D_
Antiphospholipid syndrome is the most important treatable cause of recurrent miscarriage, defined as the loss of three or more consecutive pregnancies. 15% of women with recurrent miscarriage have persistently positive tests for either lupus anticoagulant or anticardiolipin antibodies, compared to 2% with an uncomplicated obstetric history. In future untreated pregnancies, women with recurrent miscarriage and persistently positive anticardiolipin antibodies have a miscarriage rate of 90%. The majority of miscarriages occur between 7 and 12 weeks gestation, and foetuses are typically chromosomally normal. It is thought the antibodies affect trophoblast invasion and placentation.
_x000D_
Antiphospholipid syndrome is also an important cause of early onset pre-eclampsia and intra-uterine growth restriction.
_x000D_
Aspirin and low-dose heparin is the treatment of choice to reduce the risk of miscarriage in confirmed antiphospholipid syndrome. This combination has been showed to lead to a 70% live birth rate in future pregnancies. Intravenous immunoglobulin can also be used.
_x000D_
Elevated levels of circulating homocysteine increase the risk of developing coronary artery disease, peripheral vascular disease and cerebrovascular disease but they are not commonly associated with pregnancy loss.
_x000D_
An indium white blood cell scan is a nuclear medicine study in which leucocytes are removed from the patient, tagged with Indium-111 and reinjected into the patient. They can then be used to localise areas of infection and inflammation, such as thrombophlebitis and osteomyelitis.
_x000D_
Platelet function studies measure the platelet’s ability to adhere and aggregate. They are not particularly reliable or accurate, and therefore do not have a central role in clinical practice.
_x000D_
Protein C is one of the major inhibitors of the coagulation system. Deficiency is associated with an increased risk of venous thrombosis, but not classically an increase rate of miscarriage.
-
Question 42 of 114
42. Question
A 33-year-old gentleman presents to the clinic with a four month history of arthralgia, mouth ulceration and eye irritation.
_x000D_
On examination he was afebrile, had some ulceration of the mouth, bilaterally swollen wrists and effusions, with painfully reduced range of movements of both knees.
_x000D_
Examination of the external genitalia revealed a scrotal ulcer.
_x000D_
Investigations Revealed:
_x000D_
_x000D_ _x000D_
_x000D_ _x000D_ Total Leucocyte Count
_x000D_
_x000D_
_x000D_ 12.4 ×109/L
_x000D_
_x000D_
_x000D_ (4-11)
_x000D_
_x000D_
_x000D_
_x000D_ _x000D_ Serum C-Reactive Protein (CRP)
_x000D_
_x000D_
_x000D_ 118 mg/L
_x000D_
_x000D_
_x000D_ (<10)
_x000D_
_x000D_
_x000D_
_x000D_ _x000D_ Rheumatoid Factor
_x000D_
_x000D_
_x000D_ Negative
_x000D_
_x000D_
_x000D_
_x000D_
_x000D_
Which one of the following is the most likely diagnosis?
CorrectIncorrectHint
The most likely diagnosis for this gentleman is E. Behçet’s syndrome. Behçet’s syndrome is a rare disorder that causes blood vessel inflammation throughout the body, leading to various signs and symptoms that can seem unrelated at first. Some of the characteristic features of Behçet’s syndrome are:
_x000D_
- _x000D_
- Recurrent oral and genital ulcers
- Eye inflammation (uveitis or conjunctivitis)
- Skin lesions (erythema nodosum, pustular vasculitis, or pyoderma gangrenosum)
- Arthritis or arthralgia, usually affecting the knees, ankles, wrists, or elbows
- Positive pathergy test (skin reaction to minor trauma)
_x000D_
_x000D_
_x000D_
_x000D_
_x000D_
_x000D_
The gentleman in this case has oral and genital ulcers, eye irritation, and arthritis, which are consistent with Behçet’s syndrome. Additionally, he has a high white blood cell count and C-reactive protein (CRP), indicating inflammation, and a negative rheumatoid factor, ruling out rheumatoid arthritis. The diagnosis of Behçet’s syndrome is based on clinical criteria, as there is no specific laboratory test for it.
_x000D_
The other options are less likely because:
_x000D_
- _x000D_
- A. Inflammatory bowel disease (IBD) is a group of disorders that cause chronic inflammation of the digestive tract, such as ulcerative colitis or Crohn’s disease. Inflammatory bowel disease (IBD) can cause extraintestinal manifestations, such as arthritis, mouth ulcers, eye inflammation, and skin lesions, but these are usually associated with active bowel inflammation. The gentleman in this case does not have any gastrointestinal symptoms, such as diarrhoea, abdominal pain, or blood in stool, which makes inflammatory bowel disease (IBD) unlikely.
- B. Psoriatic arthritis is a type of arthritis that affects some people who have psoriasis, a chronic skin condition that causes red, scaly patches. Psoriatic arthritis can cause joint pain and swelling, nail changes, eye inflammation, and skin lesions, but it usually affects the distal joints of the fingers and toes, the spine, or the sacroiliac joints. The gentleman in this case does not have psoriasis or nail changes, and has bilateral wrist and knee involvement, which are not typical of psoriatic arthritis.
- C. Reactive arthritis is a form of arthritis that develops in response to an infection in another part of the body, usually the intestines or the genitals. Reactive arthritis can cause joint pain and swelling, eye inflammation, urethritis, and skin lesions, but it usually occurs within a few weeks of the triggering infection. The gentleman in this case has a four-month history of symptoms, which is too long for reactive arthritis. Moreover, he does not have any history of infection or urethritis, which are common triggers of reactive arthritis.
- D. Sjögren’s syndrome is an autoimmune disorder that affects the moisture-producing glands of the eyes and mouth, causing dryness. Sjögren’s syndrome can also affect other organs, such as the joints, skin, lungs, kidneys, and nerves, causing various symptoms. The gentleman in this case has mouth ulcers, not dryness, and does not have any other symptoms of Sjögren’s syndrome, such as fatigue, dry eyes, dry cough, or neuropathy.
_x000D_
_x000D_
_x000D_
_x000D_
-
Question 43 of 114
43. Question
A 32-year-old lady presents to the pain management clinic with a two-year history of periodically painful fingers. The pain occurs commonly in low ambient temperatures and there is coextensive skin colour change from white to blue to red and each episode lasts approximately for 10 – 15 minutes.
_x000D_
Her past medical history is significant of the fact that her mother suffered from rheumatoid arthritis and Raynaud’s disease.
_x000D_
Which one of the following bedside tests could be performed that may indicate an underlying connective tissue disorder as the probable diagnosis?
CorrectIncorrectHint
The most appropriate bedside test to indicate an underlying connective tissue disorder in this case is C. Nailfold capillaroscopy. This non-invasive test is used to examine the small blood vessels (capillaries) at the base of the fingernail and is particularly useful in diagnosing conditions like systemic sclerosis and distinguishing primary from secondary Raynaud’s phenomenon. Abnormalities such as capillary dilation, haemorrhages, or avascular areas can suggest the presence of a connective tissue disease.
_x000D_
Now, let us rule out the other options:
_x000D_
A. Capnography measures the concentration of carbon dioxide in exhaled air and is primarily used to monitor ventilation in various settings, such as during anaesthesia or in critical care. It is not used to diagnose connective tissue disorders and would not be relevant to the symptoms described.
_x000D_
B. Finger systolic pressure can help in the diagnosis of vascular diseases affecting the fingers, such as Raynaud’s phenomenon, but it does not specifically indicate an underlying connective tissue disorder.
_x000D_
D. Digital artery closing temperature is not a standard bedside test for connective tissue disorders. While temperature changes can affect blood flow and may be relevant in Raynaud’s phenomenon, this test does not provide specific information about connective tissue disease involvement.
_x000D_
E. Cold water challenge is a test that can trigger a Raynaud’s phenomenon episode and is used to diagnose Raynaud’s syndrome. However, it does not provide direct evidence of an underlying connective tissue disorder and can be non-specific.
_x000D_
Given the lady’s history and symptoms, nailfold capillaroscopy is the most suitable test to perform for a probable diagnosis of a connective tissue disorder.
-
Question 44 of 114
44. Question
A 32-year-old lady who leads almost a sedentary lifestyle has Systemic Lupus Erythematosus (SLE), and she had done some blood tests recently, whose results are as follows:
_x000D_
_x000D_ _x000D_
_x000D_ _x000D_ Haemoglobin
_x000D_
_x000D_
_x000D_ 125 g/L
_x000D_
_x000D_
_x000D_ (115-165)
_x000D_
_x000D_
_x000D_
_x000D_ _x000D_ Total Leucocyte Count
_x000D_
_x000D_
_x000D_ 8.7 ×109/L
_x000D_
_x000D_
_x000D_ (4-11)
_x000D_
_x000D_
_x000D_
_x000D_ _x000D_ Neutrophils
_x000D_
_x000D_
_x000D_ 82%
_x000D_
_x000D_
_x000D_ (40-75)
_x000D_
_x000D_
_x000D_
_x000D_ _x000D_ Platelet Count
_x000D_
_x000D_
_x000D_ 88 ×109/L
_x000D_
_x000D_
_x000D_ (150-400)
_x000D_
_x000D_
_x000D_
_x000D_ _x000D_ Erythrocyte Sedimentation Rate (ESR)
_x000D_
_x000D_
_x000D_ 13 mm/hr
_x000D_
_x000D_
_x000D_ (0 – 20)
_x000D_
_x000D_
_x000D_
_x000D_ _x000D_ International Normalised Ratio (INR)
_x000D_
_x000D_
_x000D_ 1.0
_x000D_
_x000D_
_x000D_ (<1.4)
_x000D_
_x000D_
_x000D_
_x000D_ _x000D_ Activated Partial Thromboplastin Time (APTT)
_x000D_
_x000D_
_x000D_ 49 Seconds
_x000D_
_x000D_
_x000D_ (30 – 40s)
_x000D_
_x000D_
_x000D_
_x000D_
_x000D_
Activated Partial Thromboplastin Time (APTT) did not normalise after addition of normal plasma.
_x000D_
Blood Film shows thrombocytopenia, no schistocytes. Serum Urea, Serum electrolytes, and Serum creatinine are all within normal limits.
_x000D_
What is the most likely explanation for the abnormal Activated Partial Thromboplastin Time (APTT)?
CorrectIncorrectHint
This lady has antiphospholipid antibody syndrome which is a non-inflammatory pro-thrombotic state in vivo, in the presence of laboratory tests suggesting an anticoagulant state.
_x000D_
The antiphospholipid antibodies (anticardiolipin and lupus anticoagulant) cause coagulation defect in-vitro (raised aPTT which fails to correct after the addition of normal human plasma). However, despite the coagulation defects and thrombocytopenia, the lupus anticoagulant causes arterial and venous thromboses in vivo.
_x000D_
Disseminated intravascular clotting and thrombotic thrombocytopenic purpura are severe systemic illnesses, where bleeding may occur due to consumption of clotting factors, and platelets by an overactive clotting cascade.
_x000D_
Idiopathic thrombocytopenic purpura does not cause raised aPTT.
-
Question 45 of 114
45. Question
A 32-year-old lady presented to the clinic with acute stiffness and swelling of her both knees and ankles, and a painful rash on her legs.
_x000D_
The erythrocyte sedimentation rate (ESR) was 88 mm in the first hour (0-15). Chest X-ray showed hilar lymphadenopathy.
_x000D_
Which one of the following is the most likely conclusion?
CorrectIncorrectHint
The most likely conclusion is D. Spontaneous improvement. This gentleman has a classic presentation of Löfgren syndrome, which is an acute form of sarcoidosis that manifests with erythema nodosum, polyarthropathy, hilar lymphadenopathy, and sometimes fever, uveitis, and parotitis. Löfgren syndrome has a good prognosis and usually resolves spontaneously over six to eight weeks without treatment.
_x000D_
The other options are less likely because:
_x000D_
- _x000D_
- A. Chronic arthritis is not a typical outcome of Löfgren syndrome, as most patients recover completely without joint damage. Chronic arthritis may occur in other forms of sarcoidosis that involve noncaseating granulomas in the joints, but this is not the case for this gentleman.
- B. Skin ulceration is not a common feature of Löfgren syndrome, which is characterized by erythema nodosum, a type of panniculitis that causes tender nodules on the lower limbs. Skin ulceration may be seen in other cutaneous manifestations of sarcoidosis, such as lupus pernio or scar sarcoidosis, but this is not the case for this gentleman.
- C. Pulmonary fibrosis is a possible complication of chronic sarcoidosis, but it is rare in Löfgren syndrome, which usually has a benign course and does not cause significant lung damage. Pulmonary fibrosis may result from persistent inflammation and granuloma formation in the lung parenchyma, leading to scarring and reduced lung function.
- E. Renal failure is not a frequent complication of Löfgren syndrome, which does not affect the kidneys. Renal failure may occur in sarcoidosis due to granulomatous interstitial nephritis, nephrocalcinosis, or renal stones, but these are uncommon and usually associated with hypercalcaemia or hypercalciuria.
_x000D_
_x000D_
_x000D_
_x000D_
-
Question 46 of 114
46. Question
A 32-year-old lady has rheumatoid arthritis and is on a stable dose of weekly methotrexate. She is keen to start a family as soon as possible and visits her rheumatologist for advice about her current regimen of methotrexate. She is advised to stop taking methotrexate as soon as she can.
_x000D_
If methotrexate treatment is stopped and there are no other contraindications for natural conception, how long should she hold back before stopping the oral contraceptive pill (OCP) and trying to conceive?
CorrectIncorrectHint
The correct answer is C. She should continue the OCP for at least three months after stopping methotrexate. This is because methotrexate is a teratogenic drug, which means it can cause harm to the developing foetus if taken during pregnancy. According to the British National Formulary (BNF) and the National Patient Safety Agency (NPSA), both men and women should avoid conception for at least three months after stopping methotrexate, as it may take some time for the drug to be cleared from the body and the tissues. Fertility may be temporarily reduced during treatment, but it usually returns to normal after stopping the drug.
_x000D_
The other options are not correct for the following reasons:
_x000D_
- _x000D_
- A. She should continue the OCP for at least two weeks after stopping methotrexate: This is incorrect because two weeks is not a sufficient period of time to ensure that methotrexate is completely eliminated from the body and that the risk of foetal anomalies is minimized. Methotrexate has a variable elimination half-life, which means it can stay in the body for different lengths of time depending on the individual and the dose. It can also accumulate in some organs, such as the liver and the kidneys, and cause damage. Therefore, a longer period of contraception is recommended by the British National Formulary (BNF) and the National Patient Safety Agency (NPSA).
- B. She can stop the OCP at the same time as she stops methotrexate: This is incorrect because stopping the OCP at the same time as methotrexate can increase the chance of ovulation and conception before methotrexate is fully cleared from the body. This can expose the developing embryo to the harmful effects of methotrexate, which can cause miscarriage or congenital malformations. The British National Formulary (BNF) and the National Patient Safety Agency (NPSA) advise that women should wait for at least one ovulatory cycle, and men should wait for at least three months, before trying to conceive after stopping methotrexate.
- D. She should continue the OCP for at least one year after stopping methotrexate: This is incorrect because one year is much longer than necessary to prevent pregnancy after stopping methotrexate. There is no evidence that methotrexate has a long-term effect on fertility or pregnancy outcomes after it is discontinued. A shorter period of contraception, such as three months, is sufficient to reduce the risk of methotrexate exposure during pregnancy, as recommended by the British National Formulary (BNF) and the National Patient Safety Agency (NPSA).
- E. She should continue the OCP for at least one month after stopping methotrexate: This is incorrect because one month is shorter than necessary to prevent pregnancy after stopping methotrexate. Although the British National Formulary (BNF) and the National Patient Safety Agency (NPSA) state that women should wait for at least one ovulatory cycle before trying to conceive, this may not be enough time to ensure that methotrexate is completely eliminated from the body and that the risk of foetal anomalies is minimized. Methotrexate has a variable elimination half-life, which means it can stay in the body for different lengths of time depending on the individual and the dose. It can also accumulate in some organs, such as the liver and the kidneys, and cause damage. Therefore, a longer period of contraception, such as three months, is recommended by the British National Formulary (BNF) and the National Patient Safety Agency (NPSA).
_x000D_
_x000D_
_x000D_
_x000D_
-
Question 47 of 114
47. Question
A 32-year-old gentleman presents to the rheumatology clinic with chronic pain. The pain changes from day to day, but often focuses in the lower back.
_x000D_
He is pale and looks unwell. He complains of waking up frequently at night, and feels not rested in the morning. He also complains of intermittent constipation and diarrhoea.
_x000D_
Examination is essentially normal – but the he complains of tenderness in multiple areas on palpation. Routine blood tests are normal.
_x000D_
Which one of the following is the most likely diagnosis?
CorrectIncorrectHint
The most likely diagnosis for the gentleman described is D. Fibromyalgia. This condition is characterized by widespread musculoskeletal pain accompanied by fatigue, sleep, memory, and mood issues. The symptoms mentioned, such as chronic pain that varies in location, tenderness in multiple areas, and normal blood tests, align with fibromyalgia. Additionally, the lack of restorative sleep and gastrointestinal symptoms like constipation and diarrhoea are common in fibromyalgia.
_x000D_
The other options are incorrect for the following reasons:
_x000D_
- _x000D_
- Somatoform disorder: This disorder involves patients having physical symptoms that are not explained by a medical condition. However, the chronic nature of the pain and its specific characteristics in this case are more indicative of fibromyalgia.
- Schizophrenia: This is a severe mental disorder characterized by distortions in thinking, perception, emotions, language, sense of self, and behavior. Chronic pain is not a typical feature of schizophrenia.
- Hypothyroidism: While hypothyroidism can cause muscle pain and weakness, it is usually accompanied by other symptoms like weight gain, cold intolerance, and changes in skin and hair. Moreover, routine blood tests would typically show abnormalities in thyroid function tests, which is not the case here.
- Depressive disorder: Depression can cause physical symptoms, including pain, but it is primarily characterized by persistent feelings of sadness and loss of interest. The physical examination findings and the variety of symptoms presented are more consistent with fibromyalgia.
_x000D_
_x000D_
_x000D_
_x000D_
_x000D_
In conclusion, based on the symptoms and examination findings provided, fibromyalgia is the most likely diagnosis.
-
Question 48 of 114
48. Question
A 31-year-old gentleman presents to the clinic with rapidly worsening Raynaud’s phenomenon and skin tightness.
_x000D_
On examination, there is sclerodactyly, skin thickening to the shoulders, and bi-basal crepitations. Muscle strength is normal.
_x000D_
Blood Tests Show:
_x000D_
_x000D_ _x000D_
_x000D_ _x000D_ Haemoglobin
_x000D_
_x000D_
_x000D_ 123 g/L
_x000D_
_x000D_
_x000D_ (115-165)
_x000D_
_x000D_
_x000D_
_x000D_ _x000D_ Total Leucocyte Count
_x000D_
_x000D_
_x000D_ 7.5 ×109/L
_x000D_
_x000D_
_x000D_ (4-11)
_x000D_
_x000D_
_x000D_
_x000D_ _x000D_ Neutrophils
_x000D_
_x000D_
_x000D_ 72%
_x000D_
_x000D_
_x000D_ (40-75)
_x000D_
_x000D_
_x000D_
_x000D_ _x000D_ Platelet Count
_x000D_
_x000D_
_x000D_ 274 ×109/L
_x000D_
_x000D_
_x000D_ (150-400)
_x000D_
_x000D_
_x000D_
_x000D_ _x000D_ Erythrocyte Sedimentation Rate (ESR)
_x000D_
_x000D_
_x000D_ 34 mm/hr
_x000D_
_x000D_
_x000D_ (0-20)
_x000D_
_x000D_
_x000D_
_x000D_ _x000D_ Anti-Nuclear Antibody (ANA)
_x000D_
_x000D_
_x000D_ Positive (1:6400)
_x000D_
_x000D_
_x000D_ (Negative at 1:20 Dil)
_x000D_
_x000D_
_x000D_
_x000D_ _x000D_ Anti-Centromere Antibody
_x000D_
_x000D_
_x000D_ Negative
_x000D_
_x000D_
_x000D_ (Negative at 1:40 Dil)
_x000D_
_x000D_
_x000D_
_x000D_ _x000D_ Anti-Scl70 Antibody
_x000D_
_x000D_
_x000D_ Positive (1:640)
_x000D_
_x000D_
_x000D_ (Negative)
_x000D_
_x000D_
_x000D_
_x000D_ _x000D_ Anti-U1RNP Antibody
_x000D_
_x000D_
_x000D_ Negative
_x000D_
_x000D_
_x000D_ (Negative)
_x000D_
_x000D_
_x000D_
_x000D_ _x000D_ Anti-Ro/La Antibody
_x000D_
_x000D_
_x000D_ Negative
_x000D_
_x000D_
_x000D_ (Negative)
_x000D_
_x000D_
_x000D_
_x000D_ _x000D_ Serum Urea, Serum Electrolytes And Serum Creatinine
_x000D_
_x000D_
_x000D_ Normal
_x000D_
_x000D_
_x000D_
_x000D_
_x000D_
Which one of the following is the most anticipated diagnosis?
CorrectIncorrectHint
The most anticipated diagnosis for the gentleman described is C. Diffuse cutaneous systemic sclerosis. This condition is characterized by rapidly progressing skin thickening involving areas proximal to the elbows and knees, and can include the trunk, which aligns with this gentleman’s symptoms of skin tightness and thickening to the shoulders. The presence of sclerodactyly, Raynaud’s phenomenon, and bi-basal crepitations are also indicative of systemic sclerosis. The positive Anti-Scl70 antibody is highly specific for systemic sclerosis, particularly the diffuse cutaneous subtype.
_x000D_
Let us evaluate the other options:
_x000D_
A. Mixed connective tissue disease (MCTD): Mixed connective tissue disease (MCTD) typically presents with features of several connective tissue diseases, including systemic lupus erythematosus, scleroderma, and polymyositis. Patients often have high titers of Anti-U1RNP antibodies, which this patient does not have, making mixed connective tissue disease (MCTD) less likely.
_x000D_
B. Dermatomyositis: This inflammatory disease is characterized by muscle weakness and a distinctive skin rash. The absence of muscle weakness and the presence of Anti-Scl70 antibodies make dermatomyositis an unlikely diagnosis for this gentleman.
_x000D_
D. Sjögren’s syndrome: Sjögren’s syndrome primarily presents with dry eyes and dry mouth due to lymphocytic infiltration of exocrine glands. This gentleman’s symptoms do not align with this diagnosis, and the serological markers (Anti-Ro/La antibodies) associated with Sjögren’s syndrome are negative.
_x000D_
E. Limited cutaneous systemic sclerosis: This subtype of systemic sclerosis is associated with skin thickening that is limited to the hands, face, feet, and forearms. This gentleman’s extensive skin involvement beyond these areas suggests the diffuse cutaneous subtype rather than the limited form.
_x000D_
In summary, the clinical presentation, serological findings, and the absence of features typical of the other conditions listed support the diagnosis of C. Diffuse cutaneous systemic sclerosis.
-
Question 49 of 114
49. Question
A 31-year-old lady presents to the Clinic with recurrent troublesome acne, episodes of palmoplantar pustules, and painful swelling of the acromioclavicular joint for a couple of weeks.
_x000D_
Investigations Show:
_x000D_
_x000D_ _x000D_
_x000D_ _x000D_ Haemoglobin
_x000D_
_x000D_
_x000D_ 124 g/L
_x000D_
_x000D_
_x000D_ (115-165)
_x000D_
_x000D_
_x000D_
_x000D_ _x000D_ Total Leucocyte Count
_x000D_
_x000D_
_x000D_ 7.6 ×109/L
_x000D_
_x000D_
_x000D_ (4-11)
_x000D_
_x000D_
_x000D_
_x000D_ _x000D_ Neutrophils
_x000D_
_x000D_
_x000D_ 72%
_x000D_
_x000D_
_x000D_ (40-75)
_x000D_
_x000D_
_x000D_
_x000D_ _x000D_ Platelet Count
_x000D_
_x000D_
_x000D_ 272 ×109/L
_x000D_
_x000D_
_x000D_ (150-400)
_x000D_
_x000D_
_x000D_
_x000D_ _x000D_ Erythrocyte Sedimentation Rate (ESR)
_x000D_
_x000D_
_x000D_ 34 mm/hr
_x000D_
_x000D_
_x000D_ (0-20)
_x000D_
_x000D_
_x000D_
_x000D_ _x000D_ Serum Bilirubin
_x000D_
_x000D_
_x000D_ 15 µmol/L
_x000D_
_x000D_
_x000D_ (1-22)
_x000D_
_x000D_
_x000D_
_x000D_ _x000D_ Serum Alanine Aminotransferase (ALT)
_x000D_
_x000D_
_x000D_ 33 IU/L
_x000D_
_x000D_
_x000D_ (5-35)
_x000D_
_x000D_
_x000D_
_x000D_ _x000D_ Serum Aspartate Aminotransferase (AST)
_x000D_
_x000D_
_x000D_ 34 IU/L
_x000D_
_x000D_
_x000D_ (1-31)
_x000D_
_x000D_
_x000D_
_x000D_ _x000D_ Alkaline Phosphatase (ALP)
_x000D_
_x000D_
_x000D_ 218 U/L
_x000D_
_x000D_
_x000D_ (45-105)
_x000D_
_x000D_
_x000D_
_x000D_
_x000D_
Serum Urea, Serum Electrolytes and Serum Creatinine are within normal ranges.
_x000D_
What is the diagnosis?
CorrectIncorrectHint
This lady has SAPHO syndrome. SAPHO is an acronym for synovitis, acne, pustulosis, hyperostosis, and osteitis. It is characterised by osteosclerotic bone lesions, sterile osteomyelitis, and a variety of skin lesions.
_x000D_
- _x000D_
- Synovitis – may be present rarely, and associates with erosions.
- Acne – may be severe (conglobate or fulminans) and recur with new bony involvement.
- Pustulosis – palmo-plantar pustulosis occurs in approximately 50% of patients, other skin lesions may include psoriasis, hidradenitis suppurativa, acne, and rarely Sweet’s syndrome.
- Hyperostosis (increase in bone substance) and osteitis (inflammation of the bones) – the bony lesions typically involve the acromioclavicular, and sternoclavicular joints. Other sites include anterior chest wall, sternum, clavicle, pubic symphysis, spine, and mandible. These lesions are visualised on 99m technetium bone scan or MRI.
_x000D_
_x000D_
_x000D_
_x000D_
_x000D_
The cause of the SAPHO syndrome is unknown. The skin lesions are characterised by neutrophilic pseudoabscesses. Bone biopsy can reveal sterile osteomyelitis.
_x000D_
Diagnosis should be suspected when there is an association of rheumatic pain with a pustular skin disease.
_x000D_
SAPHO has no specific treatment, and some cases remit spontaneously. Typical treatment can be used for the arthritic symptoms (i.e. non-steroidal anti-inflammatories and disease modifying anti-rheumatic agents). Isotretinoin and aciretin can be used to treat the skin disease. In the more severe cases corticosteroids, calcitonin, bisphosphonates and TNF-inhibitors can be used.
-
Question 50 of 114
50. Question
A 31-year-old lady at 14 week period of gestation with a history of Systemic Lupus Erythematosus (SLE) is referred by the Obstetrician to the Rheumatology Clinic for review.
_x000D_
Which one of the following is correct regarding Systemic Lupus Erythematosus (SLE) in pregnancy?
CorrectIncorrectHint
Prednisolone and hydroxychloroquine may be taken whilst breast-feeding. Azathioprine, cyclophosphamide, methotrexate and cyclosporin A are contraindicated in breast-feeding mothers.
_x000D_
In general, SLE does not affect the fertility of patients. However, fertility may be adversely affected in specific subgroups of patients such as those with renal failure, cyclophosphamide treatment, very active disease or high dose corticosteroids.
_x000D_
Anti-Ro and anti-La antibodies are associated with increased risk of congenital heart block. The risk is greater with anti-Ro positivity than anti-La positivity. The risk of congenital heart block in the presence of anti-Ro may be up to 5%.
_x000D_
Stopping any unnecessary drugs is advisable in pregnancy, however use of azathioprine, hydroxychloroquine and prednisolone in pregnancy is considered safe if these are necessary for treatment of the mother’s disease.
_x000D_
Risk of pre-eclampsia is increased in SLE. It may be difficult to differentiate between pre-eclampsia and renal flare of SLE, and both may coexist. Differentiating features include raised anti-dsDNA antibody, decreased complement levels (C3 and C4) and response to steroids in the case of renal flare.
-
Question 51 of 114
51. Question
A 30-year-old lady, known to have Systemic Lupus Erythematosus (SLE), delivers her baby at 38 weeks. During the initial neonatal check up, the baby appears to have a scaly red rash and to be bradycardic. An Electrocardiogram (ECG) shows complete heart block.
_x000D_
Which maternal antibodies are most likely to be responsible for these findings?
CorrectIncorrectHint
Anti-Ro antibodies from a mother with SLE can cause fetal heart block as part of neonatal lupus. It affects only 3-5% of babies born to mothers with SLE, and the most common cause of heart block is anti-Ro antibodies.
_x000D_
Anti-La antibodies are also a cause of fetal heart block, but less commonly than anti-Ro.
_x000D_
dsDNA is used as a marker of disease activity in Lupus, but is not associated with neonatal heart block.
_x000D_
pANCA is common in a number of vasculitides but is not specific to neonatal lupus.
_x000D_
Anti-Jo1 antiboides are associated with dermatomyositis and polymyositis.
-
Question 52 of 114
52. Question
A 30-year-old lady presents to the Outpatient Department with fever, urethritis and arthralgia. She also has slight vaginal discharge but has no menstrual disturbances. She is found to have a swollen right ankle with a pustular rash on the dorsal aspect of her foot.
_x000D_
Which one of the following is the most likely diagnosis?
CorrectIncorrectHint
The correct diagnosis for this lady presenting with fever, urethritis, arthralgia, vaginal discharge, a swollen ankle, and a pustular rash is A. Disseminated gonorrhoea. Disseminated gonococcal infection (DGI) is a serious complication of gonorrhoea caused by the bacterium Neisseria gonorrhoeae. It can lead to various systemic symptoms, including dermatitis, tenosynovitis, migratory polyarthralgias, and, less commonly, septic arthritis.
_x000D_
Let us address why the other options are less likely:
_x000D_
- _x000D_
- B. Staphylococcal arthritis: This condition typically presents with acute onset of symptoms and is associated with a more localized infection. It does not usually include urethritis or a pustular rash.
- C. Tuberculous arthritis: This form of arthritis is caused by Mycobacterium tuberculosis and usually affects a single joint, leading to chronic pain and swelling. It is not characterized by urethritis or a pustular rash.
- D. Lyme disease: Lyme disease, caused by the tick-borne bacterium Borrelia burgdorferi, typically presents with a distinctive erythema migrans rash and can affect various systems. However, this lady’s presentation does not align with the typical manifestations of Lyme disease.
- E. Reactive arthritis: This condition is an autoimmune response to an infection, often presenting with conjunctivitis, urethritis, and arthritis. The skin rash described in this lady’s case is not typical for reactive arthritis.
_x000D_
_x000D_
_x000D_
_x000D_
_x000D_
In conclusion, this lady’s symptoms align most closely with disseminated gonorrhoea, which is known to cause a pustular rash, fever, urethritis, and oligoarthritis, fitting the clinical picture described. The other conditions listed do not typically present with this combination of symptoms and are therefore less likely to be the correct diagnosis.
-
Question 53 of 114
53. Question
A 30-year-old gentleman presents to the Rheumatology Clinic with a four month history of arthralgia for a review. He has no past medical history of note.
_x000D_
Examination reveals swelling of the distal interphalangeal joints of the middle and ring fingers of the hand and wrist on the left and a swelling of the right ankle.
_x000D_
Investigations Show:
_x000D_
_x000D_ _x000D_
_x000D_ _x000D_ Erythrocyte Sedimentation Rate (ESR)
_x000D_
_x000D_
_x000D_ 42 mm/hr
_x000D_
_x000D_
_x000D_ (0-10)
_x000D_
_x000D_
_x000D_
_x000D_
_x000D_
Which one of the following is the most likely diagnosis?
CorrectIncorrectHint
This gentleman has psoriatic arthritis. Synovitis is indicative of an inflammatory arthritis. The rash typically pre-dates the arthropathy by a number of years, but the opposite can be true. Small plaques should be looked for on the elbows and scalp.
_x000D_
There are five patterns of disease:
_x000D_
- _x000D_
- Symmetrical polyarthritis (‘rheumatoid pattern’) – affects wrists, hands, feet and ankles. The distal interphalangeal joints are more commonly affected than the metacarpophalangeal joints, which helps to distinguish it from rheumatoid arthritis.
- Asymmetrical oligoarticular arthritis – dactylitis.
- Distal interphalangeal joint disease – typically in men.
- Arthritis mutilans – rare.
- Spondylitis pattern with sacroileitis.
_x000D_
_x000D_
_x000D_
_x000D_
_x000D_
_x000D_
Rheumatoid arthritis typically affects the metacarpophalangeal and proximal interphalangeal joints symmetrically. Psoriatic arthritis affects the distal interphalangeal joints and tends to be asymmetrical.
_x000D_
Joint involvement in systemic lupus erythematosus occurs in the form of a polyarticular arthralgia, frequently symmetrical and episodic. Intense tendonitis is more common than synovitis and can lead to deforming reversible subluxation of joints without erosive disease (Jaccoud’s arthropathy).
_x000D_
Osteoarthritis would be unusual in someone of this age group.
_x000D_
Reactive arthritis is a sterile inflammatory arthritis which develops as a sequel to remote infection, usually of the gastrointestinal or urogenital tract. There is no history of such infection in this case. It also classically presents as a large joint oligoarthritis.
-
Question 54 of 114
54. Question
A 28-year-old lady, at 33 weeks period of gestation with Sjögren’s syndrome (dry eyes, dry mouth, anti-Ro/La positive), is referred to the rheumatologist by her obstetrician for evaluation and management.
_x000D_
Which one of the following is a risk to the foetus?
CorrectIncorrectHint
The correct answer is E. All of the above. All of these options are potential risks to the foetus of a woman with Sjögren’s syndrome, especially if she is positive for anti-Ro/La antibodies. These antibodies can cross the placenta and affect the development and function of the foetal heart, skin, liver, and blood cells. The possible complications include:
_x000D_
- _x000D_
- A. Complete heart block: This is a condition where the electrical impulses that control the heartbeat are blocked, resulting in a slow and irregular heart rate. This can lead to heart failure, hydrops foetalis, or death in utero or shortly after birth. The incidence of complete heart block in foetuses of women with anti-Ro/La antibodies is estimated to be 1-2%.
- B. Hydrops foetalis: This is a condition where the foetus develops severe fluid accumulation in the body cavities, such as the chest, abdomen, or skin. This can cause respiratory distress, anaemia, organ failure, or death. The causes of hydrops foetalis in foetuses of women with anti-Ro/La antibodies include complete heart block, neonatal lupus, or haemolytic anaemia.
- C. Neonatal lupus: This is a condition where the foetus develops skin rashes, liver problems, or blood cell abnormalities due to the exposure to anti-Ro/La antibodies. The symptoms usually appear within the first few weeks of life and resolve within 6-12 months as the antibodies clear from the baby’s system. The incidence of neonatal lupus in foetuses of women with anti-Ro/La antibodies is estimated to be 10-20%.
- D. Congestive cardiac failure (CCF): This is a condition where the heart is unable to pump enough blood to meet the body’s needs, resulting in fluid retention, shortness of breath, fatigue, or death. The causes of congestive cardiac failure (CCF) in foetuses of women with anti-Ro/La antibodies include complete heart block, myocarditis, or cardiomyopathy.
_x000D_
_x000D_
_x000D_
_x000D_
-
Question 55 of 114
55. Question
A 34-year-old gentleman with a longstanding history of Raynaud’s Phenomenon and no other symptoms had some blood tests done as advised by his General Physician.
_x000D_
Blood Tests show that he has a homogeneous pattern anti-nuclear antibody in a titre of 1:5123, and that she is negative for Anti-dsDNA Antibody using the Crithidia Luciliae assay. His Complete Blood Count (CBC), Serum Urea, Serum Electrolytes and Serum Creatinine; Liver Function Tests (LFTs), and C3 and C4 are all within normal limits.
_x000D_
Which one of the following is the most likely diagnosis?
CorrectIncorrectHint
Diffuse staining ANA on immunofluorescence in high titre raises the possibility of systemic lupus erythematosus (SLE).
_x000D_
Further tests are required to ascertain if the diffuse staining ANA is due to anti-dsDNA (pathogenic in SLE) or due to anti ss-DNA antibody (not thought to play a pathogenic role in SLE).
_x000D_
Crithidialuciliae immunofluorescence is positive only in the presence of anti-dsDNA antibody. This is as Crithidialuciliae has a giant mitochondrion, with a circular dsDNA. It does not have ssDNA or any histones. In routine clinical practice, only anti-dsDNA are tested for. Diffuse staining ANA on immunofluorescence, in low titres (<1:160) is very common in the general population.
_x000D_
Although not mutually exclusive, different ANA immunofluorescence patterns suggest different diseases:
_x000D_
- _x000D_
- Anti-centromere pattern: limited cutaneous systemic sclerosis
- Cytoplasmic pattern: Sjögren’s syndrome, anti-synthetase syndrome
- Nucleolar pattern: diffuse cutaneous systemic sclerosis
- Speckled pattern: mixed connective tissue disease.
_x000D_
_x000D_
_x000D_
_x000D_
_x000D_
This gentleman has no symptoms to support the diagnoses of
_x000D_
- _x000D_
- Discoid lupus erythematosus (scarring photosensitive skin ulceration)
- Sjögren’s syndrome (dry eyes, dry mouth, fatigue)
- Scleroderma (Raynaud’s phenomenon, skin thickening, tightness, telangiectasia), and
- SLE (photosensitive rash).
_x000D_
_x000D_
_x000D_
_x000D_
_x000D_
Anti-dsDNA antibodies are highly specific for diagnosis of SLE. However, they are neither necessary, nor sufficient in themselves for the diagnosis of SLE. At any one time only 50% patients with SLE have the anti-dsDNA antibody, while only 70% patients with SLE develop anti-dsDNA antibodies at any one time during the course of their illness.
_x000D_
Raynaud’s phenomenon is a common clinical presentation, which may be primary or secondary to underlying disease. It can be diagnosed if there is a history of clearly demarcated pallor of the digit(s) followed by at least one other colour change (cyanosis and/or erythema). Symptoms are usually precipitated by cold (or less commonly emotion). Vasospasm without endothelial damage is thought to be the main cause for primary RP. The pathogenesis of secondary forms is probably initiated primarily by endothelial damage.
_x000D_
Physical examination, nailfold capillaroscopy and immunological tests can differentiate between primary and secondary Raynaud’s.
_x000D_
One should suspect secondary Raynaud’s phenomenon if any of the following are present:
_x000D_
- _x000D_
- Onset at more than 30 years of age
- Intense, painful or asymmetrical episodes
- Presence of additional clinical features suggestive of underlying disease
- Positive anti-nuclear antibody
- Abnormal nailfold capillaries
- Digital ulcers, gangrene or severe ischaemia of one or more digits.
_x000D_
_x000D_
_x000D_
_x000D_
_x000D_
_x000D_
_x000D_
Primary Raynaud’s can be diagnosed if all the following are present:
_x000D_
- _x000D_
- No suspicion of underlying disease
- Symmetrical episodes affecting both hands, but not necessarily all fingers
- No tissue necrosis, ulceration, gangrene or severe ischaemia
- Normal nailfold capillaries
- Normal ESR and negative anti-nuclear antibodies.
_x000D_
_x000D_
_x000D_
_x000D_
_x000D_
_x000D_
Treatment involves prevention so that permanent ischaemic damage can be avoided. Patients should avoid exposure to the cold.
_x000D_
The mild forms of primary RP can be controlled by non-pharmacological approaches alone. If insufficient, the first choice therapy is calcium channel blockers. In severe forms, intravenous prostaglandin, endothelin-1 receptor antagonists and phosphodiesterase-5 inhibitors are used.
_x000D_
Future treatment options may include selective alpha-2c adrenergic receptor blockers, tyrosine and Rho-kinase inhibitors and calcitonin gene-related peptide.
_x000D_
Differential diagnosis of Raynaud’s phenomenon includes:
_x000D_
- _x000D_
- Chilblains (perniosis): erythematous itchy swellings on fingers and toes in response to cold
- Acrocyanosis: continuous blueness of the extremities aggravated by cold
- Erythromelalgia: painful erythema caused by paroxysmal dilatation of blood vessels
- Vascular embolism
- Livedo reticularis: mottled, cyanotic discolouration of skin.
_x000D_
_x000D_
_x000D_
_x000D_
_x000D_
_x000D_
All the other features described here would be consistent with a diagnosis of primary Raynaud’s disease.
_x000D_
Whilst miscarriage can be associated with connective tissue disease, in particular antiphospholipid syndrome, it is common in the population especially in early pregnancy.
-
Question 56 of 114
56. Question
A 40-year-old woodworker, who is recently been working excessive overtime on a building project, comes to the Rheumatology Clinic complaining of pain radiating from the lateral aspect of his both elbows and travelling down to his forearms.
_x000D_
On examination he has pain on palpation over the lateral aspects of the both humerus, and on resisted dorsiflexion of the wrists.
_x000D_
Which one of the following is the most likely diagnosis?
CorrectIncorrectHint
This gentleman has symptoms that are consistent with tennis elbow (lateral epicondylitis), most probably due to excessive forearm extension as a result of his work as a woodworker.
_x000D_
Lateral epicondylitis occurs due to inflammation of the extensor forearm muscle origins, and causes lateral elbow and upper forearm pain and tenderness. On examination, patients have pain and tenderness over the lateral epicondyle of the humerus, radiating into the forearm and pain on resisted dorsiflexion of the wrist and middle finger. Pain is exacerbated by active and resisted movements of the extensor muscles of the forearm.
_x000D_
Lateral epicondylitis is a self-limiting condition, and usually resolves within 6 months to 2 years. Whilst awaiting for resolution rest, ice, activity restriction and non-steroidal anti-inflammatory drugs can reduce symptoms. Physiotherapy has been shown to be beneficial. Steroids injections have been shown to be harmful in the long-term and are therefore not recommended. Glyceryl trinitrate patches over the painful area do improve outcome.
-
Question 57 of 114
57. Question
A 40-year-old gentleman presents to the chest clinic with an eight-week history of dry cough, feeling feverish with chills and progressively worsening dyspnoea.
_x000D_
He reports that recently he has been able to walk for only five to ten minutes after which he has to stop and rest for breath. He has lost about 1.5 kg of weight over this period of time.
_x000D_
He has no recent history of travelling nor does he have any history of meeting friends or relatives from any endemic region.
_x000D_
He was diagnosed with rheumatoid arthritis seven year previously, for which he was started on etanercept five months ago.
_x000D_
Clinical examination demonstrated: Temperature – 100.4°F; Pulse Rate – 98/Minutes and regular; Heart Sounds – Normal; Respiratory Rate – 20/Minute; Chest – Mild mid zone inspiratory crepitations; SpO2 – 98% (Pre-exertion) and 88% (Post-exertion).
_x000D_
Initial Blood Tests Show:
_x000D_
_x000D_ _x000D_
_x000D_ _x000D_ Haemoglobin
_x000D_
_x000D_
_x000D_ 109 g/L
_x000D_
_x000D_
_x000D_ (115-165)
_x000D_
_x000D_
_x000D_
_x000D_ _x000D_ Mean Corpuscular Volume (MCV)
_x000D_
_x000D_
_x000D_ 90 fL
_x000D_
_x000D_
_x000D_ (80-96)
_x000D_
_x000D_
_x000D_
_x000D_ _x000D_ Total Leucocyte Count
_x000D_
_x000D_
_x000D_ 5.1 ×109/L
_x000D_
_x000D_
_x000D_ (4-11)
_x000D_
_x000D_
_x000D_
_x000D_ _x000D_ Westergren Erythrocyte Sedimentation Rate (ESR)
_x000D_
_x000D_
_x000D_ 45/hr
_x000D_
_x000D_
_x000D_ (0-20)
_x000D_
_x000D_
_x000D_
_x000D_ _x000D_ Serum C-Reactive Protein (CRP)
_x000D_
_x000D_
_x000D_ 36 mg/L
_x000D_
_x000D_
_x000D_ (<10)
_x000D_
_x000D_
_x000D_
_x000D_
_x000D_
Chest X-ray shows diffuse bilateral infiltrates.
_x000D_
Which one of the following tests is most likely to be distinctive?
CorrectIncorrectHint
The most likely diagnosis for this gentleman is pulmonary infection due to opportunistic pathogens. This is because he has a history of rheumatoid arthritis and is on etanercept, which is a tumour necrosis factor (TNF) blocker that increases the risk of serious infections. The gentleman also has symptoms and signs of systemic inflammation such as fever, chills, weight loss, elevated erythrocyte sedimentation rate (ESR) and C-reactive protein (CRP). The chest X-ray shows diffuse bilateral infiltrates, which can be caused by various infectious and non-infectious causes.
_x000D_
The most distinctive test to confirm the diagnosis and identify the causative organism is bronchoalveolar lavage (BAL). This is a procedure that involves inserting a bronchoscope into the lungs and collecting fluid samples from the alveoli. The fluid can then be analysed for microbiological cultures, stains, antigen tests, and cytology. Bronchoalveolar lavage (BAL) can help differentiate between bacterial, fungal, viral, and other opportunistic infections, as well as non-infectious causes of diffuse infiltrates such as pulmonary haemorrhage, oedema, or alveolar proteinosis.
_x000D_
The other tests are less distinctive for the following reasons:
_x000D_
- _x000D_
- High resolution computed tomography (CT) scan: This is a more detailed imaging modality than chest X-ray, but it cannot provide a definitive diagnosis of the type of infection. It can only show the pattern and distribution of the pulmonary abnormalities, such as nodules, ground-glass opacities, consolidations, or cavities. These patterns can overlap among different infectious and non-infectious causes, and may not be specific for opportunistic pathogens.
- Mycoplasma serology: This is a blood test that detects antibodies against Mycoplasma pneumoniae, a common cause of atypical pneumonia. However, this test is not very useful in the setting of immunosuppression, as the antibody response may be delayed, absent, or unreliable. Moreover, Mycoplasma pneumoniae is not a typical opportunistic pathogen, and it usually causes patchy rather than diffuse infiltrates on chest X-ray.
- Aspergillus precipitin: This is a blood test that detects antibodies against Aspergillus species, a group of fungi that can cause invasive pulmonary aspergillosis in immunocompromised patients. However, this test has low sensitivity and specificity, and may not reflect the current infection status. Furthermore, Aspergillus species are not the only fungi that can cause opportunistic pulmonary infections, and they usually cause cavitary rather than diffuse infiltrates on chest X-ray.
- Quantiferon: This is a blood test that measures the interferon-gamma release by T cells in response to Mycobacterium tuberculosis antigens. It is used to diagnose latent or active tuberculosis (TB) infection. However, this test may not be reliable in immunosuppressed patients, as the T cell response may be impaired or absent. Additionally, tuberculosis (TB) is not the only bacterial infection that can cause opportunistic pulmonary infections, and it usually causes apical rather than diffuse infiltrates on chest X-ray.
_x000D_
_x000D_
_x000D_
_x000D_
_x000D_
Therefore, the correct answer is D. Bronchoalveolar lavage.
-
Question 58 of 114
58. Question
A 40-year-old gentleman is admitted to the Emergency Department with a spiking temperature and sweats.
_x000D_
He has been unwell for the last one month with fleeting polyarthralgia and lethargy. He has a rash over his trunk which is most prevalent in the mornings.
_x000D_
Blood Cultures are sterile. His recent Transthoracic Echocardiogram is normal. Erythrocyte Sedimentation Rate (ESR) is 58 mm/hour and Serum Ferritin is elevated at 6100 mg/L. Autoimmune Screen is negative.
_x000D_
Which one of the following is the most likely diagnosis?
CorrectIncorrectHint
Still’s Disease is a febrile syndrome in young adults (16-35 years) which affects multiple organs.The diagnosis is mainly one of exclusion.
_x000D_
The clinical features include:
_x000D_
- _x000D_
- High spiking fever (once a day, with return of temperature to normal)
- Arthralgia/arthritis
- Sore throat
- Transient maculopapular rash (mildly pruritic in 1/3)
- Lymphadenopathy
- Hepatosplenomegaly, and
- Pleuritis/pericarditis.
_x000D_
_x000D_
_x000D_
_x000D_
_x000D_
_x000D_
_x000D_
_x000D_
Rarely there may be:
_x000D_
- _x000D_
- Aseptic meningitis
- Cranial nerve palsies
- Iritis, and
- Peripheral neuropathy.
_x000D_
_x000D_
_x000D_
_x000D_
_x000D_
There is often delay in diagnosis.
_x000D_
Hyperferritinaemia (greater than five times normal) is present in 90% of cases.
-
Question 59 of 114
59. Question
A 39-year-old lady with psoriasis has a two week history of painful swollen left knee and difficulty walking. She has early morning stiffness in her left lower limb which lasts for more than an hour.
_x000D_
She drinks 4 units of alcohol a week and has no recent history of having any kind of infection.
_x000D_
On examination there is a left knee effusion, and swollen and tender distal interphalangeal joints in left hand index and middle finger.
_x000D_
Recent Blood Investigations Show:
_x000D_
_x000D_ _x000D_
_x000D_ _x000D_ Haemoglobin
_x000D_
_x000D_
_x000D_ 142 g/L
_x000D_
_x000D_
_x000D_ (130-180)
_x000D_
_x000D_
_x000D_
_x000D_ _x000D_ Total Leucocyte Count
_x000D_
_x000D_
_x000D_ 9.3 ×109/L
_x000D_
_x000D_
_x000D_ (4-11)
_x000D_
_x000D_
_x000D_
_x000D_ _x000D_ Neutrophils
_x000D_
_x000D_
_x000D_ 6.2 ×109/L
_x000D_
_x000D_
_x000D_ (1.5-7.0)
_x000D_
_x000D_
_x000D_
_x000D_ _x000D_ Erythrocyte Sedimentation Rate (ESR)
_x000D_
_x000D_
_x000D_ 43 mm/hr
_x000D_
_x000D_
_x000D_ (0-15)
_x000D_
_x000D_
_x000D_
_x000D_ _x000D_ Serum Urea, Serum Electrolytes And Serum Creatinine
_x000D_
_x000D_
_x000D_ Within Normal Ranges
_x000D_
_x000D_
_x000D_
_x000D_ _x000D_ Rheumatoid Factor
_x000D_
_x000D_
_x000D_ Negative.
_x000D_
_x000D_
_x000D_
_x000D_
_x000D_
Which one of the following is the most likely diagnosis?
CorrectIncorrectHint
This lady has psoriatic arthritis.
_x000D_
Among people who develop psoriatic arthritis, two-thirds develop joint symptoms after skin involvement. In the absence of psoriasis, a positive family history of psoriasis in a first degree relative supports the diagnosis of psoriatic arthritis.
_x000D_
RA is unlikely as there is asymmetrical joint involvement – with predominant involvement of the distal interphalangeal (DIP) joints in the hands. DIP joint involvement is a distinctive feature of psoriatic arthritis. DIP joint involvement associates with dactylitis, nail pitting and onycholysis (separation of nail from nail bed).
_x000D_
The absence of preceding infection makes reactive arthritis unlikely.
_x000D_
Gout is unlikely.
_x000D_
The alcohol intake is not excessive, and except for male gender, there is no other risk factor, for example, age greater than 40 years, diuretic use, renal failure and solid organ transplant.
-
Question 60 of 114
60. Question
A 39-year-old lady presents to the Emergency Department with fever, dry cough, recurrent episodes of sinusitis, and loss of weight for last one month. She was prescribed oral amoxicillin by her General Physician but her symptoms did not subside.
_x000D_
On admission, she is noted to have a temperature of 98.6°F, blood pressure of 130/70 mm Hg, and pedal oedema. She has blood-stained nasal discharge, and is noted to have a stridor.
_x000D_
Recent Blood Tests Show:
_x000D_
_x000D_ _x000D_
_x000D_ _x000D_ Haemoglobin
_x000D_
_x000D_
_x000D_ 114 g/L
_x000D_
_x000D_
_x000D_ (130-180)
_x000D_
_x000D_
_x000D_
_x000D_ _x000D_ Total Leucocyte Count
_x000D_
_x000D_
_x000D_ 12.6 ×109/L
_x000D_
_x000D_
_x000D_ (4-11)
_x000D_
_x000D_
_x000D_
_x000D_ _x000D_ Neutrophils
_x000D_
_x000D_
_x000D_ 86%
_x000D_
_x000D_
_x000D_ (40-75)
_x000D_
_x000D_
_x000D_
_x000D_ _x000D_ Lymphocytes
_x000D_
_x000D_
_x000D_ 12%
_x000D_
_x000D_
_x000D_ (20-45)
_x000D_
_x000D_
_x000D_
_x000D_ _x000D_ Eosinophils
_x000D_
_x000D_
_x000D_ 3%
_x000D_
_x000D_
_x000D_ (1-6)
_x000D_
_x000D_
_x000D_
_x000D_ _x000D_ Platelet Count
_x000D_
_x000D_
_x000D_ 465 ×109/L
_x000D_
_x000D_
_x000D_ (150-400)
_x000D_
_x000D_
_x000D_
_x000D_ _x000D_ Erythrocyte Sedimentation Rate (ESR)
_x000D_
_x000D_
_x000D_ 84 mm/hr
_x000D_
_x000D_
_x000D_ (0-15)
_x000D_
_x000D_
_x000D_
_x000D_ _x000D_ Serum C-Reactive Protein (CRP)
_x000D_
_x000D_
_x000D_ 104 mg/L
_x000D_
_x000D_
_x000D_ (<10)
_x000D_
_x000D_
_x000D_
_x000D_ _x000D_ Anti-Proteinase 3 Antibody
_x000D_
_x000D_
_x000D_ Positive
_x000D_
_x000D_
_x000D_ (Negative)
_x000D_
_x000D_
_x000D_
_x000D_ _x000D_ Anti-Nuclear Antibody
_x000D_
_x000D_
_x000D_ Negative
_x000D_
_x000D_
_x000D_ (Negative at 1:20 Dil)
_x000D_
_x000D_
_x000D_
_x000D_ _x000D_ Anti-Neutrophil Cytoplasmic Antibodies (ANCA)
_x000D_
_x000D_
_x000D_ Positive (Cytoplasmic Pattern)
_x000D_
_x000D_
_x000D_
_x000D_ _x000D_ Urea, Electrolytes & Creatinine
_x000D_
_x000D_
_x000D_ Normal
_x000D_
_x000D_
_x000D_
_x000D_
_x000D_
What is the most likely diagnosis?
CorrectIncorrectHint
Common manifestations of Granulomatosis with polyangiitis include:
_x000D_
- _x000D_
- Constitutional symptoms like fevers, night sweats, fatigue, lethargy, weight loss, arthralgia
- Ocular involvement including conjunctivitis, episcleritis, uveitis, optic nerve vasculitis, and proptosis
- ENT symptoms like chronic sinusitis, rhinitis, otitis media and hearing loss, subglottic stenosis (leading to stridor and features of extrathoracic airway obstruction on flow-volume loop)
- Pulmonary disease, for example, pulmonary infiltrates, cough, haemoptysis, chest discomfort, and dyspnoea
- Renal disease manifests as crescentic necrotising glomerulonephritis
- Nervous system involvement manifests as mononeuritis multiplex, sensorimotor polyneuropathy, cranial nerve palsies, vasculitis of small to medium-sized vessels of the brain or spinal cord, and granulomatous masses that involve the orbit, optic nerve, meninges or brain
- Skin involvement can lead to palpable purpura or skin ulcers.
_x000D_
_x000D_
_x000D_
_x000D_
_x000D_
_x000D_
_x000D_
-
Question 61 of 114
61. Question
A 39-year-old gentleman with systemic lupus erythematosus (SLE) [Antinuclear Antibody (ANA) – Positive (1:1278), dsDNA – Positive, and Anti-Cardiolipin Antibody – Positive (on two occasions)], developed a left below knee deep vein thrombosis (DVT).
_x000D_
He has no past history of arterial or venous thrombosis.
_x000D_
Which one of the following is the most appropriate treatment plan for this gentleman?
CorrectIncorrectHint
For this 39-year-old gentleman with systemic lupus erythematosus (SLE) and a new diagnosis of deep vein thrombosis (DVT), the presence of antiphospholipid antibodies (positive Anti-Cardiolipin Antibody) suggests an increased risk of recurrent thrombotic events. The most appropriate treatment plan would be:
_x000D_
B. Warfarin for life.
_x000D_
Patients with systemic lupus erythematosus (SLE) and antiphospholipid syndrome (APS), indicated by the presence of antiphospholipid antibodies, are at a higher risk for both arterial and venous thromboses. Lifelong anticoagulation with warfarin is recommended for patients with antiphospholipid syndrome (APS) and a history of thrombosis to prevent recurrence.
_x000D_
Let us discuss why the other options are incorrect:
_x000D_
A. Warfarin for 3 months, followed by aspirin 75 mg/day: This option is incorrect because aspirin alone is not sufficient for the secondary prevention of thrombosis in patients with antiphospholipid syndrome (APS). Lifelong anticoagulation is necessary due to the high risk of recurrence.
_x000D_
C. Warfarin for 3 months: This duration is too short for this gentleman with systemic lupus erythematosus (SLE) and antiphospholipid syndrome (APS) who has already experienced a thrombotic event. Lifelong anticoagulation is typically required.
_x000D_
D. Warfarin, and aspirin 75 mg/day for life: The combination of warfarin and aspirin is not routinely recommended due to the increased risk of bleeding without evidence of added benefit in preventing recurrent thrombosis over anticoagulation alone.
_x000D_
E. Warfarin for 3 months, followed by aspirin 75 mg/day: Similar to option A, this option is incorrect because it does not provide adequate long-term prevention of thrombotic events in a patient with systemic lupus erythematosus (SLE) and antiphospholipid syndrome (APS).
_x000D_
In summary, lifelong anticoagulation with warfarin is the most appropriate treatment to prevent recurrent thrombotic events in patients with systemic lupus erythematosus (SLE) and antiphospholipid syndrome (APS).
-
Question 62 of 114
62. Question
A 38-year-old gentleman presents to a Pain Management Clinic with an eight month history of joint pain and stiffness of both hands and both feet.
_x000D_
Examination reveals a synovitis of the distal interphalangeal joints of the right index finger and the left ring finger together with the left wrist and ankle joints. On investigation, his Erythrocyte Sedimentation Rate (ESR) was 36 mm/hr (0-10).
_x000D_
Which one of the following pathological conditions is most likely to exhibit this pattern of joint involvement?
CorrectIncorrectHint
This gentleman is most likely to have psoriatic arthritis.
_x000D_
Psoriatic arthritis has been subclassified according to different patterns of arthritis.
_x000D_
The rash typically predates the arthropathy by a number of years, but the opposite can be true. Small plaques should be looked for on the elbows and scalp.
_x000D_
There are five patterns of disease:
_x000D_
- _x000D_
- Symmetrical polyarthritis (‘rheumatoid pattern’) – affects wrists, hands, feet and ankles. The distal interphalangeal joints are more commonly affected than the metacarpophalangeal joints, which helps to distinguish it from rheumatoid arthritis
- Asymmetrical oligoarticular arthritis: dactylitis
- Distal interphalangeal joint disease: typically in men
- Arthritis mutilans (rare)
- Spondylitic pattern with sacroileitis.
_x000D_
_x000D_
_x000D_
_x000D_
_x000D_
_x000D_
Osteoarthritis in this age group is unlikely.
_x000D_
Rheumatoid arthritis is a symmetrical arthritis typically affecting the metacarpophalangeal joints.
_x000D_
Arthritis does occur in systemic lupus erythematosus, however there are several other clinical features that form part of the diagnositic criteria, none of which are present here.
_x000D_
Reactive arthritis occurs following a gastrointestinal or genitourinary infection. It commonly affects the large joints.
-
Question 63 of 114
63. Question
A 37-year old enthusiastic shuttler presents to sports medicine clinic complaining of dorsoradial wrist pain for last ten months, which has kept him outside the court for about eight months.
_x000D_
On examination, tenderness localised to the dorsoradial aspect of the wrist and passive motion of the thumb causing crepitus in the same region is found. Finkelstein’s test is positive.
_x000D_
Which one of the following is the most likely diagnosis?
CorrectIncorrectHint
The correct option is D. De Quervain’s tenosynovitis. This is a condition that affects the tendons of the thumb and causes pain and inflammation at the base of the thumb and the radial side of the wrist. The symptoms and signs described are consistent with this diagnosis, such as:
_x000D_
- _x000D_
- Dorsoradial wrist pain that worsens with thumb movement or grasping.
- Tenderness and swelling over the first dorsal compartment of the wrist, which contains the abductor pollicis longus and extensor pollicis brevis tendons.
- Crepitus or a grating sensation when moving the thumb.
- Positive Finkelstein’s test, which involves flexing the thumb into the palm and then ulnarly deviating the wrist. This test reproduces the pain and confirms the diagnosis.
_x000D_
_x000D_
_x000D_
_x000D_
_x000D_
The other options are incorrect for the following reasons:
_x000D_
- _x000D_
- A. Tennis elbow, or lateral epicondylitis, is a condition that affects the tendons of the elbow and causes pain and tenderness at the lateral epicondyle of the humerus. It is usually associated with repetitive wrist extension or forearm supination, such as in racquet sports. It does not affect the thumb or the radial side of the wrist, and Finkelstein’s test is negative.
- B. Carpal tunnel syndrome is a condition that affects the median nerve and causes pain, numbness, tingling, and weakness in the thumb, index, middle, and ring fingers. It is usually associated with repetitive wrist flexion or compression of the wrist, such as in keyboard use or driving. It does not cause inflammation or crepitus at the radial side of the wrist, and Finkelstein’s test is negative.
- C. Golfer’s elbow, or medial epicondylitis, is a condition that affects the tendons of the elbow and causes pain and tenderness at the medial epicondyle of the humerus. It is usually associated with repetitive wrist flexion or forearm pronation, such as in golf or throwing sports. It does not affect the thumb or the radial side of the wrist, and Finkelstein’s test is negative.
- E. Ulnar tunnel syndrome, or Guyon’s canal syndrome, is a condition that affects the ulnar nerve and causes pain, numbness, tingling, and weakness in the little and ring fingers. It is usually associated with compression of the ulnar nerve at the wrist, such as in cycling or using crutches. It does not cause inflammation or crepitus at the radial side of the wrist, and Finkelstein’s test is negative.
_x000D_
_x000D_
_x000D_
_x000D_
-
Question 64 of 114
64. Question
A 36-year-old gentleman with shortness of breath, myalgia, arthralgia, and a skin rash is referred by his general physician to the Outpatient Department for further review.
_x000D_
Which one of the following antibodies is most specific for systemic lupus erythematosus (SLE)?
CorrectIncorrectHint
The most specific antibody for systemic lupus erythematosus (SLE) is Anti-Sm antibody (C). Anti-Sm antibodies are highly associated with systemic lupus erythematosus (SLE) and are considered a hallmark of the disease. They are not typically found in other diseases, which makes them quite specific for systemic lupus erythematosus (SLE).
_x000D_
Let us review the other options:
_x000D_
- _x000D_
- c-Anti-neutrophil cytoplasmic antibodies (c-ANCA) (A): These are primarily associated with vasculitis, particularly granulomatosis with polyangiitis, and not specific for systemic lupus erythematosus (SLE).
- Anti-Ro antibody (B): While these antibodies can be present in systemic lupus erythematosus (SLE), they are also found in other autoimmune conditions like Sjögren’s syndrome and are not as specific as Anti-Sm antibodies for systemic lupus erythematosus (SLE).
- Antinuclear antibody (ANA) (D): Antinuclear antibody (ANA) is a sensitive screening test for systemic lupus erythematosus (SLE) but is not specific as it can be positive in a variety of other autoimmune diseases and even in some healthy individuals.
- Rheumatoid factor (E): This antibody is associated with rheumatoid arthritis and other autoimmune diseases, but it is not specific for systemic lupus erythematosus (SLE).
_x000D_
_x000D_
_x000D_
_x000D_
_x000D_
In summary, while several antibodies can be present in systemic lupus erythematosus (SLE), the Anti-Sm antibody (C) is the most specific and is a key marker in the diagnosis of systemic lupus erythematosus (SLE).
-
Question 65 of 114
65. Question
A 36-year-old gentleman presents to the Pain Management Clinic with a two week history of a painful right leg.
_x000D_
His past medical history reveals that he had erythema nodosum and recurrent oral and scrotal ulceration.
_x000D_
Examination reveals a diffusely swollen right leg, and acute tenderness with erythema of the left lower leg.
_x000D_
What is the most likely cause of his swollen right leg?
CorrectIncorrectHint
This gentleman has clinical features of Behçet’s syndrome.
_x000D_
Behçet’s syndrome is a systemic vasculitis with an unknown aetiology, which affects small and large vessels (venous and arterial).
_x000D_
More than 60% of patients are HLA-B51, and there is an increased prevalence in the Mediterranean countries.
_x000D_
It is commonly associated with mucocutaneous manifestations (oro-genital ulceration, erythema nodosum), ocular disease, gastrointestinal involvement and neurological features. Venous thrombosis is a common complication, and therefore there should be a high clinical suspicion of a left sided DVT in this case. The tenderness and erythema of the right leg is most likely due to erythema nodosum.
_x000D_
None of the other conditions listed above are commonly associated with Behçet’s syndrome.
-
Question 66 of 114
66. Question
A 36-year-old gentleman presents to the Clinic with a two week history of acute left ankle pain and swelling.
_x000D_
He has urinary frequency, dysuria, painless rash on the glans penis, and a painful red left eye. No oral or genital ulcers are seen during examination.
_x000D_
Which one of the following is the most likely diagnosis?
CorrectIncorrectHint
This patient has reactive arthritis (a reactive illness with concurrent arthritis, non-infective urethritis, and conjunctivitis).
_x000D_
Other manifestations of reactive arthritis include:
_x000D_
- _x000D_
- Psoriasiform skin
- Nail and mucosal lesions
- Keratoderma blenorrhagica (palmoplantar pustulosis), and
- Circinate balanitis.
_x000D_
_x000D_
_x000D_
_x000D_
_x000D_
Behcet’s disease is unlikely as there are no orogenital ulcers. Genital ulceration in Behcet’s disease affects the scrotum in men and labia in women.
_x000D_
Peripheral arthropathy in inflammatory bowel disease (IBD) is oligoarticular, asymmetric, and predominantly involves the lower limbs. The lack of a history of diarrhoea and presence of urethritis, makes IBD unlikely.
_x000D_
Sarcoidosis leads to ankle and ocular involvement but does not lead to urethritis.
_x000D_
Similarly, Whipple’s disease manifests as:
_x000D_
- _x000D_
- Polyarticular symmetrical arthropathy
- Diarrhoea
- Weight loss
- Lymphadenopathy, and
- Fever.
_x000D_
_x000D_
_x000D_
_x000D_
_x000D_
_x000D_
Whipple’s disease does not lead to urethritis.
-
Question 67 of 114
67. Question
A 41-year-old lady presents to the Outpatient Department with an acutely painful red eye and painful lumpy bluish lesions on her shin.
_x000D_
On further questioning she gives a history of recurrent episodes of oral and genital ulcers over the past one year. Some of these ulcers have been scarring.
_x000D_
Recent blood tests show a normocytic, normochromic anaemia, normal Liver Function Tests (LFTs), normal Serum Urea, Serum Electrolytes and Serum Creatinine, and a raised Erythrocyte Sedimentation Rate (ESR) of 58 mm/hour.
_x000D_
Which one of the following is the most likely diagnosis?
CorrectIncorrectHint
This lady has Behçet’s disease. This is a clinical diagnosis. Recurrent scarring oro-genital ulcers are pathognomonic of Behçet’s disease. Recurrent oral ulcers occur in SLE but they are generally non-scarring.
_x000D_
Reactive arthritis and sarcoidosis do not associate with recurrent scarring oro-genital ulcers.
_x000D_
Uveitis does not occur in Steven-Johnson’s syndrome.
_x000D_
The International Study Group criteria for classification of Behçet’s disease require the presence of recurrent oral ulceration (minor aphthous, major aphthous or herpetiform ulceration observed by physician or patient, which have recurred at least three times in a 12 month period), and two of the following:
_x000D_
- _x000D_
- Recurrent genital ulceration – aphthous ulceration or scarring, observed by physician or patient
- Eye lesions – anterior uveitis, posterior uveitis, or cells in vitreous on slit lamp examination; or retinal vasculitis observed by ophthalmologist
- Skin lesions – erythema nodosum observed by physician or patient, pseudofolliculitis or papulopustular lesions; or acneiform nodules observed by the physician in post-adolescent patients not on corticosteroid treatment
- Positive pathergy test – read by physician at 24-48 hours.
_x000D_
_x000D_
_x000D_
_x000D_
_x000D_
Pathergy is the non-specific hyperreactivity of the skin following minor trauma, and is specific to Behçet’s disease. It involves intradermal injection of skin with a 20-gauge needle under sterile conditions. It is considered positive if an erythematous sterile papule develops within 48 hours.
-
Question 68 of 114
68. Question
A 42-year-old gentleman presents to the clinic with a two-month history of progressively increasing pain, swelling and stiffness in both knees. His symptoms are worse in the morning, and it takes almost an hour to loosen up the joints. He has had no recent history of illness and there is no personal or family history of any chronic skin conditions.
_x000D_
Since he was a boy, he has had painful fingers and toes when they are exposed to cold weather, but his digits do not change colour. He has also recently had pain and stiffness in his fingers and toes in the morning, and this fluctuates from day to day. He drinks alcohol only occasionally.
_x000D_
On examination, he is found to have reduced flexion and extension and an effusion in both knees. He has bilateral metatarsalgia on squeezing his toes. Examination of his fingers is normal and there is no psoriasis of his skin or nails.
_x000D_
Initial Investigation Showed:
_x000D_
_x000D_ _x000D_
_x000D_ _x000D_ Haemoglobin
_x000D_
_x000D_
_x000D_ 133 g/L
_x000D_
_x000D_
_x000D_ (130-180)
_x000D_
_x000D_
_x000D_
_x000D_ _x000D_ Total Leucocyte Count
_x000D_
_x000D_
_x000D_ 8.4 ×103/µl
_x000D_
_x000D_
_x000D_ (4-11)
_x000D_
_x000D_
_x000D_
_x000D_ _x000D_ Neutrophil Count
_x000D_
_x000D_
_x000D_ 5.2 ×103/µl
_x000D_
_x000D_
_x000D_ (1.5-7)
_x000D_
_x000D_
_x000D_
_x000D_ _x000D_ Platelet Count
_x000D_
_x000D_
_x000D_ 279 ×103/µl
_x000D_
_x000D_
_x000D_ (150-400)
_x000D_
_x000D_
_x000D_
_x000D_ _x000D_ Erythrocyte Sedimentation Rate (ESR)
_x000D_
_x000D_
_x000D_ 46 mm/hr
_x000D_
_x000D_
_x000D_ (0-20)
_x000D_
_x000D_
_x000D_
_x000D_ _x000D_ Serum Urea
_x000D_
_x000D_
_x000D_ 5.2 mEq/L
_x000D_
_x000D_
_x000D_ (2.5-7.5)
_x000D_
_x000D_
_x000D_
_x000D_ _x000D_ Serum Creatinine
_x000D_
_x000D_
_x000D_ 81 mEq/L
_x000D_
_x000D_
_x000D_ (60-110)
_x000D_
_x000D_
_x000D_
_x000D_ _x000D_ Serum Sodium
_x000D_
_x000D_
_x000D_ 140 mEq/L
_x000D_
_x000D_
_x000D_ (137-144)
_x000D_
_x000D_
_x000D_
_x000D_ _x000D_ Serum Potassium
_x000D_
_x000D_
_x000D_ 4.1 mEq/L
_x000D_
_x000D_
_x000D_ (3.5-4.9)
_x000D_
_x000D_
_x000D_
_x000D_
_x000D_
Rheumatoid Factor (RF): Positive (1:254).
_x000D_
Antinuclear Antibody (ANA): Positive (1:40).
_x000D_
Which one of the following is the most likely diagnosis?
CorrectIncorrectHint
The most likely diagnosis for this gentleman with bilateral knee pain, swelling, stiffness, and positive rheumatoid Factor (RF) and antinuclear antibody (ANA) is C. Rheumatoid arthritis (RA). Rheumatoid arthritis (RA) is a chronic inflammatory disorder that can affect more than just the joints. In some people, the condition can damage a wide variety of body systems, including the skin, eyes, lungs, heart, and blood vessels. The morning stiffness lasting almost an hour, the positive rheumatoid factor (RF) and antinuclear antibody (ANA), and the absence of psoriasis or family history of chronic skin conditions support the diagnosis of rheumatoid arthritis (RA) over the other options.
_x000D_
The other options are incorrect for the following reasons:
_x000D_
- _x000D_
- Pseudogout: This condition, also known as calcium pyrophosphate deposition (CPPD), typically affects older adults and is characterized by the presence of calcium pyrophosphate crystals in the joint fluid. This gentleman’s age and the absence of crystal identification in the synovial fluid make this diagnosis less likely.
- Reactive arthritis: This form of arthritis develops in response to an infection in another part of the body (cross-reactivity). Given the lack of recent illness history and the chronic nature of the symptoms, reactive arthritis is less likely.
- Psoriatic arthritis: This type of arthritis occurs in some persons with psoriasis, a condition that features red patches of skin topped with silvery scales. This gentleman has no psoriasis or nail changes, which are common in psoriatic arthritis.
- Systemic lupus erythematosus (SLE): Systemic lupus erythematosus (SLE) is an autoimmune disease that causes widespread inflammation and tissue damage in the affected organs. It can present with joint pain and positive antinuclear antibody (ANA); however, the absence of other systemic symptoms makes this diagnosis less likely compared to rheumatoid arthritis (RA).
_x000D_
_x000D_
_x000D_
_x000D_
_x000D_
In conclusion, based on the clinical presentation and serological findings, rheumatoid arthritis (RA) is the most probable diagnosis in this case.
-
Question 69 of 114
69. Question
A 43-year-old gentleman notices that he develops tingling and numbness over the palmar surface of his thumb, index, and middle fingers after several hours at his computer desk doing webpage designing. He has noticed that pain in the same area often occurs at night as well.
_x000D_
Which one of the following pathological findings is most likely responsible for his symptoms?
CorrectIncorrectHint
Carpal tunnel syndrome is one of the most common entrapment neuropathies, and is a recognised occupational disease. The carpal tunnel is an anatomical compartment bounded on three sides by the carpal bones, and the transverse carpal ligament on the palmar side. Intermittent or sustained high pressure within this compartment produces ischaemia of the median nerve, resulting in the classical symptoms or paraesthesia and pain. If allowed to progress, weakness and wasting develop which eventually become irreversible.
_x000D_
Nerve conduction studies are an important diagnostic test, and are the best predictor of symptom severity and functional status. Treatment depends on severity, and includes splinting, corticosteroid interventions or surgical decompression. In the future pulsed radio frequency may be used.
_x000D_
Gout is caused by the deposition of monosodium urate crystals within a joint, leading to excruitiating pain and swelling. It is not a common cause of the symptoms described above.
_x000D_
Hypertrophic osteoarthropathy may occur secondary to primary lung carcinoma. It presents with clubbing, arthralgia, arthritis and periostosis of the tubular bones. Pain is severe and is present throughout the day.
_x000D_
Rheumatoid arthritis is a possibility in women of this age group, but more commonly presents with swelling and pain of the small joints of the hand.
_x000D_
Toxic neuropathy presents with weakness, sensory loss and reduced reflexes secondary to diffuse nerve damage. This can be caused by a variety of agents but is much less common than carpal tunnel syndrome, making it a less likely answer in this case.
-
Question 70 of 114
70. Question
A 42-year-old gentleman with a three month history of bloody diarrhoea, cramping abdominal pain and weight loss presents to the general physician with a one month history of painful and swollen right knee. He has no other significant past medical history or family history.
_x000D_
Stool cultures done by his general physician have been negative for Clostridium difficile toxin, and have not grown any pathogenic organisms. On examination, his right knee is warm, tender, and there is a large effusion.
_x000D_
Which one of the following is the most likely diagnosis?
CorrectIncorrectHint
The correct answer is E. Inflammatory arthritis associated with inflammatory bowel disease.
_x000D_
Inflammatory bowel disease (IBD) is a group of disorders that cause chronic inflammation of the gastrointestinal tract, such as Crohn’s disease and ulcerative colitis. Arthritis is a common extraintestinal manifestation of inflammatory bowel disease (IBD), affecting 7 to 20 percent of patients. The arthritis can affect the large joints of the lower extremities, such as the knee, as well as the spine and sacroiliac joints. The arthritis usually correlates with the activity of the bowel disease, and may improve with treatment of the underlying inflammatory bowel disease (IBD).
_x000D_
The other options are incorrect for the following reasons:
_x000D_
- _x000D_
- A. Carcinomatous arthropathy is a rare paraneoplastic syndrome that causes arthritis and periostitis in patients with malignancies, such as lung, breast, or prostate cancer. This gentleman has no history of cancer or other risk factors, and the arthritis is more likely related to the bowel disease.
- B. Gout is a metabolic disorder that causes hyperuricaemia and deposition of monosodium urate crystals in the joints, leading to acute inflammatory arthritis. This gentleman has no history of gout attacks, tophi, or renal stones, and the stool cultures are negative for infection, which can trigger gout flares. The arthritis is more likely related to the bowel disease.
- C. Psoriatic arthritis is a type of spondyloarthritis that affects patients with psoriasis, a chronic skin condition that causes scaly, red patches. This gentleman has no history or signs of psoriasis, and the arthritis is more likely related to the bowel disease.
- D. Reactive arthritis is a type of spondyloarthritis that occurs after an infection, usually of the gastrointestinal or genitourinary tract. This gentleman has no history of recent infection, and the stool cultures are negative for pathogenic organisms. The arthritis is more likely related to the bowel disease.
_x000D_
_x000D_
_x000D_
_x000D_
-
Question 71 of 114
71. Question
A 44-year-old gentleman who has recently returned from a holiday to Mexico, presents to the Clinic with frequent thirst, polyuria and a rash affecting his both shins. His only past medical history of note is mild asthma, which was diagnosed by his General Physician, but he states that the inhaler which was prescribed has made no difference to his symptoms.
_x000D_
On examination, his blood pressure is 140/80 mmHg, pulse rate is 72 beats per minute and regular. There are small palpable lymph nodes on palpation of the neck and axillae. His chest is clear on auscultation, and his abdomen is soft and non-tender. He has a raised purpuric rash on both of his shins. His Body Mass Index (BMI) is 30 Kg/m2.
_x000D_
Investigations Show:
_x000D_
_x000D_ _x000D_
_x000D_ _x000D_ Haemoglobin
_x000D_
_x000D_
_x000D_ 122 g/l
_x000D_
_x000D_
_x000D_ (115-160)
_x000D_
_x000D_
_x000D_
_x000D_ _x000D_ White Blood Cell Count
_x000D_
_x000D_
_x000D_ 9.8×109/l
_x000D_
_x000D_
_x000D_ (6-10 [Atypical Lymphocytes Seen On Film])
_x000D_
_x000D_
_x000D_
_x000D_ _x000D_ Platelet Count
_x000D_
_x000D_
_x000D_ 221×109/l
_x000D_
_x000D_
_x000D_ (150-400)
_x000D_
_x000D_
_x000D_
_x000D_ _x000D_ Serum Sodium
_x000D_
_x000D_
_x000D_ 141 mmol/l
_x000D_
_x000D_
_x000D_ (135-145)
_x000D_
_x000D_
_x000D_
_x000D_ _x000D_ Serum Potassium
_x000D_
_x000D_
_x000D_ 4.3 mmol/l
_x000D_
_x000D_
_x000D_ (3.5-5.5)
_x000D_
_x000D_
_x000D_
_x000D_ _x000D_ Serum Creatinine
_x000D_
_x000D_
_x000D_ 107 µmol/l
_x000D_
_x000D_
_x000D_ (50-90)
_x000D_
_x000D_
_x000D_
_x000D_ _x000D_ Serum C-Reactive Protein (CRP)
_x000D_
_x000D_
_x000D_ 52 mg/dl
_x000D_
_x000D_
_x000D_ (<10)
_x000D_
_x000D_
_x000D_
_x000D_ _x000D_ Erythrocyte Sedimentation Rate (ESR)
_x000D_
_x000D_
_x000D_ 43 mm/hr
_x000D_
_x000D_
_x000D_ (<10)
_x000D_
_x000D_
_x000D_
_x000D_ _x000D_ Serum Calcium
_x000D_
_x000D_
_x000D_ 2.83 mmol/l
_x000D_
_x000D_
_x000D_ (2.1-2.65)
_x000D_
_x000D_
_x000D_
_x000D_
_x000D_
Which one of the following is the most likely underlying diagnosis?
CorrectIncorrectHint
The answer is sarcoidosis. The clues here include the non-responsiveness of chest symptoms to the salbutamol inhaler, erythema nodosum, non-specific elevation in inflammatory markers, and hypercalcaemia.
_x000D_
Patients with sarcoid frequently present with symptoms of hypercalcaemia after a holiday because increased sun exposure results in increased production of vitamin D. A chest radiograph, not reported here, may show a variety of abnormalities, including bilateral hilar lymphadenopathy. The mainstay of treatment of sarcoidosis is with oral corticosteroids, but patients with mild symptoms can be managed with surveillance alone.
None of the other options listed commonly causes hypercalcaemia, and the inflammatory markers are only mildly elevated, reducing the probability of her symptoms being due to a chronic infection such as tuberculosis.
_x000D_
Granulomatosis with polyangiitis is associated with symptoms of sinusitis due to granuloma formation, and Churg-Strauss syndrome would explain the asthma-like symptoms but not the others.
-
Question 72 of 114
72. Question
A 44-year-old lady has a history of multiple episodes of sudden onset of severe abdominal cramps and backache lasting for a considerable period of time. Each time this happens, her peripheral blood smear demonstrates numerous sickled erythrocytes.
_x000D_
Haemoglobin Electrophoresis Shows:
_x000D_
_x000D_ _x000D_
_x000D_ _x000D_ Hgb S
_x000D_
_x000D_
_x000D_ 92%
_x000D_
_x000D_
_x000D_
_x000D_ _x000D_ Hgb F
_x000D_
_x000D_
_x000D_ 6%
_x000D_
_x000D_
_x000D_
_x000D_ _x000D_ Hgb A2
_x000D_
_x000D_
_x000D_ 2%
_x000D_
_x000D_
_x000D_
_x000D_
_x000D_
She now has increasing pain in her left groin radiating to the anterior aspect of the left thigh and to the left knee. She is pyrexial with a temperature of 100.4°F and examination of her hip revealed pain on internal rotation. X-ray reveals irregular bony destruction of the femoral head.
_x000D_
Which one of the following organisms is most likely to be responsible for these findings?
CorrectIncorrectHint
The most likely organism responsible for the findings in this lady with sickle cell disease is Salmonella species (D). Patients with sickle cell disease are particularly susceptible to osteomyelitis caused by Salmonella due to their impaired splenic function. The clinical presentation of pain in the groin radiating to the thigh and knee, fever, and x-ray findings of irregular bony destruction are consistent with osteomyelitis, which is a known complication in sickle cell disease.
_x000D_
Let us address the other options:
_x000D_
- _x000D_
- Group B Streptococcus (A): While Group B Streptococcus can cause bone infections, it is more commonly associated with neonatal infections and is less likely to be the cause of osteomyelitis in a patient with sickle cell disease.
_x000D_
_x000D_
- _x000D_
- Clostridium perfringens (B): This organism is typically associated with gas gangrene and is not a common cause of osteomyelitis.
_x000D_
_x000D_
- _x000D_
- Yersinia pestis (C): It is the causative agent of the plague and does not typically cause osteomyelitis.
_x000D_
_x000D_
- _x000D_
- Candida albicans (E): While Candida species can cause osteomyelitis, it is usually in the setting of an immunocompromised host or intravenous drug use and is less likely in this clinical scenario.
_x000D_
_x000D_
In summary, the clinical presentation, along with the patient’s history of sickle cell disease, strongly suggests Salmonella species (D) as the causative organism for the osteomyelitis in the femoral head. The presence of sickled erythrocytes on peripheral blood smear and the haemoglobin electrophoresis results further support the diagnosis of sickle cell disease, which increases the risk of Salmonella infection.
-
Question 73 of 114
73. Question
A 48-year-old gentleman with rheumatoid arthritis is reviewed in the rheumatology clinic and commencement of methotrexate was planned.
_x000D_
How often does he require blood tests to be done after he starts taking methotrexate?
CorrectIncorrectHint
The correct answer is D. CBC, LFT and U&E at baseline and then two weekly until on stable dose of methotrexate and then every month. This is based on the guidelines for monitoring methotrexate toxicity in rheumatoid arthritis patients, which recommend that complete blood count (CBC), liver function tests (LFT) and urea and electrolytes (U&E) should be measured at baseline and then every two weeks until the gentleman is on a stable dose of methotrexate, and then every month thereafter. This is to ensure that the drug is well-tolerated and does not cause any adverse effects on the blood cells, liver or kidneys.
_x000D_
The other options are incorrect for the following reasons:
_x000D_
- _x000D_
- A. CBC, coagulation screen, LFT and ESR at baseline and then weekly until on stable dose of methotrexate and then every three months: This option is incorrect because it includes a coagulation screen and an erythrocyte sedimentation rate (ESR), which are not necessary for monitoring methotrexate toxicity. Coagulation screen is used to assess the clotting ability of the blood, and erythrocyte sedimentation rate (ESR) is a marker of inflammation, but neither of them are affected by methotrexate. Moreover, this option suggests testing every three months after reaching a stable dose, which is too infrequent and may miss early signs of toxicity.
- B. CBC, coagulation screen, U&E and Rheumatoid Factor weekly, includes baseline levels: This option is incorrect because it also includes a coagulation screen, which is not required for monitoring methotrexate toxicity, as explained above. Additionally, this option includes rheumatoid factor (RF), which is an antibody that can be detected in some patients with rheumatoid arthritis, but is not affected by methotrexate. Rheumatoid factor (RF) is used to diagnose rheumatoid arthritis, but not to monitor methotrexate therapy. Furthermore, this option suggests testing weekly, which is too frequent and unnecessary, as methotrexate toxicity usually develops over weeks to months.
- C. CBC, LFT and U&E monthly, including baseline levels: This option is incorrect because it suggests testing monthly from the start of methotrexate therapy, which is not frequent enough to detect early toxicity. Methotrexate toxicity can occur within the first few weeks of treatment, especially in patients with renal impairment, obesity, diabetes, or alcohol use. Therefore, testing every two weeks until a stable dose is achieved is recommended.
- E. CBC, uric acid, folate levels, U&E and LFT at baseline and then weekly until on stable dose of methotrexate and then every two to three months: This option is incorrect because it includes uric acid and folate levels, which are not necessary for monitoring methotrexate toxicity. Uric acid is a waste product that can accumulate in the blood and cause gout, but methotrexate does not affect uric acid levels. Folate is a vitamin that is essential for DNA synthesis and cell division, and methotrexate inhibits folate metabolism, which can lead to folate deficiency. However, folate deficiency can be prevented by taking folic acid supplements, which are usually prescribed along with methotrexate. Therefore, measuring folate levels is not required for monitoring methotrexate therapy. Moreover, this option suggests testing every two to three months after reaching a stable dose, which is too infrequent and may miss early signs of toxicity.
_x000D_
_x000D_
_x000D_
_x000D_
-
Question 74 of 114
74. Question
A 47-year-old gentleman presented to the Clinic with a one week history of pain and stiffness in his shoulders and wrists which was noticeably worse in the mornings.
_x000D_
On examination, there was synovitis of both wrists. There was no proximal muscle tenderness or weakness. His Erythrocyte Sedimentation Rate (ESR) was 48 mm/hr (0-20).
_x000D_
Which one of the following is the most likely diagnosis?
CorrectIncorrectHint
This is a slightly difficult question, but you may experience similar during the exam. It tests your ability to consider epidemiology as well as your knowledge of disease presentations.
_x000D_
In this middle aged gentleman, the acute bilateral arthritis of shoulders and wrists together with synovitis and raised ESR are highly suggestive of acute rheumatoid arthritis. The features given do not fully satisfy the ACR criteria (see the reference for updated guidelines), but it is not unusual for this to be the case in clinical practice.
_x000D_
Weakness and myalgia would be expected with polymyositis and a rash would be expected with systemic lupus erythematosus with less evidence of a synovitis.
_x000D_
There is no prior precipitant to suggest a reactive arthritis, although it is important to consider this diagnosis in young patients.
_x000D_
PMR would be less likely in this age group as it usually occurs in patients over 50 years of age
_x000D_
Proximal weakness in the morning with the gel phenomenon would be expected, and synovitis in the wrists would be less likely in PMR.
-
Question 75 of 114
75. Question
A 46-year-old lady was diagnosed with rheumatoid arthritis last year after presenting with small hand joint synovitis and stiffness. She now presents to the Rheumatology Clinic for an annual review.
_x000D_
At that time her Rheumatoid Factor and Anti-CCP levels were 1:20 Titre and 700 U/ml respectively. Unfortunately, despite twelve months on a combination of Prednisolone, Methotrexate and Leflunomide, she still has moderate disease activity based on a DAS-28 (Disease Activity Score) Score of 3.3.
_x000D_
She is being considered for Etanercept.
_x000D_
Preliminary tests demonstrate a normal Chest Radiograph, but positive Quantiferon Test. She is asymptomatic.
_x000D_
Which one of the following is the most appropriate treatment for this lady?
CorrectIncorrectHint
It is important to be aware of the following teaching points:
_x000D_
- _x000D_
- Recommendations for biologic therapy in rheumatoid arthritis (RA)
- Contraindications to biologic therapy, and
- Basic disease scoring in RA.
_x000D_
_x000D_
_x000D_
_x000D_
This lady has a DAS-28 score persistently greater than 3.2 and has failed on a combination of more than two disease modifying agents, thus fulfilling the criteria for consideration of anti-TNF (biologic) therapy.
_x000D_
Patients receiving anti-TNF alpha treatment have an increased risk of clinical tuberculosis (TB) development. The current British Thoracic Society guidelines therefore recommend a clinical examination, chest radiograph and thorough history taken to check for prior TB.
_x000D_
Any patient with an abnormal chest radiograph or previous history of TB should be referred for assessment by a specialist with an interest in TB. Those with symptoms raising a suspicion of TB should be thoroughly investigated to exclude active disease. Any patient with active TB, either pulmonary or non-pulmonary, should receive standard chemotherapy. They must complete two months full treatment before starting anti-TNF alpha treatment.
_x000D_
Patients with an abnormal chest radiograph consistent with past TB, or a history or prior extrapulmonary TB but who have received previous adequate therapy can be started on anti-TNF alpha therapy but need to be monitored regularly.
_x000D_
Where there is an abnormal chest radiograph, or a history of prior pulmonary or extrapulmonary TB not previously adequately treated, chemoprophylaxis should be given before commencing anti-TNF alpha treatment.
_x000D_
For patients with a normal chest radiograph who have not started immunosuppressive threrapy, a tuberculin test is helpful.
_x000D_
If a patient is already on immunosuppressive treatment, the result of the tuberculin test is dampened and it is therefore not useful. An individual risk assessment should be made: if the annual risk of TB is greater than that of drug-induced hepatitis then chemoprophylaxis should be given. If not, the patient should be monitored and investigated early if symptoms consistent with TB develop.
_x000D_
Chemoprophylaxis is generally with isoniazid for 6 months.
_x000D_
The BTS guidelines have not been updated to include recommendations regarding Quantiferon and Elispot tests. Practice does vary between individual NHS trusts regarding who to test with one of these, and which to use. However, the most accepted recommendations are that patients who test positive with either of these should be treated with chemoprophylaxis (either isoniazid for 6 months, or dual therapy for two months) at the same time as being started on anti-TNF alpha treatment.
-
Question 76 of 114
76. Question
A 46-year-old lady presents with acute monoarthritis of the left knee.
_x000D_
Gout is confirmed following joint aspiration and examination of the fluid under polarised light microscopy. She underwent endoscopy five weeks earlier because of dyspepsia and this confirmed a duodenal ulcer.
_x000D_
Which one of the following would be the best initial treatment for her acute episode of gout?
CorrectIncorrectHint
The best initial treatment for an acute episode of gout in this case would be Option D: Intra-articular corticosteroid injection. This option is recommended for patients with a first or infrequent recurrent gout flare affecting one to two joints, and it can be delivered in a timely manner. It is especially suitable in this scenario where this lady has a history of duodenal ulcer, as non-steroidal anti-inflammatory drugs (NSAIDs) like indomethacin could exacerbate her gastrointestinal condition.
_x000D_
The other options are incorrect because:
_x000D_
- _x000D_
- Option A: Indomethacin and misoprostol is not the best choice because, while misoprostol can protect against non-steroidal anti-inflammatory drug (NSAID)-induced gastric ulcers, this lady’s recent duodenal ulcer makes the use of non-steroidal anti-inflammatory drugs (NSAIDs) risky.
- Option B: Indomethacin alone should be avoided due to the high risk of exacerbating the patient’s duodenal ulcer.
- Option C: Indomethacin and lansoprazole is also not ideal. Lansoprazole is a proton pump inhibitor that can be used for gastroprotection, but again, the use of indomethacin is contraindicated due to the patient’s duodenal ulcer.
- Option E: Allopurinol is not used as an initial treatment for acute gout flares. It is a urate-lowering medication used in the long-term management of gout and should not be started during an acute episode. However, if a patient is already on allopurinol, it can be continued.
_x000D_
_x000D_
_x000D_
_x000D_
_x000D_
In conclusion, given this lady’s history of duodenal ulcer, intra-articular corticosteroid injection is the safest and most effective initial treatment for her acute gout episode.
-
Question 77 of 114
77. Question
A 46-year-old lady presents to the Outpatient Department with a seven month history of dyspepsia. She has a four year history of Raynaud’s Phenomenon.
_x000D_
On examination, she has telangiectasia. Her investigations reveal an Erythrocyte Sedimentation Rate (ESR) of 42 mm/hr (0-10) and positive anticentromere antibodies.
_x000D_
Which one of the following is a typical late complication of this disorder?
CorrectIncorrectHint
The history here is suggestive of a diagnosis of systemic sclerosis.
_x000D_
Pulmonary hypertension is more commonly associated with the diffuse form of systemic sclerosis, rather than the limited cutaneous form (previously referred to as CREST syndrome). Telangiectasia and dyspepsia can be associated with either form. Anti-centromere antibodies are associated with an increased risk of pulmonary hypertension.
_x000D_
Systemic sclerosis is a chronic autoimmune disease characterised by increased fibroblast activity and fibrosis in a number of different organ systems. 90-95% of patients have positive antinuclear antibodies. There are two major subtypes: limited cutaneous and diffuse cutaneous. CREST syndrome is an older term for the limited cutaneous form (calcinosis, Raynauds’ phenomenon, oeosophageal dysmotility, sclerodactyly, telangiectasia).
_x000D_
Patients with systemic sclerosis can present with skin abnormalities, musculoskeletal changes, gastrointestinal complications, pulmonary disease, renal crisis and dry eyes and mouth. Dyspepsia, restricted distension of the gastric antrum and diffuse gastrointestinal dysmotility are frequent features. This is due both to fibrosis and muscle atrophy, and autonomic neuropathy.
_x000D_
Pulmonary hypertension can present in isolation in systemic sclerosis, or in association with interstitial lung disease or cardiac dysfunction. It is a frequent cause of morbidity and mortality. Despite advances in echocardiography and biomarkers, right heart catheterisation remains the diagnostic test to differentiate pulmonary veno-occlusive disease from hypertension. This is essential because pulmonary vasodilator therapy can increase mortality in veno-occlusive disease. Recent studies have investigated the use of non-invasive screening, but their use remains controversial.
_x000D_
Treatment options are limited, and prognosis is worse than with idiopathic pulmonary hypertension. Median survival is one year from diagnosis. Extrapulmonary disease in systemic sclerosis limit candidacy for lung transplantation. New therapies that target abnormal cellular proliferation in the pulmonary vasculature are currently under investigation.
_x000D_
Erosive polyarthropathy is typically present in the earlier stages of systemic sclerosis, and would not be classified as a late complication.
_x000D_
A malar, or butterfly, rash is classically associated with systemic lupus erythematosus. It involves the bridge of the nose, but spares the naso-labial folds. It is usually well demarcated and macular. It is not pathognomonic of SLE, and can be seen in pellagra and dermatomyositis, but again none of these would account for the symptoms described above.
_x000D_
Alopecia and myositis are not commonly associated with systemic sclerosis.
-
Question 78 of 114
78. Question
A 46-year-old gentleman presents to the Rheumatology Clinic with a small joint polyarthritis and significant morning stiffness which has increased over the past few months.
_x000D_
On examination he has a symmetrical small joint polyarthritis affecting the proximal interphalangeal joints, metatarsophalangeal joints, wrists, elbows and knees; otherwise his the physical examination is unremarkable.
_x000D_
Blood Investigations Reveal:
_x000D_
_x000D_ _x000D_
_x000D_ _x000D_ Haemoglobin
_x000D_
_x000D_
_x000D_ 125 g/L
_x000D_
_x000D_
_x000D_ (115-160)
_x000D_
_x000D_
_x000D_
_x000D_ _x000D_ Total Leucocyte Count
_x000D_
_x000D_
_x000D_ 7.2 ×109/L
_x000D_
_x000D_
_x000D_ (4-10)
_x000D_
_x000D_
_x000D_
_x000D_ _x000D_ Platelet Count
_x000D_
_x000D_
_x000D_ 210 ×109/L
_x000D_
_x000D_
_x000D_ (150-400)
_x000D_
_x000D_
_x000D_
_x000D_ _x000D_ Serum Sodium
_x000D_
_x000D_
_x000D_ 142 mmol/L
_x000D_
_x000D_
_x000D_ (134-143)
_x000D_
_x000D_
_x000D_
_x000D_ _x000D_ Serum Potassium
_x000D_
_x000D_
_x000D_ 4.4 mmol/L
_x000D_
_x000D_
_x000D_ (3.5-5)
_x000D_
_x000D_
_x000D_
_x000D_ _x000D_ Serum Creatinine
_x000D_
_x000D_
_x000D_ 84 μmol/L
_x000D_
_x000D_
_x000D_ (60-120)
_x000D_
_x000D_
_x000D_
_x000D_ _x000D_ Rheumatoid Factor
_x000D_
_x000D_
_x000D_ Negative
_x000D_
_x000D_
_x000D_
_x000D_ _x000D_ Anti-CCP Antibody
_x000D_
_x000D_
_x000D_ Positive
_x000D_
_x000D_
_x000D_
_x000D_
_x000D_
Which one of the following is the most likely diagnosis?
CorrectIncorrectHint
Anticyclic citrullinated protein (CCP) antibodies were identified as early as the 1970s in patients with rheumatoid arthritis, but it is only recently that more specific assays have become available.
_x000D_
They are the most specific biomarker associated with the diagnosis of rheumatoid arthritis. It has been suggested that citrullination and the anticitrullinated peptide antibodies play a critical role in initiating the inflammatory response within rheumatoid arthritis.
_x000D_
In patients with the clinical picture of rheumatoid arthritis who are rheumatoid factor negative, anti-CCP antibodies can aid in making the diagnosis between rheumatoid and other causes of arthritis.
_x000D_
Positive rheumatoid factor is associated with a worse prognosis in rheumatoid arthritis.
_x000D_
Polymyalgia rheumatica is associated with a markedly raised erythrocyte sedimentation rate (ESR) and is characterised by severe bilateral pain and morning stiffness of the shoulder, neck and pelvic girdle.
_x000D_
Reactive arthritis is the triad of arthritis, urethritis and conjunctivitis which is classically associated with sexually transmitted or gastrointestinal infection.
_x000D_
Seronegative arthritis is a heterogenous group of inflammatory rheumatic disease with predominant involvement of axial and peripheral joints, and enthesis.
_x000D_
There is a high incidence of HLA-B27, but rheumatoid factor is typically negative. Diseases belonging to this group include ankylosing spondylitis, reactive arthritis, psoriatic arthritis and Behçet’s disease.
_x000D_
Systemic lupus erythematosus (SLE) is a heterogenous, multisystem, inflammatory autoimmune condition which is characterised by positive antinuclear antibodies.
-
Question 79 of 114
79. Question
A 45-year-old lady presents with a two year history of Raynaud’s Phenomenon, dyspepsia and arthralgia.
_x000D_
On examination, she has sclerodactyly and synovitis of the small joints of the hands. Her Erythrocyte Sedimentation Rate (ESR) is 38 mm/hr (<20), Antinuclear Antibody (ANA) is positive and Rheumatoid Factor is negative.
_x000D_
Which one of the following is most likely to develop as a further complication of this disorder?
CorrectIncorrectHint
Systemic sclerosis is a chronic autoimmune disease characterised by increased fibroblast activity and fibrosis in a number of different organ systems. 90-95% of patients have positive antinuclear antibodies. There are two major subtypes: limited cutaneous and diffuse cutaneous. CREST syndrome is an older term for the limited cutaneous form (calcinosis, Raynauds’ phenomenon, oeosophageal dysmotility, sclerodactyly, telangiectasia). Patients with systemic sclerosis can present with skin abnormalities, musculoskeletal changes, gastrointestinal complications, pulmonary disease, renal crisis and dry eyes and mouth.
_x000D_
Involvement of the gastrointestinal tract can occur from mouth to anus with varying degrees of severity. It can be present in those with both the diffuse and limited cutaneous forms. Most GIT manifestations result from dysmotility secondary to infiltration of the intestinal wall with fibrous tissue, and can cause life-threatening malabsorption and malnutrition.
_x000D_
Gastric emptying is delayed in 10-75% of patients and causes symptoms of early satiety, bloating and emesis. Treatments include metoclopramide and erythromycin. The small bowel is also involved in 20-60% of patients, due to reduced or absent migrating motor complexes predisposing to bacterial overgrowth. The contributes to malabsorption, as does associated pancreatic insufficiency. In the colon there is often development of diverticuli involving all layers of the intestinal wall, or constipation due to reduced motility.
_x000D_
Anterior uveitis can be associated with ankylosing spondylitis, reactive arthritis, inflammatory bowel disease, sarcoidosis and Behcet’s disease. None of these have clinical features which fit with the description above.
_x000D_
A malar, or butterfly rash, is classically associated with systemic lupus erythematosus. It involves the bridge of the nose, but spares the naso-labial folds. It is usually well demarcated and macular. It is not pathognomonic of SLE, and can be seen in pellagra and dermatomyositis, but again none of these would account for the symptoms described above.
_x000D_
Erosive arthritis is present in approximately 30% of cases of systemic sclerosis, which makes it less common than malabsorption .
_x000D_
Erythema nodosum has a variety of different causes, but is not commonly associated with systemic sclerosis.
-
Question 80 of 114
80. Question
A 45-year-old lady presents to the Outpatient Department complaining of progressively increasing lethargy, frequent micturition, intense thirst despite drinking plenty of fluids and pain and stiffness of the both upper limbs.
_x000D_
She has evidence of an arthropathy affecting the second and third metacarpophalangeal (MCP) joints of both hands with radiographic evidence of degenerative disease at these sites. She also has a 4 cm hepatomegaly.
_x000D_
Which one of the following is the most likely diagnosis?
CorrectIncorrectHint
This lady’s symptoms of lethargy, frequent micturition, intense thirst, and arthropathy at the metacarpophalangeal (MCP) joints, along with hepatomegaly, suggest a systemic condition affecting multiple organ systems. Here is an analysis of the possible diagnoses:
_x000D_
Correct Answer: B. Haemochromatosis. Haemochromatosis is a condition characterized by excessive iron accumulation in the body, leading to symptoms such as arthropathy, particularly affecting the metacarpophalangeal (MCP) joints, and hepatomegaly due to iron deposition in the liver. This lady’s systemic symptoms, including lethargy and frequent micturition, could also be explained by haemochromatosis-related organ damage.
_x000D_
The other options are incorrect for the following reasons:
_x000D_
- _x000D_
- A. Rheumatoid arthritis with amyloidosis: While rheumatoid arthritis can cause metacarpophalangeal (MCP) joint involvement, the presence of hepatomegaly and systemic symptoms like lethargy and frequent micturition are not typical features of rheumatoid arthritis or secondary amyloidosis.
- C. Gout: Gout could explain the arthropathy but is less likely to cause hepatomegaly or the systemic symptoms described.
- D. Osteoarthritis: Osteoarthritis typically affects the distal interphalangeal joints and is not associated with systemic symptoms or hepatomegaly.
- E. Pyrophosphate arthropathy: This condition, also known as pseudogout, can affect the metacarpophalangeal (MCP) joints but is not typically associated with systemic symptoms or hepatomegaly.
_x000D_
_x000D_
_x000D_
_x000D_
_x000D_
In summary, haemochromatosis (option B) is the most likely diagnosis given the combination of arthropathy, systemic symptoms, and hepatomegaly. The other options are less likely due to the absence of their typical clinical features in the presentation of this lady.
-
Question 81 of 114
81. Question
A 47-year-old gentleman presents to the Rheumatology Clinic with a five-month history of progressive pain, swelling, and stiffness in both wrists, 2nd , 3rd and 4th metacarpophalangeal joints (MCPJs). The symptoms are worse in morning, and it takes almost an hour to loosen up his joints.
_x000D_
There has been no recent illness and there is no personal or family history of chronic skin conditions. He smokes and drinks alcohol occasionally.
_x000D_
On examination, there is synovitis in both wrists and metacarpophalangeal joints (MCPJs). Examination of skin, nails and other joints shows no abnormality.
_x000D_
Recent Blood Tests Show:
_x000D_
_x000D_ _x000D_
_x000D_ _x000D_ Haemoglobin
_x000D_
_x000D_
_x000D_ 135 g/L
_x000D_
_x000D_
_x000D_ (115-165)
_x000D_
_x000D_
_x000D_
_x000D_ _x000D_ White Blood Cell Count
_x000D_
_x000D_
_x000D_ 8.4 ×109/L
_x000D_
_x000D_
_x000D_ (4-11)
_x000D_
_x000D_
_x000D_
_x000D_ _x000D_ Neutrophils
_x000D_
_x000D_
_x000D_ 5.3 ×109/L
_x000D_
_x000D_
_x000D_ (1.5-7.0)
_x000D_
_x000D_
_x000D_
_x000D_ _x000D_ Platelet Count
_x000D_
_x000D_
_x000D_ 285 ×109/L
_x000D_
_x000D_
_x000D_ (150-400)
_x000D_
_x000D_
_x000D_
_x000D_ _x000D_ Erythrocyte Sedimentation Rate (ESR)
_x000D_
_x000D_
_x000D_ 46 mm/hr
_x000D_
_x000D_
_x000D_ (0-20)
_x000D_
_x000D_
_x000D_
_x000D_ _x000D_ Rheumatoid Factor
_x000D_
_x000D_
_x000D_ Positive (1:254)
_x000D_
_x000D_
_x000D_ (<30 k IU/L)
_x000D_
_x000D_
_x000D_
_x000D_ _x000D_ Serum Urea, Serum Electrolytes And Serum Creatinine
_x000D_
_x000D_
_x000D_ Normal
_x000D_
_x000D_
_x000D_
_x000D_
_x000D_
What should be the most appropriate first step for long-term management of his pathology?
CorrectIncorrectHint
The symptoms, signs, and blood results described above suggest a diagnosis of rheumatoid arthritis (RA).
_x000D_
Rheumatoid arthritis is a complex disease which has many manifestations, and management should attempt to address all aspects. The British Society of Rheumatology has published guidelines on the management of RA within the first two years.
_x000D_
These state that a diagnosis of RA should be made as early as possible, on the basis of persistent joint inflammation affecting at least three joints with early morning stiffness of at least 30 minutes. Such patients should have rapid access to a specialist team who can help to plan care, including training patients to self-manage some aspects of their disease. Specialist rheumatology nurses have a critical role to play within this team, as do physiotherapists, podiatrists, occupational therapists, and occasionally psychologists.
_x000D_
If treatment is warranted, the first step is DMARD therapy (disease modifying anti-rheumatic drugs). All patients should have their disease and its impact assessed and documented at onset, prior to starting DMARD therapy. Treatment should then be started as soon as possible after the diagnosis is made, ideally within three months of the onset of persistent symptoms.
_x000D_
The most commonly used DMARD is methotrexate. Such treatment should be part of an aggressive package of care, including escalating doses, intra-articular steroids, parenteral methotrexate and combination therapy. Once established on DMARD therapy, all patients should have a formal assessment of treatment response, or lack of it, in order to justify continuing therapy or changing it. These assessments can include measurement of CRP and disease activity scores such as DAS28. Remission should be documented prior to reducing therapy.
_x000D_
In addition to managing the musculoskeletal aspects of disease, it must not be forgotten that RA is a significant independent risk factor for ischaemic heart disease, and other risk factors should also be aggressively controlled.
_x000D_
Systemic steroids can have an important role in establishing disease control, or bridging gaps between different DMARD therapies, but long-term use is not justified. Intra-articular corticosteroid injections may be considered for symptom control during a mono- or oligo-articular flare of RA. This is a useful adjunct to therapy but is not appropriate long-term management in itself.
_x000D_
Long-term used of NSAIDs should be at the lowest effective dose and should be avoided in those with high cardiovascular risk.
_x000D_
Surgery may be indicated if any of the following persist, despite optimum medical therapy:
_x000D_
- _x000D_
- persistent pain as a result of joint or soft tissue damage
- worsening joint function
- progressive deformity
- persistent localised synovitis
- imminent or actual tendon rupture
- nerve compression, or
- stress fracture
_x000D_
_x000D_
_x000D_
_x000D_
_x000D_
_x000D_
_x000D_
_x000D_
There is little evidence for the long-term efficacy of dietary change, or complementary therapies, although a Mediterranean diet should be recommended. Patients should be helped to contact support organisations. The role of fatigue should be recognised and managed. Aerobic exercise should be encouraged.
_x000D_
Over recent years, biological agents (especially inhibitors of tumour necrosis factor) have been developed. In the United Kingdom, anti-tumour necrosis factor (TNF) agents are used for the treatment of RA only if the patient fails to respond to an adequate trial of two DMARDs (eg.lefunomide, methotrexate, sulfasalazine). However, even with these, the frequency and degree of responses are restricted. Newer agents are therefore being introduced, including rituximab (anti-CD20), abatacept (cytotoxic T-lymphocyte antigen 4 immunoglobulin) and tocilizumab (anti-interleukin 6 receptor).
-
Question 82 of 114
82. Question
A 55-year-old gentleman presents to the General Physician with dry eyes, a dry mouth, an erythematous rash and polyarthralgia for last three months.
_x000D_
Investigations Show:
_x000D_
_x000D_ _x000D_
_x000D_ _x000D_ Anti-Nuclear Antibody (ANA)
_x000D_
_x000D_
_x000D_ Strongly positive (1:1600)
_x000D_
_x000D_
_x000D_
_x000D_ _x000D_ Anti-Ro/SSA Antibodies
_x000D_
_x000D_
_x000D_ Strongly positive
_x000D_
_x000D_
_x000D_
_x000D_ _x000D_ Rheumatoid Factor
_x000D_
_x000D_
_x000D_ Positive
_x000D_
_x000D_
_x000D_
_x000D_ _x000D_ IgG
_x000D_
_x000D_
_x000D_ 48 g/L (<15)
_x000D_
_x000D_
_x000D_
_x000D_ _x000D_ IgM
_x000D_
_x000D_
_x000D_ Normal
_x000D_
_x000D_
_x000D_
_x000D_ _x000D_ IgA
_x000D_
_x000D_
_x000D_ Normal
_x000D_
_x000D_
_x000D_
_x000D_ _x000D_ Kappa/Lambda Ratio
_x000D_
_x000D_
_x000D_ Normal
_x000D_
_x000D_
_x000D_
_x000D_
_x000D_
What is the most likely diagnosis?
CorrectIncorrectHint
The clinical features and the serology are typical of primary Sjögren’s syndrome (occurs alone and more likely to have positive anti Ro SSA antibodies than secondary Sjögren’s).
_x000D_
Hypergammaglobulinaemia is present in 80% of individuals.
_x000D_
The normal kappa/lambda ratio confirms the hypergammaglobulinaemia is polyclonal. Autoantibodies include rheumaroid factors, antinuclear antibodies, and multiple organ-specific antibodies (inc. antigastric parietal cells, thyroglobulin thyroid microsomal, mitochondrial, smooth muscle and salivary duct antibodies). Antibodies to Ro (SS-A) and La (SS-B) are also common. ANA and anti-Ro/SSA antibodies are present in approximately 90% of individuals as is a weakly positive rheumatoid factor.
_x000D_
Typically secondary Sjögren’s has pre-existent rheumatoid or systemic lupus erythematosus before the development of Sjögren’s symptoms.
-
Question 83 of 114
83. Question
A 55-year-old gentleman presents with a three week history of malaise and lower limb joint pain, associated with a vasculitic rash over his shins, thighs and buttocks.
_x000D_
Investigations Revealed:
_x000D_
_x000D_ _x000D_
_x000D_ _x000D_ Haemoglobin
_x000D_
_x000D_
_x000D_ 100 g/L
_x000D_
_x000D_
_x000D_ (115-165)
_x000D_
_x000D_
_x000D_
_x000D_ _x000D_ Platelet Count
_x000D_
_x000D_
_x000D_ 278 ×109/L
_x000D_
_x000D_
_x000D_ (150-400)
_x000D_
_x000D_
_x000D_
_x000D_ _x000D_ Serum Creatinine Concentration
_x000D_
_x000D_
_x000D_ 450 µmol/L
_x000D_
_x000D_
_x000D_ (60-110)
_x000D_
_x000D_
_x000D_
_x000D_ _x000D_ Antinuclear Antibodies
_x000D_
_x000D_
_x000D_ Negative
_x000D_
_x000D_
_x000D_
_x000D_ _x000D_ Antineutrophil Cytoplasmic Antibodies
_x000D_
_x000D_
_x000D_ Negative
_x000D_
_x000D_
_x000D_
_x000D_ _x000D_ Antiglomerular Basement Membrane Antibodies
_x000D_
_x000D_
_x000D_ Negative
_x000D_
_x000D_
_x000D_
_x000D_ _x000D_ Dipstick Urinalysis
_x000D_
_x000D_
_x000D_ Blood 3+; Protein 1+
_x000D_
_x000D_
_x000D_
_x000D_
_x000D_
What is the most likely diagnosis?
CorrectIncorrectHint
The distribution of the rash together with lower limb joint pains and renal involvement are most suggestive of Henoch-Schönlein purpura (HSP).
_x000D_
HSP is a small vessel vasculitis mediated by IgA-immune complex deposition. It is characterised by the tetrad of:
_x000D_
- _x000D_
- purpura
- abdominal pain
- arthritis, and
- renal involvement (haematuria and proteinuria).
_x000D_
_x000D_
_x000D_
_x000D_
_x000D_
The diagnosis can be easily missed, and a high degree of suspicion is required. Skin biopsy and immunofluorescence demonstrate leukocytoclastic vasculitis with IgA deposition, which is pathognomonic for HSP.
_x000D_
90% of cases of HSP occur in children aged 2-10 years but can occur in any age group. It is typically commoner in males, and may follow an infectious agent. An important risk factor for the development of HSP in adults is thought to be chronic alcohol intake. A variety of disorders have been associated with HSP in adults, including Helicobacter pylori, hepatitis B and malignancy. In some cases, overlap with polyangiitis or polyarteritis-like disease is seen.
_x000D_
Management of HSP in adults often involves the use of immunomodulatory or immune-suppressive regiments (in contrast to children where the majority of cases resolve spontaneously). There is often a more complicated course in adults, and 50% of patients who present with renal involvement develop renal insufficiency. In addition to renal disease, cardiac, pulmonary, ocular, gastrointestinal and neurological complications have been described.
_x000D_
Amyloidosis is a clinical disorder caused by extracellular and or intracellular deposition of insoluble abnormal amyloid fibrils, which alter normal tissue function. It is classified chemically and is associated with a number of different conditions. There is typically a combination of symptoms which affect more than one system, including massive proteinuria, peripheral neuropathy, hepatomegaly and heart failure. A vasculitic rash affecting the lower limbs is not typical.
_x000D_
Haemolytic ueaemic syndrome (HUS) is a triad of microangiopathic haemolytic anaemia, thrombocytopenia and acute renal failure. It is most commonly associated with Escherichia coli O157:H7. The classical presentation is profuse diarrhoea with blood, and the absence of this in the above scenario makes HUS unlikely.
_x000D_
Membranous nephropathy is a histological diagnosis and usually presents with proteinuria without haematuria. 85% of cases are idiopathic, and the remainder are secondary to autoimmune conditions (SLE), infection (hepatitis B), drugs (captopril, NSAIDs) and malignancy.
_x000D_
In myeloma there is malignant proliferation of plasma cells, which produces marrow infiltration and overproduction of a monoclonal antibody detectable in serum and/or urine (Bence Jones protein). Common presenting features include pathological fractures, anaemia, anorexia, bruising and infection. Myeloma can rarely cause vasculitis which is antineutrophil cytoplasmic antibody (ANCA) negative but this is rare and less likely than HSP in this scenario.
-
Question 84 of 114
84. Question
A 55-year-old gentleman complaining of a three day history of malaise, subjective fever, sweating, nausea, abdominal cramps and foul smelling diarrhoea is brought in to the Emergency Department by ambulance
_x000D_
His past medical history is significant for Crohn’s Disease, which has been inactive for two years following the initiation of immunosuppressive therapy. He was commenced on Allopurinol a month ago, after suffering another flare of his gout.
_x000D_
Clinical examination demonstrated: he is confused, clinically dehydrated. His Glasgow Coma Scale Score (GCS) is 14; temperature is 95.2°F; pulse rate is 100 beats per minute and regular; blood pressure is 100/60 mmHg; chest sounds are normal; abdomen has generalised tenderness in all quadrants with hyperkinetic bowel sounds.
_x000D_
Blood Tests Showed:
_x000D_
_x000D_ _x000D_
_x000D_ _x000D_ Haemoglobin
_x000D_
_x000D_
_x000D_ 122 g/L
_x000D_
_x000D_
_x000D_ (130-180)
_x000D_
_x000D_
_x000D_
_x000D_ _x000D_ Mean Corpuscular Volume (MCV)
_x000D_
_x000D_
_x000D_ 91 fL
_x000D_
_x000D_
_x000D_ (80-96)
_x000D_
_x000D_
_x000D_
_x000D_ _x000D_ Total Leucocyte Count
_x000D_
_x000D_
_x000D_ 1.8 ×109/L
_x000D_
_x000D_
_x000D_ (4-11)
_x000D_
_x000D_
_x000D_
_x000D_ _x000D_ Neutrophils
_x000D_
_x000D_
_x000D_ 1.2 ×109/L
_x000D_
_x000D_
_x000D_ (1.5-7)
_x000D_
_x000D_
_x000D_
_x000D_ _x000D_ Lymphocytes
_x000D_
_x000D_
_x000D_ 0.9 ×109/L
_x000D_
_x000D_
_x000D_ (1.5-4)
_x000D_
_x000D_
_x000D_
_x000D_ _x000D_ Platelet Count
_x000D_
_x000D_
_x000D_ 54 ×109/L
_x000D_
_x000D_
_x000D_ (150-400)
_x000D_
_x000D_
_x000D_
_x000D_ _x000D_ Serum Creatinine
_x000D_
_x000D_
_x000D_ 152 µmol/L
_x000D_
_x000D_
_x000D_ (60-110)
_x000D_
_x000D_
_x000D_
_x000D_ _x000D_ Serum Urea
_x000D_
_x000D_
_x000D_ 9.1 mmol/L
_x000D_
_x000D_
_x000D_ (2.5-7.5)
_x000D_
_x000D_
_x000D_
_x000D_ _x000D_ Serum Alanine Aminotransferase (ALT)
_x000D_
_x000D_
_x000D_ 52 U/L
_x000D_
_x000D_
_x000D_ (5-35)
_x000D_
_x000D_
_x000D_
_x000D_ _x000D_ Serum Amylase
_x000D_
_x000D_
_x000D_ 74 U/L
_x000D_
_x000D_
_x000D_ (60-180)
_x000D_
_x000D_
_x000D_
_x000D_ _x000D_ Serum C-Reactive Protein (CRP)
_x000D_
_x000D_
_x000D_ 11 mg/L
_x000D_
_x000D_
_x000D_ (<10)
_x000D_
_x000D_
_x000D_
_x000D_ _x000D_ Westergren Erythrocyte Sedimentation Rate (ESR)
_x000D_
_x000D_
_x000D_ 26 mm/hr
_x000D_
_x000D_
_x000D_ (0 – 20)
_x000D_
_x000D_
_x000D_
_x000D_ _x000D_ Serum Uric Acid
_x000D_
_x000D_
_x000D_ 195 mmol/L
_x000D_
_x000D_
_x000D_
_x000D_
_x000D_
What is the most likely underlying cause of his presentation?
CorrectIncorrectHint
This questions aims to cover:
_x000D_
- _x000D_
- Basic mechanism of action and pharmacokinetics of allopurinol
- Basic mechanism of action of azathioprine, and
- Potential consequence of their interaction.
_x000D_
_x000D_
_x000D_
_x000D_
This patient has developed bowel sepsis (foul smelling diarrhoea, hypothermic, tachycardic, hypotensive) secondary to pancytopenia induced by the co-administration of allopurinol and azathioprine.
_x000D_
The prodrug azathioprine is metabolised to its active compound 6-mercaptopurine (6-MP). 6-MP undergoes catabolic oxidation to 6-thiouric acid by xanthine oxidase. Allopurinol has a peak onset of action of one to two weeks and works by inhibiting xanthine oxidase. Co-administration of these drugs may lead to accumulation of 6-MP and increases the risk of myelosuppression. The newer xanthine oxidase inhibitor, febuxostat, can also result in the same problem and is also contraindicated.
_x000D_
A Crohn’s flare would not typically cause pancytopenia.
_x000D_
Normal amylase refutes pancreatitis.
-
Question 85 of 114
85. Question
A 54-year-old lady is suffering from a pain in her left knee for the past seven years. She has no joint swelling or history of trauma. She states that initially the pain used to get better on movement but recently it has just became the opposite whilst exercise now exacerbates the pain. She denies weight loss or significant history of fractures. She does not smoke or drinks alcohol and has no family history of bone disease.
_x000D_
Which one of the following is most probably the underlying disease process?
CorrectIncorrectHint
The most likely underlying disease process in this case is D. Osteoarthritis.
_x000D_
Osteoarthritis is a degenerative joint disease that often affects weight-bearing joints like the knees. The key findings that support osteoarthritis as the diagnosis are:
_x000D_
- _x000D_
- Knee pain: The lady has been suffering from left knee pain for an extended period of 7 years, which is a typical presentation of osteoarthritis.
- No history of trauma: The absence of a history of trauma or injury makes osteoarthritis more likely, as it is a degenerative condition rather than a traumatic one.
- Initial improvement with movement, later worsening: In the early stages of osteoarthritis, movement can help alleviate stiffness and pain. However, as the disease progresses, exercise and movement can exacerbate the pain due to increased joint stress and inflammation.
- No joint swelling: While swelling can occur in advanced osteoarthritis, the absence of swelling does not rule out the diagnosis, especially in the earlier stages.
- Age: This lady is 54 years old, which is within the age range where osteoarthritis becomes more prevalent, particularly in weight-bearing joints like the knees.
_x000D_
_x000D_
_x000D_
_x000D_
_x000D_
_x000D_
Osteoarthritis is characterised by the gradual breakdown of cartilage within the joints, leading to pain, stiffness, and reduced mobility. The absence of risk factors for other bone diseases, such as osteomalacia, osteoporosis, or osteopetrosis, further supports osteoarthritis as the most likely diagnosis.
_x000D_
The other options are incorrect because:
_x000D_
- _x000D_
- Osteomalacia: While the worsening of pain with exercise could be a feature of osteomalacia, the absence of risk factors like vitamin D deficiency and the long duration of symptoms make osteomalacia less likely.
- Osteochondroma: Osteochondromas are benign bone tumours that typically present as painless masses or swellings around joints, which is not consistent with the patient’s presentation.
- Osteopetrosis: Osteopetrosis is a rare inherited disorder characterized by increased bone density, and it is usually diagnosed in childhood, making it an unlikely cause of knee pain in a 54-year-old lady.
- Osteoporosis: While osteoporosis can increase the risk of fractures, it is not typically associated with chronic joint pain, especially in the absence of a history of fractures or significant risk factors.
_x000D_
_x000D_
_x000D_
_x000D_
_x000D_
In summary, the progressive nature of the knee pain, the absence of risk factors for other conditions, and the characteristic initial improvement with movement followed by worsening pain with exercise strongly suggest that osteoarthritis is the most probable underlying disease process in this case.
_x000D_
This inquiry assesses comprehension of disease prevalence and the relative significance of specific symptoms and signs in distinguishing diseases. It is crucial to recognise that patients seldom exhibit the classic textbook symptom descriptions, whether in exams or clinical settings. Among the provided options, osteoarthritis appears to be the most probable diagnosis.
-
Question 86 of 114
86. Question
A 54-year-old gentleman presented to the Outpatient Department with an eight week history of general malaise and a three day history of a left foot drop, right ulnar nerve palsy and a widespread purpuric rash.
_x000D_
He complained of arthralgia but had no clinical evidence of inflammatory joint disease.
_x000D_
Investigations Revealed:
_x000D_
_x000D_ _x000D_
_x000D_ _x000D_ Erythrocyte Sedimentation Rate (ESR)
_x000D_
_x000D_
_x000D_ 98 mm/hr
_x000D_
_x000D_
_x000D_ (0-20)
_x000D_
_x000D_
_x000D_
_x000D_ _x000D_ Anti-Neutrophil Cytoplasmic Antibodies (ANCA)
_x000D_
_x000D_
_x000D_ Negative
_x000D_
_x000D_
_x000D_
_x000D_ _x000D_ Antinuclear Antibody (ANA)
_x000D_
_x000D_
_x000D_ Negative
_x000D_
_x000D_
_x000D_
_x000D_ _x000D_ Rheumatoid Factor
_x000D_
_x000D_
_x000D_ Strongly Positive
_x000D_
_x000D_
_x000D_
_x000D_ _x000D_ C3
_x000D_
_x000D_
_x000D_ 0.7 g/L
_x000D_
_x000D_
_x000D_ (0.75-1.6)
_x000D_
_x000D_
_x000D_
_x000D_ _x000D_ C4
_x000D_
_x000D_
_x000D_ 0.03 g/L
_x000D_
_x000D_
_x000D_ (0.14-0.5)
_x000D_
_x000D_
_x000D_
_x000D_ _x000D_ Urine Dipstick
_x000D_
_x000D_
_x000D_ Blood ++; No Protein
_x000D_
_x000D_
_x000D_
_x000D_
_x000D_
An Electrocardiogram (ECG) was normal and three sets of blood cultures were sterile.
_x000D_
Which one of the following is the most likely diagnosis?
CorrectIncorrectHint
The history is strongly suggestive of systemic vasculitis with mononeuritis multiplex, purpuric rash and haematuria.
_x000D_
It is important to exclude conditions which can mimic vasculitis such as infective endocarditis. The normal echocardiogram and negative blood cultures make this unlikely.
_x000D_
Whilst polyarteritis nodosa can present with exactly this clinical picture, the marked consumption of C4 together with a strongly positive rheumatoid factor strongly suggests cryoglobulinaemia as the underlying cause.
_x000D_
Cryoglobulins are immunoglobulins which precipitate in the cold. They can be
_x000D_
- _x000D_
- Type I (monoclonal)
- Type II (mixed monoclonal and polyclonal) or
- Type III (polyclonal).
_x000D_
_x000D_
_x000D_
_x000D_
Type I cryoglobulinaemia is associated with haematological diseases such as myeloma and Waldenstrom’s.
_x000D_
Type II and Type III cryoglobulinaemia can be associated with many connective tissue disorders, chronic infections and most importantly, hepatitis C infection which should always be excluded.
_x000D_
Treatment of cryoglobulinaemia would include plasmaphoresis, high dose steroids and cyclophosphamide.
_x000D_
The following conditions may be associated with a positive rheumatoid factor:
_x000D_
- _x000D_
- Rheumatoid arthritis – 26 to 90%
- Sjögren’s syndrome – 75 to 95%
- Mixed connective tissue disease – 50 to 60%
- Mixed cryoglobulinemia (types II and III) – 40 to 100%
- Systemic lupus erythematosus – 15 to 35%
- Polymyositis/dermatomyositis – 5 to 10%
_x000D_
_x000D_
_x000D_
_x000D_
_x000D_
_x000D_
-
Question 87 of 114
87. Question
A 52-year-old lady underwent a bone densitometry examination, which demonstrated a bone mass decreased more than two standard deviations below the mean for her age in her right femoral head, wrist, and lumbar vertebral region.
_x000D_
Eight months later the amount of bone loss was seen to be increased by repeat densitometry examination.
_x000D_
With which of the following serum laboratory test abnormalities these findings are most likely to be associated?
CorrectIncorrectHint
She has osteoporosis with decreased bone mass. Most cases do not have a specific aetiology, but Cushing’s syndrome with hypercortisolism can promote osteoporosis. Her age should make one suspicious.
_x000D_
Hypoparathyroidism is not going to accelerate bone loss. The bone resorption that accompanies hyperparathyroidism can cause osteoporosis.
_x000D_
Over 95% of cases of osteoporosis are ‘primary’ with unknown cause. Elevated serum globulin should make you suspect a monoclonal gammopathy, but myeloma leads to focal bone lytic lesions.
_x000D_
Hyperuricaemia can be associated with gout that can cause focal bone destruction near affected joints, the bone mass overall is not decreased.
-
Question 88 of 114
88. Question
A 52-year-old gentleman is suffering from rheumatoid arthritis for three years. His recent complete blood count (CBC) shows:
_x000D_
_x000D_ _x000D_
_x000D_ _x000D_ Haemoglobin
_x000D_
_x000D_
_x000D_ 112 g/L
_x000D_
_x000D_
_x000D_ (120-165)
_x000D_
_x000D_
_x000D_
_x000D_ _x000D_ Platelet Count
_x000D_
_x000D_
_x000D_ 474 ×109/L
_x000D_
_x000D_
_x000D_ (150-450)
_x000D_
_x000D_
_x000D_
_x000D_ _x000D_ Total Leucocyte Count
_x000D_
_x000D_
_x000D_ 9.2 ×109/L
_x000D_
_x000D_
_x000D_ (4-10)
_x000D_
_x000D_
_x000D_
_x000D_ _x000D_ Mean Corpuscular Volume (MCV)
_x000D_
_x000D_
_x000D_ 101 fL
_x000D_
_x000D_
_x000D_ (83-95)
_x000D_
_x000D_
_x000D_
_x000D_
_x000D_
Which one of the drugs is most likely responsible for his complete blood count (CBC) as shown above?
CorrectIncorrectHint
This gentleman’s complete blood count (CBC) results show mild anaemia, thrombocytosis, and macrocytosis. These findings can be associated with certain medications used in the treatment of rheumatoid arthritis (RA). Let us evaluate the options:
_x000D_
- _x000D_
- Leflunomide – Incorrect. Leflunomide can cause liver enzyme elevations and diarrhoea but is not typically associated with the haematological changes seen in this gentleman.
- Myocrisin – Incorrect. Myocrisin, or gold salts, can cause various side effects including renal toxicity, dermatitis, and stomatitis, but they are not known to cause macrocytosis.
- Hydroxychloroquine – Incorrect. Hydroxychloroquine is associated with retinal toxicity but not typically with haematological abnormalities.
- Methotrexate – Correct. Methotrexate can cause macrocytic anaemia due to folate deficiency, which can also lead to thrombocytosis as a reactive process.
- Ciclosporin – Incorrect. Ciclosporin is more commonly associated with renal dysfunction and hypertension, not the haematological profile observed here.
_x000D_
_x000D_
_x000D_
_x000D_
_x000D_
_x000D_
Therefore, the most likely drug responsible for the haematological changes in this gentleman is D. Methotrexate. It is known to cause macrocytic anaemia, and the thrombocytosis could be a secondary reactive process to the anaemia. This gentleman’s physician might consider evaluating his folate levels and discussing the potential need for folate supplementation.
-
Question 89 of 114
89. Question
A 51-year-old lady presents with an increasing backache and left hip pain for the past eight years. The pain is worse at the end of the day. She is otherwise healthy and has no significant medical history.
_x000D_
She has bony enlargement of the distal interphalangeal joints. X-ray of the spine reveals the presence of prominent osteophytes involving the vertebral bodies. There is sclerosis with narrowing of the joint space at the right acetabulum seen on a radiograph of the pelvis.
_x000D_
Which one of the following pathologic processes is most likely to be taking place in this lady?
CorrectIncorrectHint
The most likely diagnosis for this lady described is Osteoarthritis (A). This condition is characterized by the breakdown of joint cartilage and underlying bone, most commonly due to age and wear and tear on a joint. The symptoms described, such as backache and hip pain that worsens by the end of the day, along with the radiographic findings of prominent osteophytes and joint space narrowing, are classic signs of osteoarthritis.
_x000D_
The other options are not correct because:
_x000D_
- _x000D_
- Rheumatoid arthritis (B) is less likely because it is an autoimmune disorder that typically presents with symmetrical joint involvement and systemic symptoms like fatigue and fever, which are not mentioned in the case. Moreover, rheumatoid arthritis usually affects the proximal interphalangeal joints rather than the distal ones.
_x000D_
_x000D_
- _x000D_
- Gout (C) is a type of inflammatory arthritis caused by the deposition of urate crystals in the joint, leading to sudden, severe episodes of pain and swelling. The chronic and gradual nature of the pain described does not fit the typical acute presentation of gout.
_x000D_
_x000D_
- _x000D_
- Osteomyelitis (D) is an infection of the bone that can cause pain, but it would be unusual for it to present without fever or other systemic signs of infection. Additionally, osteomyelitis would likely show different radiographic features, such as areas of bone destruction, which are not mentioned.
_x000D_
_x000D_
- _x000D_
- Lyme disease (E) can cause joint pain, but it is typically associated with a history of tick bites and a characteristic skin rash, neither of which is described in the case. Lyme disease also tends to cause migratory arthralgias rather than the chronic, localised pain seen here.
_x000D_
_x000D_
In conclusion, the clinical presentation and radiographic findings strongly suggest that osteoarthritis is the most likely pathologic process occurring in this lady.
-
Question 90 of 114
90. Question
A 50-year-old lady presents to the pain management clinic with an eight-month history of gradually worsening left knee pain, swelling, and restricted movements. She has no history of injury, or recent infections. She is not on any regular medication, and there is no significant past medical history or family history of note.
_x000D_
On examination, her left knee is grossly swollen, warm, non-tender, and there is restricted flexion and extension. The synovial fluid aspirated from the left knee is brown stained.
_x000D_
Which one of the following is the most likely diagnosis?
CorrectIncorrectHint
Given the clinical presentation and examination findings, the most likely diagnosis for this lady with a swollen, warm, non-tender knee and brown-stained synovial fluid is Pigmented villonodular synovitis (PVNS). Pigmented villonodular synovitis (PVNS) is characterized by joint effusions, expansion of the synovium, and bony erosions, often presenting with hemosiderin-laden fluid which can appear brown. The non-tender nature of the swelling and the chronicity of symptoms align with pigmented villonodular synovitis (PVNS), which is a locally aggressive neoplastic synovial disease.
_x000D_
The other options can be ruled out based on the following:
_x000D_
- _x000D_
- Reactive arthritis – Typically presents with symptoms of joint pain, eye inflammation, and urethritis following an infection. This lady has no history of recent infections, and the presentation is not consistent with reactive arthritis, which often affects multiple joints and is associated with systemic symptoms.
- Acute calcium pyrophosphate crystal arthritis (pseudogout) – This condition usually presents with acute attacks of joint pain and swelling, often affecting the knee, but the synovial fluid is typically less discoloured and the presentation is more acute rather than chronic.
- Gout – Gout is characterized by sudden, severe attacks of pain, redness, and tenderness in joints, most often the big toe, and is associated with elevated levels of uric acid. The chronic nature and specific synovial fluid findings in this case do not align with gout.
- Meniscal tear – A meniscal tear would typically present with a history of injury or trauma, and the pain is often localized to a specific area of the knee with possible clicking or locking sensations. The diffuse swelling and lack of injury history make a meniscal tear less likely.
_x000D_
_x000D_
_x000D_
_x000D_
_x000D_
Therefore, based on the provided information, option A. Pigmented villonodular synovitis is the most likely diagnosis.
-
Question 91 of 114
91. Question
A 51-year-old gentleman visits the general physician with complaint of increasing pain at both thumb bases over the last four months. He is a gardener by profession and his pain is aggravated while he is at work in the gardens.
_x000D_
Over the last few months he has noticed firm to hard swelling on several distal and proximal interphalangeal joints which were painful at onset, but are relatively painless now. He denies any episode of early morning stiffness.
_x000D_
On examination, there are hard (bony) swellings on several distal and proximal interphalangeal joints, and squaring of the thumbs. A complete blood count (CBC), erythrocyte sedimentation rate (ESR), and serum C-reactive protein (CRP) done by the general physician have been normal.
_x000D_
Which one of the following is the most likely diagnosis?
CorrectIncorrectHint
The most likely diagnosis for the gentleman described is E. Osteoarthritis (OA). Osteoarthritis (OA) is characterized by pain at the base of the thumb, especially during activities that involve gripping or pinching, and the presence of hard, bony swellings on the distal and proximal interphalangeal joints. The lack of early morning stiffness and normal inflammatory markers (CBC, ESR, and CRP) also support this diagnosis, as osteoarthritis (OA) is a non-inflammatory type of arthritis.
_x000D_
The other options are less likely for the following reasons:
_x000D_
- _x000D_
- Tophaceous gout: This condition typically presents with episodes of severe pain, redness, and swelling, often starting in the big toe but can affect other joints. The presence of tophi (firm swellings due to urate crystal deposition) could be a feature, but this gentleman’s normal inflammatory markers and the chronic nature of the symptoms without acute flares make gout less likely.
- Rheumatoid arthritis: Rheumatoid arthritis (RA) is a systemic inflammatory condition that usually presents with symmetrical joint involvement and significant morning stiffness lasting more than an hour. The normal erythrocyte sedimentation rate (ESR) and C-reactive protein (CRP) levels, along with the absence of morning stiffness, make RA an unlikely diagnosis.
- Psoriatic arthritis: This type of arthritis is associated with psoriasis and can present with dactylitis (“sausage” fingers) and nail changes. This gentleman’s lack of skin symptoms, nail changes, and the pattern of joint involvement do not fit with psoriatic arthritis.
- RS3PE syndrome: Remitting seronegative symmetrical synovitis with pitting oedema (RS3PE) is characterized by sudden onset of pitting oedema of the hands and feet, symmetrical arthritis, and a good response to corticosteroids. This gentleman’s lack of oedema and asymmetrical joint involvement make this diagnosis unlikely.
_x000D_
_x000D_
_x000D_
_x000D_
_x000D_
In conclusion, the clinical presentation and normal laboratory results are most consistent with E. Osteoarthritis (OA), which is a common condition in individuals who perform repetitive thumb and hand movements, such as gardeners
-
Question 92 of 114
92. Question
A 51-year-old gentleman presented to the general physician with complaint of difficulty in swallowing, hard calcified nodules in the fingers, and cold hands for about last seven years.
_x000D_
Examination revealed calcified nodules, sclerodactyly, and facial telangiectasia.
_x000D_
Which one of the following antibodies is most likely to be found in his blood?
CorrectIncorrectHint
The symptoms described for this gentleman are indicative of CREST syndrome, which is a variant of systemic sclerosis. The correct antibody most likely to be found in his blood is:
_x000D_
- _x000D_
- Anticentromere antibodies: These are commonly associated with CREST syndrome, which includes the symptoms of calcinosis, Raynaud’s phenomenon, oesophageal dysmotility, sclerodactyly, and telangiectasia.
_x000D_
_x000D_
Let us rule out the other options:
_x000D_
- _x000D_
- Anticardiolipin antibodies: These are associated with antiphospholipid syndrome, which is characterized by blood clots and pregnancy complications, not the symptoms described in the case.
- Antimyeloperoxidase antibodies: Also known as p-anti-neutrophil cytoplasmic antibodies (p-ANCA), these are typically associated with microscopic polyangiitis and other vasculitides, not with CREST syndrome.
- Anti-DNA antibodies: These are strongly associated with systemic lupus erythematosus (SLE), particularly with lupus nephritis. They are not typically found in patients with CREST syndrome.
- Antimitochondrial antibodies: These are most commonly associated with primary biliary cholangitis, a chronic liver disease, and are not related to the symptoms of CREST syndrome.
_x000D_
_x000D_
_x000D_
_x000D_
_x000D_
In summary, the presence of anticentromere antibodies (Option B) is the most consistent with the clinical presentation of CREST syndrome, which aligns with the symptoms in this gentleman.
-
Question 93 of 114
93. Question
A 50-year-old lady is referred to the outpatient clinic with a five day history of a painful swollen right knee. She drinks 30 units of alcohol per week. Her father has psoriasis.
_x000D_
Physical examination demonstrates nail pitting. Erythrocyte sedimentation rate (ESR) is elevated at 88 mm/hr. Her total leucocyte count is normal.
_x000D_
Which one of the following is the next most appropriate step in her management?
CorrectIncorrectHint
The correct option is B. Joint aspiration and analysis of synovial fluid for Gram stain, microscopy and culture. This is because the lady has signs and symptoms suggestive of septic arthritis, which is a medical emergency that requires prompt diagnosis and treatment. Septic arthritis is an infection of the joint space caused by bacteria, fungi, or viruses. It can cause severe joint damage and systemic complications if left untreated. The lady has risk factors for septic arthritis, such as alcohol abuse, psoriasis, and nail pitting. She also has a painful swollen knee, which is the most common site of septic arthritis, and an elevated erythrocyte sedimentation rate (ESR), which indicates inflammation. The definitive diagnosis of septic arthritis is made by joint aspiration and analysis of synovial fluid, which can reveal the causative organism and guide antibiotic therapy.
_x000D_
The other options are incorrect because:
_x000D_
- _x000D_
- A. Commence allopurinol: This is incorrect because allopurinol is a medication that reduces the production of uric acid in the body. It is used to treat gout, kidney stones, and high levels of uric acid in cancer patients. It is not indicated for septic arthritis, which is caused by an infection, not by uric acid crystals. Allopurinol can also cause serious side effects, such as rash, liver toxicity, and hypersensitivity syndrome.
- C. Commence non-steroidal anti-inflammatory drugs (NSAIDs): This is incorrect because non-steroidal anti-inflammatory drugs (NSAIDs) are medications that reduce pain and inflammation by inhibiting the enzyme cyclooxygenase. They are used to treat various conditions, such as arthritis, headache, and menstrual cramps. They are not effective for septic arthritis, which requires antibiotic therapy to eradicate the infection. Non-steroidal anti-inflammatory drugs (NSAIDs) can also cause adverse effects, such as gastrointestinal bleeding, renal impairment, and cardiovascular events.
- D. Check rheumatoid factor, anticyclic citrullinated peptide antibody and knee radiograph: This is incorrect because these tests are used to diagnose rheumatoid arthritis, which is a chronic autoimmune disease that causes joint inflammation and damage. Rheumatoid arthritis is unlikely in this case, as the lady has only one affected joint, a normal leucocyte count, and a family history of psoriasis, which is more associated with psoriatic arthritis. Moreover, these tests are not urgent and can be delayed until septic arthritis is ruled out.
- E. Commence oral steroids and a disease-modifying antirheumatic drug (DMARD): This is incorrect because oral steroids and disease-modifying antirheumatic drugs (DMARDs) are medications that suppress the immune system and reduce inflammation. They are used to treat various autoimmune diseases, such as rheumatoid arthritis, psoriatic arthritis, and systemic lupus erythematosus. They are contraindicated in septic arthritis, as they can worsen the infection and mask the symptoms. They can also cause serious side effects, such as infection, osteoporosis, diabetes, and hypertension.
_x000D_
_x000D_
_x000D_
_x000D_
-
Question 94 of 114
94. Question
A 50-year-old lady attends the clinic for a routine health check-up. She is apparently in good health, fit and well, with no complication.
_x000D_
Clinical examination was normal but her biochemistry shows that she has a serum urate concentration of 0.52 mmol/L (0.25-0.45).
_x000D_
Which one of the following is the most appropriate management for this lady?
CorrectIncorrectHint
The most appropriate management for this lady is B. Lifestyle advice. This is because she has asymptomatic hyperuricaemia, which means she has a high serum urate concentration but no symptoms or signs of gout. Asymptomatic hyperuricaemia is not an indication for pharmacological treatment, as it has not been shown to be beneficial and may cause adverse effects. Instead, lifestyle advice can help lower the serum urate concentration and reduce the risk of developing gout in the future. Lifestyle advice includes:
_x000D_
- _x000D_
- Avoiding or limiting foods and drinks that are high in purines, such as red meat, seafood, organ meats, beer, and wine.
- Drinking plenty of water to prevent dehydration and facilitate urate excretion34
- Maintaining a healthy weight and avoiding rapid weight loss, as obesity and fasting can increase serum urate concentration.
- Controlling other medical conditions that may affect serum urate concentration, such as hypertension, diabetes, and dyslipidaemia.
_x000D_
_x000D_
_x000D_
_x000D_
_x000D_
The other options are less appropriate for the following reasons:
_x000D_
- _x000D_
- A. Start colchicine: Colchicine is an anti-inflammatory drug that is used to treat acute gout attacks and prevent recurrent gout attacks. However, it is not indicated for asymptomatic hyperuricaemia, as it does not lower the serum urate concentration and may cause gastrointestinal side effects.
- C. Start allopurinol: Allopurinol is a xanthine oxidase inhibitor that lowers the serum urate concentration by blocking the synthesis of uric acid. It is the first-line drug for urate-lowering therapy in patients with gout. However, it is not recommended for asymptomatic hyperuricaemia, as it may trigger gout attacks, cause hypersensitivity reactions, and interact with other drugs.
- D. Start prednisolone: Prednisolone is a glucocorticoid that has anti-inflammatory and immunosuppressive effects. It is used to treat acute gout attacks and other inflammatory conditions. However, it is not suitable for asymptomatic hyperuricaemia, as it does not lower the serum urate concentration and may cause serious side effects such as osteoporosis, diabetes, infections, and adrenal suppression.
- E. Start diclofenac: Diclofenac is a non-steroidal anti-inflammatory drug (NSAID) that is used to treat acute gout attacks and other painful conditions. However, it is not indicated for asymptomatic hyperuricaemia, as it does not lower the serum urate concentration and may cause gastrointestinal bleeding, renal impairment, and cardiovascular events.
_x000D_
_x000D_
_x000D_
_x000D_
_x000D_
Therefore, the correct answer is B. Lifestyle advice.
_x000D_
-
Question 95 of 114
95. Question
A 50-year-old gentleman with rheumatoid arthritis visits the general physician complaining of a sore throat. His current medications include methotrexate, folic acid, adcal-D3, ibuprofen and paracetamol.
_x000D_
On examination he is found to have enlarged tonsils with pus, tender cervical lymphadenopathy and a fever of 101.3°F. He has no cough.
_x000D_
Which one of the following options represents the most appropriate treatment plan for this gentleman?
CorrectIncorrectHint
The most appropriate treatment plan for this gentleman, considering his symptoms and current medications, would be option D: Send an urgent venous blood sample for complete blood count and commence phenoxymethylpenicillin 500 mg four times daily for 10 days. This option is suitable because this gentleman’s symptoms suggest a bacterial infection, possibly streptococcal tonsillitis, which requires antibiotic treatment. Phenoxymethylpenicillin is effective against the bacteria commonly responsible for tonsillitis. Additionally, the blood count can help assess for neutropenia, which is a concern given the patient’s use of methotrexate.
_x000D_
Let us rule out the other options:
_x000D_
- _x000D_
- Admit the gentleman to hospital as an emergency with suspected neutropaenic sepsis: This option might be considered if this gentleman were showing signs of systemic infection or if neutropenia was confirmed. However, immediate hospital admission may not be necessary without evidence of neutropenia or sepsis.
- Give the gentleman advice about self-management of sore throat and advise to return if he does not improve in the next five days: This option is not appropriate because the presence of pus on the tonsils and fever suggests a bacterial infection that likely requires antibiotic treatment.
- Commence benzylpenicillin 500 mg four times daily for 10 days: While benzylpenicillin could be used to treat bacterial tonsillitis, it is typically administered intravenously or intramuscularly, not orally, which makes it less practical for outpatient treatment.
- Send an urgent venous blood sample for complete blood count and give the patient advice about self-management of sore throat: This option does not address the need for antibiotics, which this gentleman likely requires given the clinical presentation.
_x000D_
_x000D_
_x000D_
_x000D_
_x000D_
It is important to note that while methotrexate can increase the risk of myelosuppression, leading to neutropaenia, this is relatively rare. However, given the potential severity of neutropaenic sepsis, it is prudent to check this gentleman’s blood count urgently. The gentleman should be monitored closely, and if there is any indication of neutropaenia or worsening of symptoms, he should seek immediate medical attention.
-
Question 96 of 114
96. Question
A 50-year-old gentleman with increasing shortness of breath over an eight week period of time is referred to the pulmonology clinic for an urgent review. He is a smoker and has a history of longstanding rheumatoid arthritis.
_x000D_
Lung function tests demonstrate a marked fall in the FEV1 which is noticeably lower than the tests last taken three months back. The residual volume (RV) is increased by three litres but the measurements of diffusion are normal.
_x000D_
Which one of the following is the most likely diagnosis?
CorrectIncorrectHint
The correct answer is B. Bronchiolitis obliterans. This is the most likely diagnosis for this gentleman with rheumatoid arthritis presenting with a progressive and significant fall in FEV1. This condition is characterised by inflammation and fibrosis of the small airways, leading to obstructive spirometry and air trapping, which is reflected in the increased residual volume (RV). This gentleman’s history of rheumatoid arthritis and the rapid decline in lung function are consistent with bronchiolitis obliterans, which can be confirmed histologically by the presence of mural concentric narrowing of the bronchioles.
_x000D_
The other options are incorrect for the following reasons:
_x000D_
- _x000D_
- Chronic obstructive pulmonary disease (COPD): While chronic obstructive pulmonary disease (COPD) could present with a decrease in FEV1 and is associated with smoking, it typically progresses more slowly than described in the case of this gentleman. Additionally, chronic obstructive pulmonary disease (COPD) does not have a direct association with rheumatoid arthritis.
- Acute interstitial pneumonitis: This condition is usually related to drug treatment for rheumatoid arthritis, such as methotrexate. It presents with a rapid onset and progression, which does not align with this gentleman’s eight-week history of symptoms.
- Atypical pneumonia: This gentleman’s prolonged history of symptoms and the absence of acute infectious signs make atypical pneumonia an unlikely diagnosis. Moreover, atypical pneumonia would not typically cause a marked fall in FEV1 or a significant increase in residual volume (RV).
- Caplan’s syndrome: This syndrome is associated with rheumatoid arthritis and exposure to coal dust, leading to the development of rheumatoid nodules in the lungs. However, the rapid progression of lung function decline and the absence of pneumoconiosis make this diagnosis less likely compared to bronchiolitis obliterans.
_x000D_
_x000D_
_x000D_
_x000D_
_x000D_
In summary, this gentleman’s symptoms, history, and lung function tests point towards bronchiolitis obliterans as the most likely diagnosis, which is further supported by the potential for rapid progression and response to corticosteroids. The other conditions listed do not align as closely with the clinical presentation and lung function test results.
-
Question 97 of 114
97. Question
A 50-year-old gentleman has had arthritis for past four months and plans to visit a Rheumatology Clinic for a review. He has morning stiffness lasting for about one hour. His hands, wrists, left elbow and knees are swollen. He also complains of painful feet.
_x000D_
Recent Investigations show an Erythrocyte Sedimentation Rate (ESR) of 40 mm/hour and Serum C-Reactive Protein (CRP) is 32 mg/L. Complete Blood Count (CBC) is within normal range.
_x000D_
Which antibody test should be requested if it is suspected that he had an early rheumatoid arthritis?
CorrectIncorrectHint
High titres of antinuclear antibodies (ANA) are associated with a large number of autoimmune diseases, most commonly systemic lupus erythematosus (SLE).
_x000D_
Anticyclic citrullinated peptide antibodies (anti-CCP antibodies) are highly specific and sensitive for rheumatoid arthritis and their titre correlates with erosive disease. Anticyclic citrullinated peptide antibodies should be used as one of the first line immunological investigations in suspected rheumatoid arthritis.
_x000D_
Antineutrophil cytoplasmic antibodies (ANCA) are more commonly associated with vasculitides which the history does not suggest in this case.
_x000D_
Antiphospholipid antibodies would be requested when the clinical presentation suggests a diagnosis of antiphospholipid syndrome.
_x000D_
Complement is not an antibody test. Levels are generally performed in specific cases, for example, cryoglobulinaemia and SLE.
-
Question 98 of 114
98. Question
A 49-year-old gentleman presents to his General Physician with complaints of generalised weakness, easy fatigability and tiredness.
_x000D_
He has recently noted that his eyes feel dry and grainy, and he requires water to swallow his food. Examination shows that there is salivary gland enlargement in his neck.
_x000D_
Biochemistry Shows:
_x000D_
_x000D_ _x000D_
_x000D_ _x000D_ Haemoglobin
_x000D_
_x000D_
_x000D_ 125 g/L
_x000D_
_x000D_
_x000D_ (115-165)
_x000D_
_x000D_
_x000D_
_x000D_ _x000D_ Total Leucocyte Count
_x000D_
_x000D_
_x000D_ 7.5 ×109/L
_x000D_
_x000D_
_x000D_ (4-11)
_x000D_
_x000D_
_x000D_
_x000D_ _x000D_ Neutrophils
_x000D_
_x000D_
_x000D_ 71%
_x000D_
_x000D_
_x000D_ (40-75)
_x000D_
_x000D_
_x000D_
_x000D_ _x000D_ Platelet Count
_x000D_
_x000D_
_x000D_ 265 ×109/L
_x000D_
_x000D_
_x000D_ (150-400)
_x000D_
_x000D_
_x000D_
_x000D_ _x000D_ Erythrocyte Sedimentation Rate (ESR)
_x000D_
_x000D_
_x000D_ 35 mm/hr
_x000D_
_x000D_
_x000D_ (0-20)
_x000D_
_x000D_
_x000D_
_x000D_ _x000D_ Antinuclear Antibody (ANA)
_x000D_
_x000D_
_x000D_ Positive (1:80)
_x000D_
_x000D_
_x000D_ (Negative at 1:20 Dil)
_x000D_
_x000D_
_x000D_
_x000D_ _x000D_ Anti-Centromere Antibody (ACA)
_x000D_
_x000D_
_x000D_ Negative
_x000D_
_x000D_
_x000D_ (Negative at 1:40 Dil)
_x000D_
_x000D_
_x000D_
_x000D_ _x000D_ Anti-Scl-70 Antibody
_x000D_
_x000D_
_x000D_ Negative
_x000D_
_x000D_
_x000D_ (Negative)
_x000D_
_x000D_
_x000D_
_x000D_ _x000D_ Anti-U1RNP Antibody
_x000D_
_x000D_
_x000D_ Negative
_x000D_
_x000D_
_x000D_ (Negative)
_x000D_
_x000D_
_x000D_
_x000D_ _x000D_ Anti-Ro/La Antibody
_x000D_
_x000D_
_x000D_ Positive
_x000D_
_x000D_
_x000D_ (Negative)
_x000D_
_x000D_
_x000D_
_x000D_ _x000D_ Urea, Serum Electrolytes And Serum Creatinine
_x000D_
_x000D_
_x000D_ Normal
_x000D_
_x000D_
_x000D_
_x000D_
_x000D_
Which one of the following is the most likely diagnosis?
CorrectIncorrectHint
This gentleman has Sjögren’s syndrome.
_x000D_
Other causes of dry eyes, and/or dry mouth include:
_x000D_
- _x000D_
- past head and neck radiation
- hepatitis C infection
- acquired immunodeficiency disease
- pre-existing lymphoma
- sarcoidosis
- graft versus host disease, or
- the use of an anticholinergic drugs.
_x000D_
_x000D_
_x000D_
_x000D_
_x000D_
_x000D_
_x000D_
_x000D_
Patients are at a higher risk of developing lymphoma (non-Hodgkin’s lymphoma [NHL] B cell), and should be monitored for this.
-
Question 99 of 114
99. Question
A 49-year-old lady presented to the Emergency Department with a two-day history of bilateral swollen ankles. She is known to have a history of an acute episode of gout affecting her right first Metatarsophalangeal Joint (MTPJ).
_x000D_
She had recently been travelling in the Northern Europe and returned a day before she presented to the Emergency Department. During her trip she had had one episode of night sweats which she attributed to the possibility of having caught flu, but she mentioned that she had noticed feeling less fit than previously during her regular activities.
_x000D_
Clinically, she had low grade pyrexia of 99.8°F but she was otherwise well. Complete Blood Count (CBC) showed an elevated Erythrocyte Sedimentation Rate (ESR) of 38 mm/hr and a Serum C-Reactive Protein (CRP) of 15 mg/L. Her Complete Blood Count (CBC), Renal Profile Test and Liver Function Tests (LFTs) were otherwise unremarkable.
_x000D_
Physical examination revealed normal abdominal, chest and cardiovascular findings. She was found to have three painful patches of raised rash over her knees and there were no features of synovial inflammation elsewhere.
_x000D_
Which one of the following would be the most appropriate next investigation?
CorrectIncorrectHint
This lady has bilateral swollen ankles with raised erythrocyte sedimentation rate (ESR) and mildly raised C reactive protein (CRP), some constitutional symptoms and a rash suggestive of erythema nodosum. Sarcoidosis is therefore high on the list of differentials.
_x000D_
The typical erythema nodosum (EN) rash consists of a sudden onset of symmetrical, tender, erythematous, warm nodules and raised plaques usually located on the shins, ankles and knees. The description above should therefore lead you to consider EN.
_x000D_
Joint aspirate is not the next best investigation as the patient is not clinically septic and has bilateral swelling which decreases the likelihood of septic arthritis.
_x000D_
Blood cultures are not clinically appropriate currently, as the patient is afebrile and not clinically septic.
_x000D_
A chest x ray may show bilateral hilar lymphadenopathy to guide towards a diagnosis of sarcoidosis and is the most appropriate investigation listed here.
_x000D_
The history is not typical of gout and urate levels will not alter treatment nor is this a diagnostic test.
_x000D_
The results of a skin biopsy will take up to a week, and it is therefore not the most appropriate first line investigation. In general, a biopsy will only be taken if the clinical diagnosis is in doubt. If done the histopathology will show a mostly septal panniculitis, with no vasculitis. The septa of the subcutaneous fat are thickened, and variously infiltrated by inflammatory cells that extend to the periseptal areas of the fat lobules. The composition of the inflammatory infiltrate in the septa varies with the age of the lesion1. Miescher’s radial granulomas (relatively small histiocytes radially placed around a central cleft) may be present, which are a characteristic marker of erythema nodosum2.
-
Question 100 of 114
100. Question
A 49-year-old gentleman, who works as a builder, presents with progressively increasing lower back pain for the last seven months. His pain gradually increases during his activities in the morning, and gets worse in the evening. He has no history of weight loss, night pain, or fever. His back remains stiff for about twenty minutes in the morning.
_x000D_
Over the last few months he has developed firm to hard swelling on several distal and proximal interphalangeal joints, and has anterior knee pain worsened by climbing stairs. A Complete Blood Count (CBC), Erythrocyte Sedimentation Rate (ESR), and Serum C-Reactive Protein (CRP) done by his General Physician are normal.
_x000D_
Which one of the following is the most likely diagnosis?
CorrectIncorrectHint
This gentleman has generalised osteoarthritis (GOA), as there are OA related symptoms in at least three joint areas, namely:
_x000D_
- _x000D_
- Bony swellings at distal and proximal IPJs termed Heberden’s and Bouchard’s nodes respectively
- Anterior knee pain, worse on climbing stairs, suggesting patella-femoral joint OA, and
- Low back pain, suggesting spinal degenerative changes.
_x000D_
_x000D_
_x000D_
_x000D_
Ankylosing spondylitis typically occurs in young men, and associates with pronounced early morning stiffness and buttock pain.
_x000D_
Osteoporosis is not symptomatic, unless accompanied by a spinal fracture. Osteoporotic spinal fractures present with acute pain which improves over a period of few weeks to a couple of months.
_x000D_
There are no red-flag symptoms to raise the possibility of discitis or malignancy.
-
Question 101 of 114
101. Question
A 48-year-old lady with longstanding Insulin Dependent Diabetes Mellitus (IDDM) presents to the Pain Management Clinic with a three week history of a severely painful, erythematosus, swollen right mid-foot for the last three weeks. She does not drink alcohol, and has had no recent history of injuries to the foot.
_x000D_
On examination, her mid-foot is warm and pedal pulses are intact. There is a symmetrical sensory loss in both of her hands and feet bilaterally. Recent blood tests show a normal Complete Blood Count (CBC), Serum C-Reactive Protein (CRP), Serum Urea, Serum Electrolytes and Serum Creatinine.
_x000D_
Which one of the following is the most likely diagnosis?
CorrectIncorrectHint
In patients with longstanding diabetes and peripheral neuropathy, a red hot swollen foot should raise suspicion of Charcot neuroarthropathy.
_x000D_
Charcot neuropathy presents as a warm, swollen, erythematous foot and ankle, and infection is important to exclude. The majority of patients are in their 50-60s, and they often present in the latter stages of the disease.
_x000D_
It can occur in association with a variety of conditions, including leprosy, poliomyelitis, and rheumatoid arthritis, although today the most common cause is diabetes mellitus.
_x000D_
The pathophysiology of Charcot neuroarthropathy is not completely understood, but is thought to start with peripheral neuropathy. The lack of pain sensation may mean that patients subject the foot joints (commonly the midfoot) to stress injuries that lead to the Charcot process. It is important to note however that about half of patients present with pain.
_x000D_
Four stages of Charcot neuropathy are recognised:
_x000D_
- _x000D_
- Stage 0 (inflammation) – characterised by erythema and oedema, but no structural changes
- Stage 1 (development) – bone resorption, fragmentation and joint dislocation. Swelling, warmth and erythema persist but there are also radiographic changes such as debris formation at the articular margins, osseous fragmentation, and joint disruption
- Stage 2 (coalescence) – bony consolidation, osteosclerosis, and fusion are all seen on plain radiographs
- Stage 3 (reconstruction) – osteogenesis, decreased osteosclerosis, progressive fusion. Healing and new bone formation occur, and the deformity becomes permanent.
_x000D_
_x000D_
_x000D_
_x000D_
_x000D_
Radiographs are an important part of investigating a patient with possible Charcot arthropathy. All radiographs should be taken in the weight-bearing position.
_x000D_
MRI can demonstrate changes in the earlier stages of the condition, and is therefore important in allowing treatment to be instigated earlier.
_x000D_
In stages 0 and 1, the treatment is immediate immobilisation and avoidance of weight bearing. A total-contact cast is worn until the redness, swelling and heat subside (generally 8-12 weeks, changed every 1-2 weeks to minimise skin damage). After this, the patient should use a removable brace for a total of four to six months.
_x000D_
Bisphosphonates can be used, but evidence of clinical benefit is lacking. Surgery is reserved for severe deformities that are susceptible to ulceration, and where braces and orthotic devices are difficult to use.
_x000D_
A normal FBC and CRP in this case make cellulitis unlikely. There is no swelling of the calf to suggest a deep vein thrombosis.
_x000D_
Fragility fractures are those which are caused by a force equivalent to a fall from the height of a chair or less. They are typically seen on a background of osteoporosis and there is usually a history of trauma.
_x000D_
Gout classically causes an acute monoarthritis and the presentation is typically more acute than described here.
-
Question 102 of 114
102. Question
A 48-year-old lady with a poor glycaemic control presents to the Clinic with a three day history of painful swollen left knee and difficulty in walking.
_x000D_
She has had no recent infections, does not drink alcohol, and there is no significant past medical or family history.
_x000D_
On examination, she is febrile with a temperature of 100°F. The left knee is warm, tender, and there is a tense left knee effusion. Knee flexion is painful and restricted.
_x000D_
Which one of the following is the most appropriate investigation for this lady?
CorrectIncorrectHint
Septic arthritis must be excluded in an individual with acute mono-arthritis.
_x000D_
Septic arthritis may be oligo- or polyarticular in the immunosuppressed, and may present without pyrexia. Joint aspiration, followed by microscopy and culture of the synovial fluid is critical to the diagnosis of septic arthritis.
_x000D_
Examination of joint fluid under polarised microscope may show monosodium urate (negatively birefringent) or calcium pyrophosphate (weakly positively birefringent) crystals, and lead to a diagnosis of acute gout or acute CPP crystal arthritis (pseudogout).
_x000D_
Knee radiograph is likely to be normal in someone with inflammatory joint symptoms of short duration.
_x000D_
A blood culture may be negative in over half of patients with septic arthritis, and a high CRP does not differentiate between causes of acute hot swollen joint.
_x000D_
Knee MRI will show a wide range of non-specific changes in this scenario, for example, effusion, synovial proliferation, bone marrow oedema, none of which is specific to septic arthritis.
_x000D_
There is no role for serum urate in the diagnosis of acute gout. Serum urate, a negative acute phase reactant (like albumin), reduces during an acute illness.
-
Question 103 of 114
103. Question
A 58-year-old lady visits the Inflammatory Arthropathy Clinic for her two month follow up.
_x000D_
She reports satisfactory symptom relief with Prednisolone 10 mg once daily after failing several other disease modifying agents. Her past medical history includes Coeliac Disease and Smith’s Fracture. She is advised by the Consultant that this may be her long term treatment of choice.
_x000D_
Which further measures are necessary before proceeding with Prednisolone, regarding preservation of bone mineral density?
CorrectIncorrectHint
The following learning points are covered by this question:
_x000D_
- _x000D_
- Bone mineral density measurement
- Indications for bisphosphonate prophylaxis with glucocorticoid therapy
- Specific bisphosphonate pharmacotherapy.
_x000D_
_x000D_
_x000D_
_x000D_
This lady has three independent risk factors for the development of osteoporosis (coeliac disease, previous fragility fracture, long term glucocorticoid therapy).
_x000D_
In the context (long term glucocorticoid therapy), due to her previous fragility fracture and irrespective of her age, this patient should be commenced on bisphosphonate therapy without the need for bone mineral density quantification with DEXA scanning.
_x000D_
Indications for bisphosphonate prophylaxis in glucocorticoid use for a period > 3 months:
_x000D_
_x000D_ _x000D_
_x000D_ _x000D_ <65 years
_x000D_
_x000D_
_x000D_ >65 year
_x000D_
_x000D_
_x000D_
_x000D_ _x000D_ Fragility fracture*
_x000D_
_x000D_
_x000D_ All patients
_x000D_
_x000D_
_x000D_
_x000D_ _x000D_ T-score below −1.5
_x000D_
_x000D_
_x000D_
_x000D_
_x000D_
NB: if T-score 0 to −1.5 repeat in 1-3 years.
_x000D_
* Fragility fracture – defined by The World Health Organisation as resulting from a mechanical force equivalent to a fall from standing height or less which should not ordinarily cause a fracture.
-
Question 104 of 114
104. Question
A 58-year-old lady presents to the pain clinic with a one week history of sharp stabbing pain radiating from her back towards the lateral aspects of her both legs. The pain is associated with sensation of uncomfortable tingling and prickling.
_x000D_
On examination, she is found to have sensory loss on the lateral aspects of her both legs, dorsa of feet, and there is also a partial left foot drop that occurs frequently and intermittently in the right foot.
_x000D_
Which lumbar spine nerve root is affected?
CorrectIncorrectHint
The correct answer is A. L5. This is the nerve root that exits the spine between the L4 and L5 vertebrae and supplies the lateral aspect of the leg, the dorsum of the foot, and the big toe. The symptoms and signs that have described here are consistent with this diagnosis, such as:
_x000D_
- _x000D_
- Sharp stabbing pain radiating from the back towards the lateral aspects of the both legs, which is typical of lumbar radiculopathy, a condition that causes pain and other neurological symptoms due to pressure on a nerve root in the lower back.
- Sensation of uncomfortable tingling and prickling, which indicates nerve damage or irritation.
- Sensory loss on the lateral aspects of the both legs and dorsa of feet, which corresponds to the dermatomal distribution of the L5 nerve root.
- Partial left foot drop and intermittent right foot drop, which indicate weakness of the muscles that are innervated by the L5 nerve root, such as the tibialis anterior, extensor hallucis longus, and extensor digitorum longus.
_x000D_
_x000D_
_x000D_
_x000D_
_x000D_
The other options are incorrect for the following reasons:
_x000D_
- _x000D_
- B. L4 nerve root exits the spine between the L3 and L4 vertebrae and supplies the medial aspect of the leg, the medial side of the foot, and the big toe. It does not cause sensory loss on the lateral aspects of the legs or dorsa of feet, and it does not cause foot drop.
- C. S1 nerve root exits the spine between the L5 and S1 vertebrae and supplies the posterior aspect of the leg, the lateral side of the foot, and the little toe. It does not cause sensory loss on the lateral aspects of the legs or dorsa of feet, and it causes foot drop only if it is combined with L5 nerve root injury.
- D. L2 nerve root exits the spine between the L1 and L2 vertebrae and supplies the anterior aspect of the thigh and the medial side of the leg. It does not cause sensory loss on the lateral aspects of the legs or dorsa of feet, and it does not cause foot drop.
- E. L3 nerve root exits the spine between the L2 and L3 vertebrae and supplies the anterior aspect of the thigh and the medial side of the knee. It does not cause sensory loss on the lateral aspects of the legs or dorsa of feet, and it does not cause foot drop.
_x000D_
_x000D_
_x000D_
_x000D_
_x000D_
_x000D_ _x000D_
_x000D_ _x000D_ Root
_x000D_
_x000D_
_x000D_ Dermatome distribution
_x000D_
_x000D_
_x000D_ Myotome distribution
_x000D_
_x000D_
_x000D_ Tendon reflex
_x000D_
_x000D_
_x000D_
_x000D_
_x000D__x000D_ _x000D_ L1
_x000D_
_x000D_
_x000D_ Skin above, and below the inguinal ligament
_x000D_
_x000D_
_x000D_ None
_x000D_
_x000D_
_x000D_ Nil
_x000D_
_x000D_
_x000D_
_x000D_ _x000D_ L2
_x000D_
_x000D_
_x000D_ Upper anterior, and medial thigh
_x000D_
_x000D_
_x000D_ Psoas hip abductors
_x000D_
_x000D_
_x000D_ Nil
_x000D_
_x000D_
_x000D_
_x000D_ _x000D_ L3
_x000D_
_x000D_
_x000D_ Mid anterior, and medial thigh
_x000D_
_x000D_
_x000D_ Psoas quadriceps
_x000D_
_x000D_
_x000D_ Patella (L3-4)
_x000D_
_x000D_
_x000D_
_x000D_ _x000D_ L4
_x000D_
_x000D_
_x000D_ Medial aspect of leg, front of knee, and lower lateral thigh
_x000D_
_x000D_
_x000D_ Tibialis anterior, extensor hallucis
_x000D_
_x000D_
_x000D_ Patella (L3 – 4)
_x000D_
_x000D_
_x000D_
_x000D_ _x000D_ L5
_x000D_
_x000D_
_x000D_ Lateral aspect of leg and dorsum of foot (except for the lateral border which is supplied by S1)
_x000D_
_x000D_
_x000D_ Extensor halluces, peroneal, gluteus medius, dorsiflexors, hamstrings
_x000D_
_x000D_
_x000D_ Plantar (L5, S1-2)
_x000D_
_x000D_
_x000D_
_x000D_ _x000D_ S1
_x000D_
_x000D_
_x000D_ Posterior lateral thigh and calf
_x000D_
_x000D_
_x000D_ Peroneal, plantar flexors
_x000D_
_x000D_
_x000D_ Ankle (S1-2)
_x000D_
_x000D_
_x000D_
_x000D_ _x000D_ S2
_x000D_
_x000D_
_x000D_ Popliteal fossa
_x000D_
_x000D_
_x000D_ Many, in combination with other nerve roots – including knee flexors
_x000D_
_x000D_
_x000D_ Ankle (S1-2)
_x000D_
_x000D_
_x000D_
_x000D_ _x000D_ S3 – 5
_x000D_
_x000D_
_x000D_ Medial buttock and perianal skin in a concentric manner with S3 most lateral, and s5 closest to the anus
_x000D_
_x000D_
_x000D_ Bladder, rectum
_x000D_
_x000D_
_x000D_ Nil
_x000D_
_x000D_
_x000D_
_x000D_
-
Question 105 of 114
105. Question
A 58-year-old gentleman with longstanding well controlled seropositive Rheumatoid Arthritis, treated with Methotrexate 20 mg weekly and Folic Acid 5 mg daily, presents to the Clinic with a cough productive of green sputum, pyrexia of 101.3°F, and severe sore throat for three days.
_x000D_
On examination, his Blood Pressure is 100/70 mmHg; SpO2 is 98% on Room Air and there is an occasional crackle at the left base. His chest X-Ray does not reveal any abnormality. Complete Blood Count (CBC), U&Es, Serum Creatinine, and Liver Function Tests (LFTs) are also normal. The Serum C-Reactive Protein (CRP) is 33 mg/L.
_x000D_
Which one of the following is the most appropriate course of action to be taken?
CorrectIncorrectHint
The most likely diagnosis in this scenario is a lower respiratory tract infection.
_x000D_
In the setting of acute infection, most DMARDs (except hydroxychloroquine) should be discontinued until the infectious process has resolved. Whilst it is possible the infection could be successfully treated with oral antibiotics, there is not an option here which includes this with stopping methotrexate therefore this is the correct answer.
_x000D_
Methotrexate is a folic acid antagonist. It can result in a pneumonitis that manifests with non-specific symptoms such as fever, fatigue, cough and dyspnoea. It is rare and the presence of a productive cough here makes the diagnosis less likely but if suspected methotrexate should be stopped immediately. Additional adverse effects include hepatotoxicity, alopecia, mouth ulcers and nausea. Folic acid should be given daily to reduce the incidence of these in patients treated with methotrexate.
_x000D_
Taken in overdose, methotrexate can cause multiple organ damage. Folinic acid is indicated and should be given intravenously as soon as it is suspected to limit toxicity. There is no suggestion of methotrexate overdose in this patient, and therefore folinic acid is not indicated.
-
Question 106 of 114
106. Question
A 58-year-old gentleman with a long history of chronic alcohol abuse presents to the rheumatology clinic with gouty chalkstones.
_x000D_
He is commenced on allopurinol but he develops severe joint pains three days later.
_x000D_
On examination, he is pyrexial with a temperature of 102.2°F, and is found to have erythematous swelling of his both hands, knees and ankles.
_x000D_
Investigations Reveal:
_x000D_
_x000D_ _x000D_
_x000D_ _x000D_ Serum Urate
_x000D_
_x000D_
_x000D_ 0.58 mmol/L
_x000D_
_x000D_
_x000D_ (0.23-0.46)
_x000D_
_x000D_
_x000D_
_x000D_ _x000D_ Serum C-Reactive Protein (CRP)
_x000D_
_x000D_
_x000D_ 152 mg/L
_x000D_
_x000D_
_x000D_ (<10)
_x000D_
_x000D_
_x000D_
_x000D_
_x000D_
Which one of the following is the most likely cause for his presentation?
CorrectIncorrectHint
The correct answer is B. Treatment with allopurinol. This gentleman is experiencing an acute gout attack following the initiation of allopurinol therapy. Allopurinol, an antihyperuricaemic agent, can precipitate gout attacks when started during or shortly after an acute episode. This gentleman’s symptoms are consistent with this complication, and the recent commencement of allopurinol is the most likely trigger.
_x000D_
The other options are not likely responsible for this presentation because:
_x000D_
- _x000D_
- A. Acute rheumatoid arthritis: While rheumatoid arthritis can cause joint inflammation, the gentleman’s presentation with a fever and acute onset following allopurinol initiation is more characteristic of a gout flare rather than rheumatoid arthritis.
- C. Acute pyrophosphate arthropathy: Also known as pseudogout, it typically affects larger joints like the knees and does not present with systemic symptoms such as fever, making it less likely in this case.
- D. Septic arthritis: This condition usually affects a single joint and is less likely to present with the polyarticular involvement seen in this gentleman.
- E. Allopurinol allergy: Although allopurinol hypersensitivity includes rash and fever, the absence of a rash and the polyarticular joint involvement following allopurinol initiation suggest an acute gout attack rather than an allergic reaction.
_x000D_
_x000D_
_x000D_
_x000D_
_x000D_
In summary, this gentleman’s symptoms are best explained by an acute gout attack triggered by the recent start of allopurinol therapy, making option B the correct answer. The other options are less likely due to the absence of their typical clinical features in this gentleman’s presentation.
-
Question 107 of 114
107. Question
A 58-year-old gentleman with Type 2 Diabetes Mellitus (T2DM) with a three month history of painful hands and feet is referred by his Endocrinologist to the Rheumatology Clinic for a consultation and review.
_x000D_
Investigations confirm a diagnosis of seropositive erosive rheumatoid arthritis. He has some pain relief from non-steroidal anti-inflammatory agents. He is currently on Metformin 500 mg thrice daily and has good glycaemic control as reflected by a HbA1c of 48 mmol/mol (20-46).
_x000D_
Which of the following Disease-Modifying Antirheumatic Drugs (DMARDs) would be most appropriate initial treatment of his early rheumatoid arthritis?
CorrectIncorrectHint
Guidance recommends the use of disease modifying anti-rheumatic drugs (DMARDs) early in the treatment of rheumatoid arthritis, maintaining function and reducing progression of the disease (SIGN 2001). First line agents include methotrexate and sulfasalazine (SIGN 2000) and most subjects receive methotrexate.
_x000D_
Generally gold is considered more toxic than the former two and hydroxychloroquine is probably less effective.
_x000D_
Ciclosporin is again rather more toxic than either methotrexate or sulfasalazine, with nephrotoxicity and immunosuppression and is generally reserved for RhA with systemic features such as vasculitis.
_x000D_
The tumour necrosis factor (TNF) alpha antagonists, etanercept and infliximab, are generally reserved for individuals unresponsive to traditional DMARDS.
-
Question 108 of 114
108. Question
A 58-year-old gentleman receiving 10 mg of methotrexate and 5 mg of folate weekly presents to the Clinic with a sore left finger following a finger prick while gardening.
_x000D_
On examination he has a swollen erythematous left ring finger up to the proximal interphalangeal joint and a cellulitis is diagnosed. As he is allergic to penicillins, he is prescribed erythromycin. He has been receiving the methotrexate for more than one year without any complain and all routine blood tests are within normal ranges.
_x000D_
While monitoring the response of the infection to treatment, which one of the following is the most appropriate plan regarding his methotrexate therapy?
CorrectIncorrectHint
In the circumstances of infection one should consider temporarily stopping methotrexate as it is an immunosuppressant.
_x000D_
Any infection should be treated as usual and the response to treatment monitored. Once the infection has been successfully treated methotrexate can be reinstated. However, if the patient has recurrent serious infections while taking methotrexate its continued long term use should be discussed with the patient’s rheumatologist.
_x000D_
*Some local variations may exist regarding dose and frequency of folate therapy. Please be aware of your local guidelines.
-
Question 109 of 114
109. Question
A 56-year-old gentleman with rheumatoid arthritis was referred to the Gastroenterology Clinic with iron deficiency anaemia.
_x000D_
Endoscopy revealed several superficial antral erosions with small bowel biopsy showing mild villous blunting, apoptotic bodies, occasional eosinophils and mild increase in chronic inflammatory cells.
_x000D_
Colonoscopy was reported as normal.
_x000D_
Which one of the following is the most likely cause of these findings?
CorrectIncorrectHint
This salient features in this patient’s case revolve around the fact that he has rheumatoid arthritis (hence the requirement for NSAIDs), the iron deficiency anaemia and the superficial ulceration on endoscopy with features indicative of inflammation due to the chronic NSAID use.
_x000D_
Coeliac disease is associated with villous atrophy and lymphocyte infiltration. There is no suggestion on the biopsy of lymphocyte infiltration which argues against lymphoma or coeliac.
-
Question 110 of 114
110. Question
A 56-year-old gentleman presents to the pain management clinic with complaints of arthritis and swelling of his bilateral limb joints over the past six months.
_x000D_
On examination, he has a symmetrical inflammation with painful movements of all of his four limbs and he also has swelling of both knees, which suggests a diagnosis of rheumatoid arthritis.
_x000D_
Which one of the following suggests an adverse prognosis regarding his joint disease?
CorrectIncorrectHint
The correct answer is A. Articular erosions on x-ray. Articular erosions are a sign of joint damage caused by rheumatoid arthritis (RA). They indicate that the inflammation has eroded the cartilage and bone of the joint, leading to deformity and disability. Articular erosions are associated with a poor prognosis and a higher risk of mortality in rheumatoid arthritis (RA).
_x000D_
The other answer options are incorrect for the following reasons:
_x000D_
- _x000D_
- B. Enthesitis: Enthesitis is the inflammation of the entheses, which are the sites where tendons or ligaments attach to bones. Enthesitis is more common in spondyloarthritis, a group of inflammatory diseases that affect the spine and joints, than in rheumatoid arthritis (RA). Enthesitis does not necessarily imply a worse prognosis in rheumatoid arthritis (RA).
- C. Elevated serum C-reactive protein (CRP): C-reactive protein (CRP) is a marker of inflammation that can be measured in the blood. Elevated C-reactive protein (CRP) levels indicate active inflammation in rheumatoid arthritis (RA), but they do not necessarily predict the severity or progression of the disease. C-reactive protein (CRP) levels can vary depending on many factors, such as infection, medication, or other conditions. C-reactive protein (CRP) levels can also decrease with effective treatment of rheumatoid arthritis (RA).
- D. Acuteness of presentation: Acuteness of presentation refers to how quickly the symptoms of rheumatoid arthritis (RA) develop. Some people may have a sudden onset of rheumatoid arthritis (RA), while others may have a gradual or insidious onset. Acuteness of presentation does not affect the prognosis of rheumatoid arthritis (RA), as long as the diagnosis and treatment are timely and appropriate.
- E. Sero-negative for rheumatoid factor: Rheumatoid factor (RF) is an antibody that can be detected in the blood of some people with rheumatoid arthritis (RA). However, not all people with rheumatoid arthritis (RA) have rheumatoid factor (RF), and some people without rheumatoid arthritis (RA) may have rheumatoid factor (RF). Therefore, rheumatoid factor (RF) is not a definitive diagnostic test for rheumatoid arthritis (RA). Sero-negative rheumatoid arthritis (RA) means that the person does not have RF in their blood. Sero-negative rheumatoid arthritis (RA) does not have a worse prognosis than sero-positive rheumatoid arthritis (RA), although it may have some differences in clinical features and response to treatment.
_x000D_
_x000D_
_x000D_
_x000D_
-
Question 111 of 114
111. Question
A 56-year-old gentleman presents to the General Physician with complaint of progressively increasing lethargy over the past seven months. He has had to give up his job as a shopkeeper, and he now takes almost 10 minutes to get up one flight of stairs and he has difficulty getting up out of couches and chairs.
_x000D_
On examination, his blood pressure is 140/80 mmHg, pulse rate is 86 and regular. His heart sounds are normal and his chest is clear. He has a clear proximal muscle weakness, with sparing of distal muscle power. His Serum Creatine Kinase is elevated at 1150 U/L.
_x000D_
Which one of the following antibodies is most likely to be elevated?
CorrectIncorrectHint
The proximal myopathy and raised creatine kinase (CK) in the absence of a violaceous rash suggests that polymyositis is the most likely diagnosis.
_x000D_
Around 20% of patients with polymyositis have anti-Jo 1 antibodies. Corticosteroids are the mainstay of immunosuppression. Patients with anti-Jo 1 antibodies are at increased risk of lung fibrosis, and therefore immunosuppression is usually continued long term. A steroid sparing agent, such as azathioprine, may well be added to reduce the corticosteroid dose.
There are a variety of other autoantibodies present in polymyositis, including anti-signal recognition particle antibody, which can be associated with different forms of the disease._x000D_
Anti-Rho and -La antibodies are seen in systemic lupus erythematosus (SLE) and are associated with the development of neonatal lupus.
_x000D_
Antismooth muscle antibodies are classically associated with autoimmune hepatitis
-
Question 112 of 114
112. Question
A 55-year-old lady with rheumatoid arthritis presents to the Clinic for a review. She is allergic to penicillin and co-trimoxazole.
_x000D_
Which of the following drugs is contraindicated in this scenario?
CorrectIncorrectHint
Both co-trimoxazole and sulphasalazine contain sulphonamide groups and hence an allergy to co-trimoxazole would be a contraindication to the use of sulphasalazine.
_x000D_
Co-trimoxazole is a mixture of trimethoprim and sulfamethoxazole.
_x000D_
Sulphasalazine is a combination of 5-aminosalicyclic acid and sulfapyridine. It is commonly used in the treatment of inflammatory bowel disease, and can also be used in rheumatoid and psoriatic arthritis.
_x000D_
Azathioprine is a purine analogue which is commonly used as a steroid-sparing agent.
_x000D_
Ciclosporin is a calcineurin inhibitor, used for the prevention of transplant rejection.
_x000D_
Gold therapy was previously used as a disease modifying agent in rheumatoid arthritis, but this has now been replaced by methotrexate which is a dihydrofolate reductase inhibitor.
-
Question 113 of 114
113. Question
A 55-year-old lady presents to the Neurology Clinic with a three month history of progressive painless weakness affecting the proximal arms and legs.
_x000D_
She has noticed difficulty getting out of a low chair and soft couch and some difficulty swallowing but denies any rashes or visual symptoms.
_x000D_
Investigations show a Serum Creatine Kinase of 4500 IU/l.
_x000D_
Which one of the following is the most likely diagnosis?
CorrectIncorrectHint
Polymyositis classically presents with relatively painless progressive proximal muscle weakness. Dysphagia is common but the ocular muscles are very rarely involved unlike myasthenia gravis where this is a predominant feature.
_x000D_
Diagnosis of polymyositis confirmed by elevated muscle enzymes (creatine kinase) and typical EMG and muscle biopsy findings.
_x000D_
PMR is characterised by marked proximal stiffness and pain but rarely weakness and the muscle enzymes are typically normal.
_x000D_
Although hypothyroidism can present with a proximal myopathy and elevated creatine kinase (CK) levels the latter are rarely elevated above 500IU/l and dysphagia would not be typical.
_x000D_
Guillain-Barre syndrome causes demyelination and axonal degeneration, which results in acute, ascending and progressive neuropathy. 75% of patients have a history of preceding infection, usually of the respiratory and gastrointestinal tract. Mild rises in CK can be seen but they are not as marked as in polymyositis, and the progression of disease is much quicker.
-
Question 114 of 114
114. Question
A 55-year-old gentleman with a history of longstanding Type 2 Diabetes Mellitus (T2DM) presented to the Gastroenterology Clinic with hepatomegaly.
_x000D_
Investigations Revealed:
_x000D_
_x000D_ _x000D_
_x000D_ _x000D_ Serum Albumin
_x000D_
_x000D_
_x000D_ 32 g/L
_x000D_
_x000D_
_x000D_ (37-49)
_x000D_
_x000D_
_x000D_
_x000D_ _x000D_ Serum Total Bilirubin
_x000D_
_x000D_
_x000D_ 21 µmol/L
_x000D_
_x000D_
_x000D_ (1-22)
_x000D_
_x000D_
_x000D_
_x000D_ _x000D_ Serum Alkaline Phosphatase (ALP)
_x000D_
_x000D_
_x000D_ 136 U/L
_x000D_
_x000D_
_x000D_ (60-110)
_x000D_
_x000D_
_x000D_
_x000D_ _x000D_ Serum Alanine Aminotransferase (ALT)
_x000D_
_x000D_
_x000D_ 92 U/L
_x000D_
_x000D_
_x000D_ (5-35)
_x000D_
_x000D_
_x000D_
_x000D_ _x000D_ Serum Gamma-Glutamyl Transferase (GGT)
_x000D_
_x000D_
_x000D_ 128 U/L
_x000D_
_x000D_
_x000D_ (<50)
_x000D_
_x000D_
_x000D_
_x000D_ _x000D_ Serum Ferritin
_x000D_
_x000D_
_x000D_ 1445 µg/L
_x000D_
_x000D_
_x000D_ (15-300)
_x000D_
_x000D_
_x000D_
_x000D_
_x000D_
Which one of the following features would be most suggestive of a diagnosis of haemochromatosis?
CorrectIncorrectHint
This gentleman with diabetes has evidence of liver disease with grossly elevated ferritin, suggesting a diagnosis of haemochromatosis.
_x000D_
Haemachromatosis is caused by dysregulated iron homeostatis due to inappropriate increased iron absorption in the duodenum and proximal small intestine. It is an autosomal recessive hereditary condition which is associated with homozygous C282Y mutation of the HFE gene in North Europeans.
_x000D_
Increased absorption of iron results in its deposition in the organs, notably the liver, pancreas, heart, joints, skin, and pituitary. This causes cirrhosis, restrictive cardiomyopathy, diabetes mellitus, arthropathy, skin hyperpigmentation, and gonadal failure.
_x000D_
Males and females are affected equally, but females are often ‘protected’ from the clinical features by menstrual blood loss.
_x000D_
Arthropathy is relatively common. It is chronic and progressive and mildly inflammatory. There is a predilection for the MCP joints and it is often accompanied by chondrocalcinosis. Iron load is probably a major determinant but it does not usually respond to venesection.
_x000D_
Early diagnosis and treatment is critical in haemochromatosis as survival and morbidity are improved if phlebotomy is initiated prior to the development of cirrhosis.
_x000D_
Transferrin saturation is suggested as the initial screening test: a level of more than 45% warrants further investigation (less than 45% usually excludes the diagnosis). Genetic screening is then performed. If the usual C282Y HFE mutation is found this makes the diagnosis.
_x000D_
Ferritin is measured to help guide further investigation and treatment: if more than 1000 a liver biopsy should be performed and treatment initiated. If the ferritin is within normal range and the liver function tests are normal patients can be followed closely. If the C282Y HFE mutation is not present other genotypes should be looked for and if present a liver biopsy is indicated.
_x000D_
The goal of therapy is to remove excess body iron stores; it is commonly done via phlebotomy. Initially, this is weekly or twice weekly (if tolerated) venesections of 500 cm3 until the serum ferritin is less than 50 ng/mL.Transferritin saturation should also be reduced to less than 50% if possible.
_x000D_
After these goals are reached maintenance therapy is typically required three to four times per year. When iron overload and anaemia are present concomitantly chelation with desferoxamine may be required. Patients should be told to avoid vitamin C supplementation as this can enhance iron toxicity.
_x000D_
End stage liver disease, portal hypertension, and hepatocellular carcinoma (which is increased up to 200-fold) may necessitate liver transplantation. This is associated with poorer outcome compared with other indications because of increased incidence of infection and cardiac complications.
_x000D_
Haemochromatosis is classically associated with a non-migratory, rather than migratory, polyarthritis. This particularly affects the hands: in over 50% of patients there is involvement of the second and third metacarpophalangeal joints, but the proximal interphalangeal joints, knees, feet, wrists, back and neck are also commonly involved.
_x000D_
Skin pigmentation rather than a rash is more typical.
_x000D_
Myxoedema is not a feature of haemochromatosis.
_x000D_
Gynaecomastia is a feature of liver disease/cirrhosis per se and not just haemochromatosis.
_x000D_
Haemochromatosis can cause hypogonadism which can also be associated with gynaecomastia but costochondrosis is a more reliable sign.